датчик расстояния

advertisement
ПРАКТИКУМ ПО ФИЗИКЕ
ДЛЯ 8-11 КЛАССОВ
ПРОФИЛЬНОГО УРОВНЯ ОБУЧЕНИЯ
ЛАБОРАТОРНЫЕ РАБОТЫ С ПРИМЕНЕНИЕМ
ЦИФРОВЫХ ЛАБОРАТОРИЙ
АНДРЕЙ ЕВГЕНЬЕВИЧ ТАРЧЕВСКИЙ
ПОД РЕДАКЦИЕЙ
СЕРГЕЯ ДМИТРИЕВИЧА ВАРЛАМОВА
школа 179 МИОО, г. Москва
11 апреля 2011
по сравнению с версией от 24 апреля 2010 переработаны разделы:
"Обзор работ для учителя", материал "Необходимо знать…" Л.р. № 7 (она стала №1), 8, 9, 10, 11, 12, 14, 19, 20, 22, 24
добавлены работы № 5, 6, 18
Появились разделы:
-Компьютеры NOVA 5000 и программа Multilab. проблемы в работе и способы справляться с ними;
- Особенности датчиков лаборатории "Архимед".проблемы с датчиками и как с ними справляться
во многие места текста также внесены поправки и дополнения, устранены неточности.
ОБ ЭТОЙ КНИГЕ:
ЦЕЛИ ЭТОЙ РАБОТЫ:
- Предоставить созданные материалы и методические пособия в помощь коллегам,
использующим цифровые лаборатории. Сборник ориентирован на широко распространенные
лаборатории "Архимед", но приведенные работы можно с успехом выполнять и на других
лабораториях.
- Поделиться накопленным опытом и дать полезные советы по организации и обеспечению
успешной работы при проведении практикума для учащихся профильных классов.
- Проанализировать характерные трудности, возникающие при использовании лабораторий
"Архимед" и показать пути их успешного преодоления.
Представлен ряд лабораторных работ по физике по темам "Механика", "Газы",
"Электричество", "Колебания" для проведения практикума в 8 - 11 классах с углубленным
изучением предмета. Большинство работ многократно выполнялось учащимися школы №179
МИОО г. Москвы. Предложенные материалы могут использоваться на практикуме по
физике, а также при выполнении демонстраций учителем.
ОСОБЕННОСТИ РАЗРАБОТАННЫХ ПОСОБИЙ:
Каждая работа - небольшое научное исследование.
Глубоко изучается «физика явлений».
Создано, проверено, описано работающее оборудование.
Учтены характерные трудности, с которыми сталкивались школьники.
Описаны типичные ошибки при выполнении работ.
Разработаны многочисленные рекомендации для успешного выполнения работ.
Учтены особенности и проблемы, связанные с компьютерами NOVA-5000SE
и программой MULTILAB
 В каждой методичке сформулированы чёткие требования к отчёту об исследовании.
 Разработаны вспомогательные материалы в помощь учителю.
 Разработаны вспомогательные материалы для школьников.







Разрешается копирование и некоммерческое применение данного материала.
Методические пособия постоянно обновляются. Используя работу, пожалуйста, свяжитесь с
автором - возможно, появилась новая исправленная версия! Кроме того, разрабатываются
новые методические пособия. Автор готов общаться с заинтересованными людьми, будет рад
конструктивным замечаниям и контактам с коллегами, которые поделятся отлаженными
пособиями для проведения интересных работ по физике.
Тарчевский Андрей Евгеньевич
школа 179 МИОО, г. Москва
Контакты :
"vkontakte" Андрей Тарчевский
2
СОДЕРЖАНИЕ
ЗАЧЕМ НУЖНЫ ЛАБОРАТОРИИ "АРХИМЕД"
КОГДА НЕ НУЖНЫ ЛАБОРАТОРИИ "АРХИМЕД"
УЧИТЕЛЮ О РАБОТАХ С "АРХИМЕДОМ"
- НАВЫКИ ШКОЛЬНИКА ЗАСТАВЛЯТЬ ОБОРУДОВАНИЕ РАБОТАТЬ
- НАВЫКИ ШКОЛЬНИКА ПРИМЕНЯТЬ ТЕОРИЮ НА ПРАКТИКЕ
- ПРИДЁТСЯ ТРЕБОВАТЬ ОТ ДЕТЕЙ РЕАЛЬНЫХ РЕЗУЛЬТАТОВ
- РАБОТЫ ДЕЛАЮТСЯ ДОЛГО
- ПРАКТИКУМ - ДЛЯ ЗАИНТЕРЕСОВАННЫХ ЛЮДЕЙ
- ПОРЯДОК И БЕСПОРЯДОК
- КОМПЬЮТЕРЫ NOVA-5000 ВЕСЬМА ДАЛЕКИ ОТ СОВЕРШЕНСТВА
- НЕСООТВЕТСТВИЕ ДАТЧИКОВ ЗАЯВЛЕННЫМ ПАРАМЕТРАМ НА НЕКОТОРЫХ РЕЖИМАХ
ИЗМЕРЕНИЙ
- ЗАЧЕМ Я НАПИСАЛ РУКОВОДСТВА ДЛЯ НЕКОТОРЫХ РАБОТ
КОМПЬЮТЕРЫ NOVA 5000 И ПРОГРАММА MULTILAB. ПРОБЛЕМЫ В РАБОТЕ И СПОСОБЫ
СПРАВЛЯТЬСЯ С НИМИ
ОСОБЕННОСТИ ДАТЧИКОВ ЛАБОРАТОРИИ "АРХИМЕД". ПРОБЛЕМЫ С ДАТЧИКАМИ И КАК С НИМИ
СПРАВЛЯТЬСЯ
ПРАВИЛА ВЫПОЛНЕНИЯ ЛАБОРАТОРНЫХ РАБОТ
ОБЗОР РАБОТ ДЛЯ УЧИТЕЛЯ
О ГРАФИКАХ И ПРАВИЛЬНЫХ МАСШТАБАХ
О ПОСТРОЕНИИ ГРАФИКОВ С ПОМОЩЬЮ ПРОГРАММЫ "MULTILAB"
НЕОБХОДИМО ЗНАТЬ ПРИ РАБОТЕ С КОМПЬЮТЕРАМИ "NOVA"
Работа № 1
ИЗУЧЕНИЕ ЛАБОРАТОРИИ "АРХИМЕД"
Работа № 5
ИЗУЧЕНИЕ ПРОЦЕССА КРИСТАЛЛИЗАЦИИ И ПЛАВЛЕНИЯ ВЕЩЕСТВА
Работа № 6
ИЗУЧЕНИЕ НАСОСА
Работа № 8
ИЗУЧЕНИЕ ИЗОХОРНОГО ПРОЦЕССА
Работа № 9
ИЗМЕРЕНИЕ ДАВЛЕНИЯ ПАРОВ АЦЕТОНА
Работа № 9А
ИЗМЕРЕНИЕ ДАВЛЕНИЯ В ГАЗОВОЙ ЗАЖИГАЛКЕ
Работа № 10
ИЗУЧЕНИЕ ЗАВИСИМОСТИ ДАВЛЕНИЯ НАСЫЩЕННГО ПАРА ВОДЫ ОТ ТЕМПЕРАТУРЫ
Работа № 11
ИЗУЧЕНИЕ ИЗОТЕРМИЧЕСКОГО И АДИАБАТНОГО ПРОЦЕССА
Работа № 12
ИССЛЕДОВАНИЕ ПРОЦЕССА ЗАРЯДКИ И РАЗРЯДКИ КОНДЕНСАТОРА
Работа № 13
ИЗМЕРЕНИЕ ЁМКОСТИ ГАЛЬВАНИЧЕСКОГО ЭЛЕМЕНТА (АККУМУЛЯТОРА)
Работа № 14
ИЗУЧЕНИЕ ЗАВИСИМОСТИ ВНУТРЕННЕГО СОПРОТИВЛЕНИЯ ИСТОЧНИКА ТОКА ОТ ТОКА В ЦЕПИ
Работа № 15
3
ИЗМЕРЕНИЕ ИНДУКЦИИ МАГНИТНОГО ПОЛЯ ЗЕМЛИ
Работа № 17
ИЗУЧЕНИЕ ТЕМПЕРАТУРЫ И ЯРКОСТИ СВЕЧЕНИЯ НИТИ НАКАЛА ЛАМПОЧКИ
Работа № 18
ИЗМЕРЕНИЕ МАГНИТНОГО ПОЛЯ СОЛЕНОИДА
Работа № 19
ПРОВЕРКА ЗАКОНА ЭЛЕКТРОМАГНИТНОЙ ИНДУКЦИИ
Работа № 20
ИЗУЧЕНИЕ СТОЯЧЕЙ ВОЛНЫ В ТРУБЕ
Работа № 22
ИЗУЧЕНИЕ РАСТЯЖЕНИЯ ПРОВОЛОКИ
Работа № 23
ИЗУЧЕНИЕ СВОБОДНОГО ПАДЕНИЯ ТЕННИСНОГО ШАРИКА
Работа № 24
ИЗУЧЕНИЕ КОЛЕБАНИЙ МАТЕМАТИЧЕСКОГО МАЯТНИКА
ЗАЧЕМ НУЖНЫ ЛАБОРАТОРИИ "АРХИМЕД"
Применение лаборатории открывает ряд уникальных возможностей:
 "Архимед" содержит ряд редких для школы датчиков (давления, магнитного поля,
счётчик Гейгера и др.). В некоторых случаях очень удобно использовать лабораторию в
качестве измерительного прибора. Можно проводить как демонстрационный эксперимент,
так и лабораторные работы.
 Наблюдение и изучение динамических процессов.
Целый ряд простых и наглядных работ получается с датчиками давления - здесь и
изучение изохорного процесса (с расчётом "абсолютного нуля" температуры), и наблюдение
процесса испарения жидкости в закрытом сосуде, и исследование зависимости давления
насыщенного пара воды от температуры.
Множество уникальных работ можно выполнить и при изучении электричества и
магнетизма. Например, изучить и сравнить с теоретическими предсказаниями динамику
процесса разрядки конденсатора, "почувствовать" распределение напряжений и сдвиги фаз в
RLC цепочке, наблюдать резонанс в колебательном контуре.
Можно ярко проиллюстрировать "вечно обиженную" красивыми экспериментами
механику. Например, показать зависимость координаты, скорости, ускорения теннисного
шарика от времени при отскоках от пола (правда, здесь придётся говорить и о дискретной
записи данных и о неверно рассчитываемом ускорении при дифференцировании скорости).
Можно изучить колебания математического маятника, измеряя одновременно угол
отклонения и силу натяжения нити.
 Несомненное достоинство лабораторий - возможность достаточно серьёзной обработки
экспериментальных данных (выбор фрагмента графика, различные виды аппроксимации,
интегрирование и взятие производной, различные операции с графиками).
4
КОГДА НЕ НУЖНЫ ЛАБОРАТОРИИ "АРХИМЕД"
 Когда есть соблазн вместо участия в процессе исследования (включая оценку
достоверности и точности получаемых данных) самоустраниться и решить, что лаборатория
сама по себе даст верные результаты от нажимания на "кнопочки" в соответствии с
инструкцией. Коллеги, ведь это же глобальная проблема нашего времени - нежелание
глубоко вникать в "физику явлений". Сегодня многим хочется получить "компьютерный",
уже готовый результат, совсем не думая о том, какими приборами он получен и насколько
достоверен. А мы ведь хотим вырастить не "пользователя телевизора", а инженера и учёного.
А вы?
Например, в работе "Изучение вольт- амперной характеристики диода" "NOVA"
совсем неполезна. Помните, там график зависимости тока от подаваемого напряжения идёт
до 0,5 вольта около нуля, потом ток очень резко возрастает. А при измерении обратного тока
диода (а он очень мал) - надо взять другие приборы и выбрать другой масштаб графика.
Школьник должен понять, что за график он строит, честно измерять ток, повышая
напряжение например, на 0,1 вольта (а когда надо, то на меньшую величину - он сам увидит
это по ходу опыта). Построить аккуратный график на большом листе миллиметровки, с
правильным масштабом. Понять, при каком напряжении диод "открывается", увидеть это на
графике. Может быть, выполнить дополнительные измерения для обнаружения этой точки.
Понять, а при меньших напряжениях был ток нулевым или всё же не нулевым? При
измерении обратного тока он, вероятно, удивится малости тока утечки, возьмёт блок питания
вольт на 100. Позовёт учителя и под его присмотром будет работать с высоким напряжением
(раньше всё можно было трогать руками, а теперь - страшно и опасно), возьмёт
чувствительный микроамперметр и построит график обратного тока, поймёт, что он не виден
в масштабе первого графика и построит его на отдельном листе. И вот тогда он поймёт, что
такое вольт-амперная характеристика диода. Что по сравнению с этим может значить не
особенно наглядная половинка графика, извлеченного из недр компьютера "NOVA"? (график
обратного тока вообще не получится).
 При выполнении работы "Исследование влияния масляной плёнки на быстроту остывания
воды в стаканчике" есть соблазн дать новичкам- восьмиклассникам вместо термометра и
секундомера компьютер с датчиками температуры, тогда нужный красивый график
получится быстро и легко. Но сделав так, мы упустим главное – ученик не увидит и не
почувствует, как собираются данные, с какой периодичностью разумно делать измерения,
как выбрать масштабы на осях, какие числа следует там писать, а какие - не следует, как не
перепутать оси, как выбрать хороший масштаб, как соединять полученные точки (и почему
их не надо соединять, а следует провести линию наилучшего приближения). Большая работа,
куча ошибок, их исправление, получение своего научного результата и – понимание того,
как строится график. Заменить процесс познания готовым ответом из компьютера в данном
случае - плохо. А вот в следующей похожей работе - конечно, можно использовать и
цифровую лабораторию.
 Также совсем не обязательно использовать лабораторию для простых работ, таких как
изучение закона Ома, гидростатическое взвешивание (если хочется "точных" весов – гораздо
лучше приспособить школьные рычажные весы на штативе, одна из чашек весов легко
ставится в вертикальное положение). Те работы, где легко обойтись без "Архимеда", я бы
предпочёл делать традиционными "честными" способами. Тем самым не только избавимся от
компьютерных "глюков", но детям будет понятно, что и как измеряется, они научатся
работать с настоящими приборами, честно строить графики, обрабатывать данные "вручную"
и еще многим полезным вещам…
5
УЧИТЕЛЮ О РАБОТАХ С "АРХИМЕДОМ"
В ЭТОЙ СТАТЬЕ РАССМОТРЕНЫ ПРОБЛЕМЫ, С КОТОРЫМИ ВСТРЕТЯТСЯ
МНОГИЕ УЧИТЕЛЯ ПРИ ВЫПОЛНЕНИИ ДОСТАТОЧНО СЛОЖНЫХ РАБОТ
ФИЗПРАКТИКУМА. Возможно, читатель будет удивлён количеством трудностей и
неполадок, связанных с лабораторией "Архимед" и описанных ниже. Действительно, если
использовать лабораторию для простых измерений (например, просто измерить силу,
температуру или использовать датчик напряжения вместо обычного вольтметра) – всё,
вероятно, будет работать вполне успешно и достаточно надёжно. А вот если одновременно
подключить к "NOVA-5000" датчик тока и три датчика напряжения для изучения
напряжений на элементах RLC цепочки, установить максимально возможную частоту записи
(а без этого не увидеть "красивых" графиков), а после опыта провести обработку данных
(интегрирование тока, сложение графиков напряжений на элементах цепи) - лаборатория
подвергнется гораздо более суровому испытанию. Предлагаемый сборник посвящён именно
работам, многие из которых требуют выполнения относительно сложных измерений и
серьёзной обработки данных. А при этом, к сожалению, часто бывают проблемы.
У вас в школе есть лаборатории "Архимед" с компьютерами NOVA? Совесть
подсказывает, что дорогое и интересное оборудование не должно пылиться в шкафу ? Тогда вперед, смело ныряем в мутную воду физпрактикума, где, скорее всего, столкнемся с рядом
проблем. Примерно таких: (Все примеры в этом тексте - реальные факты.)
ПРОБЛЕМА № 1
НАВЫКИ ШКОЛЬНИКА ЗАСТАВЛЯТЬ ОБОРУДОВАНИЕ РАБОТАТЬ
Мы - представители последнего советского поколения. Помните, в конце прошлого
века приезжающие иностранцы удивлялись:- "Как, вы можете сами починить розетку? и
телевизор?! и унитаз?!! и машину свою сами ремонтируете?!!! А у нас это делают только
специалисты узкого профиля…"
Способность делать руками что-то, не связанное с нажатием на клавиши, существенно
уменьшилась в последние десятилетия, а дети - наши ученики, как правило, совсем неопытны
в таких делах.
Поэтому придется потратить немало сил и времени на простые и иногда досадные
вещи - один ребенок не может герметично подключить шланг датчика давления, другой не
понимает, что такое "плотно закрыть пробку", третий не может найти в учительской принтер,
на котором печатают графики, четвертый пытается подключить принтер неправильным
кабелем, пятый… Короче говоря, у каждого будут свои трудности, с которыми он может
долго и не всегда успешно справляться, а вы, учитель, не распознаете их сразу и не всегда
сможете помочь. Конечно, есть надежда, что в результате четырех часов, потраченных на
обнаружение утечки от проткнутой кем-то микроскопической дырочки в резиновой пробке
дети стали способнее. Может быть, даже осознали, что такое инженерный труд. Но долог
путь к тому, чтобы разные технические "мелочи" стали подчиняться ученику… Опыт
показал, что дети обычно беспомощны в самообеспечении работы необходимыми
"мелочами", поэтому отсутствие хорошо подобранного и заранее отлаженного оборудования
приведет к огромным потерям времени.
Придется учителю готовить и самому проверять оборудование, а затем положить его в
отдельную для каждой работы коробку, со всеми датчиками, колбочками, переходничками и
другими мелкими, но необходимыми штучками.
Полезна будет и помощь в проведении работ для новичков со стороны более опытных
старших учащихся.
ПРОБЛЕМА № 2
6
НАВЫКИ ШКОЛЬНИКА ПРИМЕНЯТЬ ТЕОРИЮ НА ПРАКТИКЕ
Вот, вот она, цель обучения! Вот десятый математический класс решает простую
систему из двух уравнений и получает семь различных ответов… Типичная картина? А вот
мы даем термометр, чтобы узнать температуру воздуха в закрытой колбе (колба в тепловом
равновесии с окружающей средой) и девочка старательно прислоняет градусник к
стеклянному боку сосуда…
Сначала я пробовал давать детям большие исследовательские работы без множества
подсказок в описаниях. Сформулировать конечную цель, наметить путь к ней. Ребята
справлялись плохо, почти бесконечно долго. Пришлось разделить большие работы на
несколько относительно простых. Пришлось также написать, где, как и какие законы
применить. А ещё описать те ошибки, которые "любят" делать дети в каждом месте работы.
Хотя я часто предлагаю школьникам вывести формулы самостоятельно, всё же конечные
результаты пришлось написать.
И всё равно работы делаются дольше, чем хочется.
ПРОБЛЕМА №3
ПРИДЁТСЯ ТРЕБОВАТЬ ОТ ДЕТЕЙ РЕАЛЬНЫХ РЕЗУЛЬТАТОВ
Принимая выполненную работу, мы стремимся подвести школьника к получению
красивого научного результата. Отчет о работе - это маленький, но настоящий научный
отчёт, который должен быть чётким, ясным, аккуратным и понятным читателю. В каждой
работе что-то исследуется и обязательно должно получиться что-то интересное. Некоторые
ребята вместо получения результата будет пытаться много говорить, делать умный вид,
делать что-то еще. Никогда не принимайте халтуру и они научатся получать результат.
В помощь учителю- материалы "Правила выполнения работ", "Построение графиков",
приведенные ниже. Очень помогают.
ПРОБЛЕМА № 4
РАБОТЫ ДЕЛАЮТСЯ ДОЛГО
Дети, выполняя исследование, сталкиваются с разными трудностями, да и не всегда
торопятся. Большую часть работ практикума непросто выполнить, оформить и обсудить с
учителем за два часа. Часто приходится разрешать детям доделывать задание дома. В таком
случае есть опасность растянуть выполнение работы навсегда, потому что дети придут на
следующее занятие с несделанной работой, забудут ее дома, один из пары исследователей
заболеет… Так можно растянуть выполнение одной работы на месяц. Известны даже люди,
умудрившиеся за целую четверть не сдать ни одной работы, однако присутствовавшие на
занятиях, занимающие место, время и силы учителя. Требуйте завершения работы в
отведенное время. Если это невозможно – требуйте отчета ДО следующего дня практикума.
Заведите специальный журнал, каждый раз записывайте в него, кто что делал и с какими
результатами, о чем и с кем вы договорились. Такой журнал поможет сохранить контроль
над процессом и существенно ускорить его.
Можно в начале практикума составить график, согласно которому на двухчасовом
занятии одна половина учащихся делает работы, другая – сдаёт. Но дети, однако, не
студенты, они ведь любят пропускать уроки по болезни… А работы обычно делают вдвоём…
Опыт показал, что дети быстро осваивают компьютер "NOVA" и не испытывают
серьезных затруднений при использовании его для обработки экспериментального
материала. До выполнения исследовательских работ следует дать специальную
ознакомительную работу с компьютером.
ПРОБЛЕМА № 5
ПРАКТИКУМ - ДЛЯ ЗАИНТЕРЕСОВАННЫХ ЛЮДЕЙ
7
Может случиться, что получение интересного результата исследования - это ваша
идея, в то время как цель ребенка - получить отметку с минимальной затратой сил. Вы
требуете ясно написанного отчёта, а он постоянно "впаривает" халтуру. Вы показываете, в
чём "неправильность", а вам дают следующую халтуру, вы снова показываете, что
исправить, а результаты измерений им уже утеряны. С обидами, "качанием прав", попытками
доказать, что "в методичке вот это написано". Выполнение одной работы растягивается на
несколько занятий и впустую расходует ваши силы и энергию. Распознайте вовремя такого
человека и уладьте ситуацию. Если он действительно не хочет работать - не допускайте его
на практикум. Работайте с теми, кому это нужно.
ПРОБЛЕМА № 6
ПОРЯДОК И БЕСПОРЯДОК
В работах много разных приборов, деталей, описаний, и всё это должно быть в
идеальном порядке. Каждая мелочь, которую сломали, потеряли, оставили на столе, бросили
не в тот ящик, может вызвать серьезную проблему при выполнении работы следующей
группой, а это надолго "загрузит" учителя в самый неподходящий момент. Я слишком часто
наблюдаю детей, которые привыкли дома сваливать наведение порядка на родителей, они же
не прочь загрузить этим и учителя. С первого занятия жёстко требуйте наведения идеального
порядка после выполнения работы.
ПРОБЛЕМА № 7
КОМПЬЮТЕРЫ NOVA-5000 И ПРОГРАММА ОБРАБОТКИ ДАННЫХ ВЕСЬМА
ДАЛЕКИ ОТ СОВЕРШЕНСТВА,
часто выходят из строя, дают сбои, не выполняют заявленных действий, что вызывает
множество проблем.
Подробнее – в разделе …" Компьютеры NOVA-5000 и программа Multilab. Проблемы в работе и
способы справляться с ними."
ПРОБЛЕМА № 8
НЕСООТВЕТСТВИЕ ДАТЧИКОВ ЗАЯВЛЕННЫМ ПАРАМЕТРАМ НА НЕКОТОРЫХ
РЕЖИМАХ ИЗМЕРЕНИЙ.
Подробнее - в разделе "Особенности датчиков лаборатории "Архимед". Проблемы с
датчиками и как с ними справляться."
ПРОБЛЕМА № 9
НЕТ ХОРОШЕЙ БИБЛИОТЕКИ ОТЛАЖЕННЫХ МЕТОДИЧЕСКИХ ПОСОБИЙ
ДЛЯ ЛАБОРАТОРИИ "АРХИМЕД"
Да, и не ожидайте, что вы откроете синюю методичку, придающуюся в комплекте к
лабораториям, найдёте подходящую работу, дадите детям инструкцию, и они быстро сделают
опыты и увидят то новое, что предполагалось, научатся тем методам, научить которым их
планировалось. Не получается так.
Во-первых, многие интересные опыты не описаны.
Во-вторых, то, что описано, обычно не получается с первого раза (и со второго тоже,
придется учителю самому подобрать и отладить оборудование. И это будет нелегко и начнёт
работать не сразу).
В-третьих, многие описания, на мой взгляд, весьма неудачны. Иногда мне казалось,
что люди, написавшие их, не делали этой работы с детьми. Я тоже так умею писать. И тоже с
первого раза получается плохо. Хорошая методичка создаётся после неоднократной проверки
и корректировки её при реальном выполнении работ школьниками. Вот почему мне
пришлось написать свои методички ко многим работам. Готов ими поделиться.
8
К сожалению, пока (октябрь 2010) не организован единый открытый удобный сайт,
где желающие могли бы поместить и скачать описания работ, оставить комментарии к
каждой из них, задать вопросы и обменяться опытом.
В англоязычном Интернете тоже не удалось найти подобных материалов, там, как
водится, предлагают купить методички.
ЗАЧЕМ Я НАПИСАЛ РУКОВОДСТВА ДЛЯ НЕКОТОРЫХ РАБОТ
Потому что по теме "Газы" работ в методичке для "NOVA" нет.
Потому что, когда я давал детям описания из методички к NOVA, ребята испытывали
слишком много проблем при выполнении работ, а также не получали разумных результатов.
Потому что в тех описаниях, что есть в книгах или на сайтах, далеко не всегда видна физика
явления, часто непонятно, зачем делаем то и это, школьники испытывают трудности из-за
методических недочетов, да и язык казенный…
За этими описаниями работ стоит три года физпрактикума с учащимися класса 179. Хочу
выразить признательность ребятам, выполнявшим неотлаженные работы и сталкивавшимися
с многочисленными трудностями - именно выявленные проблемы помогли найти более
прямой путь к получению красивых научных результатов.
Особая благодарность Сергею Дмитриевичу Варламову за множество ценных идей и
редактирование этого сборника.
9
КОМПЬЮТЕРЫ NOVA-5000 И ПРОГРАММА MULTILAB.
ПРОБЛЕМЫ В РАБОТЕ И СПОСОБЫ СПРАВЛЯТЬСЯ С
НИМИ. ОБОБЩЕНИЕ ПРАКТИЧЕСКОГО ОПЫТА.
Обычно лаборатория работает и радует учащихся интересными результатами. Но не все так
хорошо с компьютерами и датчиками, как хотелось бы. Есть ряд проблем, отнимающих
существенное время при выполнении работ:
1. Компьютеры NOVA-5000 SX (Ver 02.09), к сожалению, весьма ненадёжны и часто
выходят из строя.
ПОДРОБНЕЕ:
Из сорока новых проверенных (далеко не на всех режимах) компьютеров нашей
школы на семи из них сразу обнаружились проблемы с запуском программы
"Multilab" или с подключением принтера и внешних устройств.
Из 32 компьютеров NOVA-5000 SX (Ver 02.09) за два года работы:
4- сгорели, на 8- не запускается система, на 2- не работает Multilab, на 6 Multilab запускается, но отчаянно "глючит", 1- не работает с внешними
устройствами 1- страшно свистит, 1- не показывает изображения (белый
экран), 1- не работает кнопка выключения, 1- тормозит так, что работа с
ним невозможна, 8- работают нормально, но вот надолго ли?
СОВЕТЫ:
- Не пугайтесь. Это нормально. И это не ваша вина.
- Иногда удаётся восстановить заводские установки или перезагрузить систему.
(Новая версия системы на старый компьютер не устанавливается.)
- Компьютеры в школах - на гарантии сроком два года. Отправьте факс от школы в
службу ремонта вычислительной техники (ООО «РЕНЕ», г. Москва), приедут
специалисты и через месяц вернут Ваш компьютер, возможно, с улучшенной
"начинкой".
- Будьте очень внимательны с кабелями (подробнее- ниже) – это одна из частых
причин выхода компьютеров из строя.
- Необходимо пронумеровать все имеющиеся компьютеры и организовать в
специальный журнал для записи выявленных неисправностей и сбоев программы для
каждого компьютера.
- Надо откладывать неисправные компьютеры, кабели и датчики в отдельный ящик.
Неисправное оборудование надо сразу пометить, прикрепить бирку с указанием
неисправности. При этом стоит убедиться, что проблема вызвана компьютером, а не
неправильными действиями учащихся с ним.
- При выполнении работ на NOVA-5000 следует сразу сохранять полученные ценные
экспериментальные результаты в файл (а особо важные - на "флешку".)
10
2. На NOVА-5000 и датчиках установлены крайне неудачные разъёмы для подключения
кабелей, связывающих компьютер с датчиками. Неаккуратное подсоединение кабеля легко
приводит к изгибу и замыканию его штырьков, а затем "сгорают" порты всех компьютеров, в
которые этот кабель включают.
СОВЕТЫ:
- ПЕРЕД ВКЛЮЧЕНИЕМ ЛЮБОГО КАБЕЛЯ В КОМПЬЮТЕР ИЛИ В ДАТЧИК
УБЕДИТЕСЬ, ЧТО ШТЫРЬКИ РАЗЪЁМА НЕ ПОГНУТЫ И НЕ ЗАМКНУТЫ.
Кабели "с замкнутыми ножками", которые пытались вставлять в компьютер в
неправильном положении, впоследствии сжигают порты компьютеров!
- Если у группы экспериментаторов не работает компьютер, проверьте в первую
очередь кабели подсоединения датчиков. Пока не проверен кабель, не давайте
экспериментаторам другой компьютер, а то они сожгут и его.
- Предупредите об особенностях разъёмов всех учащихся.
- Можно также отметить краской на кабелях и на компьютерах верное положение
разъёма при его включении в компьютер.
3. NOVA-5000 SX "медленна" и имеет весьма ограниченные возможности по сравнению с
современными и даже устаревшими ноутбуками и "нетбуками" (и при этом она дороже
ноутбука!)
СОВЕТЫ:
B настоящее время можно обойтись вообще без NOVA-5000 - существует блок
USBLink для согласования датчиков с обычным ноутбуком. Это позволяет справиться
со многими (но далеко не всеми!) проблемами. Значительно повышаются
разрешенные частоты записи данных, улучшается быстродействие и наглядность,
меньше проблем с обработкой и распечаткой результатов.
4. Часто, особенно при сложной обработке данных, компьютер или MULTILAB "виснет"
в самый неподходящий момент, иногда "глючат" (в ответ на правильное действие не дают
стабильного правильного результата).
СОВЕТЫ:
- При выполнении работ на NOVA-5000 следует сразу сохранять полученные ценные
экспериментальные результаты в файл (а особо важные - на "флешку".)
- Если с лабораторией "Архимед" происходят странные вещи – это, к сожалению,
обычное явление. Относитесь к "Архимеду" как к "не вполне управляемой" системе.
Просто двигайтесь в нужном направлении и у вас всё получится (кроме того, что не
работает в принципе, такие вещи также случаются.).
- Перезагрузите "Multilab" или компьютер.
- Замените компьютер.
- Используйте USBlink вместо NOVA-5000.
- Используйте для обработки данных обычный компьютере или "нетбук", установив на
него программу "Multilab для Windows", которой не присущи многие недостатки
11
сокращённой версии, установленной на NOVA-5000. Перенести данные на компьютер
нетрудно через "флешку".
- Знайте, что нередко результат опыта нестабилен или неверно записывается из-за
проблем компьютера, а не потому, что Ваш опыт "не работает".
Например, если подключить цепь с датчиками тока и напряжения к блоку
питания (тому, который "для учителя"), почти наверняка результаты
получатся нестабильными - вы увидите непонятные пульсации
напряжения со значительными амплитудами. А с батарейками всё
прекрасно получается.
- Одновременное подключение многих датчиков и использование высоких частот
записи является дополнительной нагрузкой для "Архимеда" и иногда приводит к
нестабильной работе.
Если одновременно подключить к "NOVA-5000" датчик тока и три
датчика напряжения для изучения напряжений на элементах RLC цепочки,
установить максимально возможную частоту записи (а без этого не
увидеть "красивых" графиков), а после опыта провести обработку данных
(интегрирование тока, сложение графиков напряжений на элементах цепи)
– лаборатория, скорее всего, не оправдает Ваших ожиданий.
5. Компьютеры NOVA-5000 упорно "не дружат" со многими принтерами, включая
заявленные с поддержкой PCL, а установить нужный драйвер невозможно. (Наши
программисты многократно пытались сделать это, затем мы обратились в службу поддержки
фирмы fourier и узнали, что нужных драйверов не существует)
СОВЕТЫ:
- Найдите в школе разные принтеры и попробуйте, будут ли они работать с NOVA5000. Например, принтер HP1022 работает, а HP1020-нет.
- Установите Multilab на обычный компьютер и переносите данные через "флешку"
или по беспроводной сети (последнее мы не пробовали). При этом настройки и
масштабы графиков будут сбиваться!
6. Невозможно обновить программное обеспечение для NOVA 5000.
СОВЕТЫ:
- переходите на USBlink.
- Экспортируйте результаты в Excel и обрабатывайте их там.
7. Программа "Multilab" для NOVA5000, часто даёт сбои, временами "выполняет
недопустимую операцию" делает не то действие, которое требуется.
В Multilab плохо работает Мастер анализа.
Даже простые действия типа умножения функции на число могут быть выполнены
неверно. А при логарифмировании или аппроксимации полученной зависимости
экспонентой часто возникают проблемы ( полученный график неверен) (как на NOVA5000, так и на компьютере), устранить проблему не удаётся.
СОВЕТЫ:
- Иногда помогает перезагрузка программы, также всегда следует сразу сохранять
ценные файлы.
- Один из возможных путей- экспорт таблицы из Multilab в Excel и дальнейшая
обработка данных в Excel.
12
- Используйте для обработки данных обычный компьютере или "нетбук", установив
на него программу "Multilab для Windows", которой не присущи многие недостатки
сокращённой версии, установленной на NOVA-5000. Перенести данные на компьютер
нетрудно через "флешку".
Однако и Multulab для Windows РС (как версия 1.4.14 от 2006 года, так и
самая новая версия 1.5, (2010)) также даёт досадные сбои, и тоже очень
плохо автоматически масштабирует графики.
8. Программа "Multilab", включая версию 1.5 для Windows от 28.09.2010 отвратительно
масштабирует графики в автоматическом режиме. Она "не умеет" подбирать масштаб
так, чтобы график занимал почти всё поле, не может подбирать нужное количество
"клеточек" на оси, часто выводит "неудобные" числа на графики.
Программа не имеет опции автоматического отображения нескольких графиков в
одинаковых масштабах, что затрудняет их сравнение.
СОВЕТЫ:
- Требуйте от учащихся задавать правильные масштабы "вручную". По-настоящему
"красивые" графики не получатся, но нетрудно сделать, чтобы график занимал почти
всё поле.
- Иногда при "ручном" масштабировании бывают сбои. Удобнее обрабатывать
сложные графики это не на NOVA-5000, а на персональном компьютере.
- Не забывайте устанавливать "высокое качество" при распечатке графика с
NOVA5000, иначе не только качество печати будет плохим, но и числа станут
"наползать" друг на друга.
9. "ОДНОВРЕМЕННОСТЬ" СНЯТИЯ ДАННЫХ С ДАТЧИКОВ
Ряд работ, где нужно точно сравнить моменты поступления сигналов от нескольких
разных датчиков, систематически не получаются. Например, при изучении закона
электромагнитной индукции появляется "необъяснимый" сдвиг фаз между ЭДС в
катушке и производной измеренного магнитного поля. Также плохо работает и
изучение сдвига фаз напряжений в RLC цепочке. Плохо получается и прямое
измерение скорости звука микрофонными датчиками. Вероятная причина неодновременный опрос датчиков компьютером.
10. НОВЫЕ И СТРАРЫЕ ВЕРСИИ КОМПЬЮТЕРА NOVA-5000
После ремонта неисправных компьютеров NOVA-5000 SX (Ver 02.09) в ООО "РЕНЕ"
на некоторые из них был установлен другой, более быстрый процессор и Windows
Ver. 3.00.27. Увеличилось быстродействие и память. Тем не менее, проблемы с
принтером, "зависаниями" во время опытов и ошибки в Multilab (Ver. 2.25) остались.
11. ГДЕ "СКАЧАТЬ" ПРОГРАММУ "MULTILAB" для PC
Данные получены от службы технической поддержки компании "Fourier".
Официально разрешено бесплатно использовать эти программы.
ftp://MultiLabVA:DataLogger@fourier.exavault.com/MultiLab_v1.5_28.09.2010.zip
Если потребуются пароли, то вот они: Username: MultiLabVA, Password: DataLogger,
хотя скачивание должно работать и без ввода паролей (они вписаны в адрес).
13
ftp://MultiLabVA:DataLogger@fourier.exavault.com (Здесь то же самое, но там
выложены и прочие версии, предназначенные для MAC 10.4 и выше, а также Windows
XP, Vista, windows 7 -32 бит.) - в том числе и версия для PC от 03.2011, она почему-то
загружается только на испанском языке и тоже отвратительно масштабирует графики.)
УСТАНОВКА MULTILAB на Windows Vista:
( Проблема - при "обычной" установке на Vista не работал Usblink).
Запустите setup.exe и начните установку.
После этого вручную запустите файлы из основной директории установки Мультилаб:
Microsoft Windows Media Audio and Video 8 codecs.exe
Microsoft Windows Media Encoder 7.1 Distribution.exe
(Windows Vista блокирует эти файлы при установке, из-за этого потом Multilab
работает неправильно.) Также иногда антивирусная программа блокирует установку
этих файлов.
MULTILAB ДЛЯ MAC
USBlink не будет работать с МАСом . Fourier "сейчас разрабатывает соответствующее
программное обеспечение" (апрель 2011).
14
ОСОБЕННОСТИ ДАТЧИКОВ ЛАБОРАТОРИИ "АРХИМЕД".
ПРОБЛЕМЫ С ДАТЧИКАМИ И КАК С НИМИ СПРАВЛЯТЬСЯ
- Некоторые датчики не соответствуют заявленным параметрам на ряде режимов как по
точности, так и по диапазону измерений. (Подробности - ниже).
- Не все датчики подходят для выполнения ряда измерений. Например, чувствительность
датчика магнитного поля оставляет желать лучшего при измерении магнитного поля Земли, а
обратный ток диода невозможно измерить датчиком тока на 250 мА (нужен чувствительный
микроамперметр). Надо понимать, что возможности датчиков не безграничны.
- Затрудняет выполнение некоторых работ отсутствие в комплекте более чувствительных
датчиков тока и напряжения. Электрические колебания из-за этого приходится изучать при
помощи огромных катушек и конденсаторов больших емкостей; да и напряжение "от
батарейки" явно маловато для датчика на ±25 Вольт, нужен блок питания, а с ним датчики
работают нестабильно.
- Бывает, датчики определяются неправильно, например, датчик расстояния иногда
определяется на NOVA-5000 как датчик влажности.
- Датчики иногда выходят из строя. Например, устройство USBlink (исправно работающее с
датчиками тока и напряжения), вывело из строя один за другим два датчика давления.
- Некоторые датчики иногда сильно "шумят". (Например, график температуры может
получиться с "шумами", достигающими 0,5- 1 0С )
- Отсутствует чёткое описание принципа работы датчиков, не представлены разумные
данные по погрешностям. (Например, как понимать фразу: "точность датчика напряжения 3%
во всём диапазоне измерений". Значит ли это, что напряжение 0,01 В измеряется с точностью
3% ? Конечно же нет, такое напряжение этот датчик вообще не способен измерить.)
ПОДРОБНЕЕ ПРО НЕКОТОРЫЕ ДАТЧИКИ
ДАТЧИК ДАВЛЕНИЯ
УТЕЧКИ:
Все поступившие в нашу школу датчики “DT015-1” (16 шт.) без
дополнительной доработки давали утечку. Причина - неплотное крепление шланга к
датчику (а под давлением шланг растягивается). С датчиков давления, которые, как
заявлено, измеряют давление в диапазоне 0-700 кПа, при давлении около 300 кПа
срывает шланг… А если и нет, то ВСЕ(!) датчики дают небольшую утечку в месте
подсоединения шланга даже при давлении 200-300 кПа, которую трудно заметить.
Часто (и незаметно) трескаются хрупкие пластмассовые переходнички для этих
датчиков, появляется утечка, опыты из-за этого не получаются.
СОВЕТЫ:
- Настоятельно рекомендую (если вы собираетесь измерять давления, большие
атмосферного) самостоятельно отремонтировать датчики давления. Для этого надо
снять крышку корпуса и надеть дополнительный кусочек плотной пластиковой трубки
длиной около сантиметра поверх шланга в месте его подсоединения к датчику.
15
(Причина утечки в том, что мягкий шланг раздувается и начинает пропускать воздух
при больших давлениях.)
- При выполнении ЛЮБЫХ работ
необходимо проверять датчики (иначе
работы не будут получаться!). Для этого
дополнительная
можно подсоединить шприц объёмом 2-3 мл
трубка
с резиновым поршнем, сильно, до упора,
сжать воздух, подержать 10-20 секунд в
таком
состоянии,
затем
посмотреть,
вернется ли поршень в первоначальное
положение (можно чуть "помогать" ему).
Если поршень получил новое "устойчивое"
положение - была утечка. (Давление при
этом должно достигать 4-5 атмосфер.)
- Можно подсоединить к датчику сухой шприц на 5-10 мл. с резиновым поршнем,
включить регистрацию данных, создать давление 300- 400 кПа, убедиться, что оно не
падает со временем в течение 15-20 секунд.) Поршень шприца надо удерживать
неподвижно.
- Герметичность переходника легко проверить опусканием в воду (при этом в шлангах
надо создать давление). Сам датчик мочить нельзя.
ТОЧНОСТЬ:
Все имеющиеся у нас датчики давления (16 штук) с приемлемой точностью могут
измерять только атмосферное давление. Например, нулевое давление определяется как 15-18
кПа, а вместо 200 кПа датчик покажет 170. Нами были проведены детальные исследования
этих датчиков (работа Велиева и Коломейцева).
16
усредненные
показания
датчиков, кПа
показания
"эталонного"
манометра, кПа
Оказалось, что показания датчиков верные только в точке, равной атмосферному
давлению. В связи с тем, что датчики давления без дополнительной калибровки
совершенно непригодны для измерения малых давлений и дают погрешность до 20%
при измерении высоких давлений, не получается много интересных работ, например,
измерение давления насыщенного пара воды при различных температурах.
Разброс показаний датчиков между собой в диапазоне 0-350кПа не превышает 2-3%, а
в диапазоне 350-500кПа не превышает 3-4%, что позволило вывести калибровочную
функцию для корректировки показаний всех экспериментальных датчиков.
Калибровочная функция для корректирования показаний датчиков :
P = 1,209*Pдатч-22,6 (Рдатч- измеренное давление, Р- верное), её рекомендуется
применять при выполнении работ с лабораторией "Архимед". При помощи этой
функции можно скорректировать данные непосредственно в программе Multilab.
(Следует отметить, что через мастер анализа выполнить это простое линейное
преобразование на наших NOVA-5000 не получается!). При этом калибровка
стандартными средствами программы Multilab также не получилась. Калибровочный
17
винт, имеющийся на датчике, позволяет верно установить лишь одно значение
давления, например, атмосферное.
Точность показаний откалиброванных датчиков - порядка 3 - 5%
ЕЩЁ:
- Датчик имеет шланг объёмом около 0,7 см3, что существенно в некоторых опытах.
Кроме того, этот объём зависит от давления (трубка растягивается).
- Датчик расхода воздуха – это чувствительный датчик разности давлений. Можно
попробовать приспособить его для соответствующих работ.
ДАТЧИК ТЕМПЕРАТУРЫ
Датчик температуры (усовершенствованный термопастой) НЕ может измерять
температуру выше 100 0С (его показания в этом диапазоне явно занижены), несмотря
на заявленные в паспорте 110 0С.
В диапазоне от 20 0С до 100,0 0С существенных отклонений от показаний "эталонных"
термометров не выявлено.
Термопара в трубке датчика греется "через воздух", показания запаздывают на 20-30
секунд, в то время как улучшенный нами при помощи термопасты датчик работает в
4 раза быстрее. В результате многие исследования получаются плохо.
Погрешность и разброс показаний разных датчиков не превышает 1 OС.
СОВЕТЫ:
Вполне можно и нужно самостоятельно увеличить быстродействие этих датчиков. Для
этого датчик разбирают (черную трубку можно немного обрезать), на дно
металлического корпуса при помощи тонкого "инсулинового" шприца с насаженной
на него тонкой длинной трубкой помещают разумное количество теплопроводящей
пасты (её приобретают в любом компьютерном магазине), собирают датчик,
фиксируют соединение с помощью, например, надетой сверху "термоусадочной"
трубки. Постарайтесь при этом обеспечить контакт экранирующего провода (вы
увидите его, разобрав датчик) с трубкой.
"Усовершенствованный" датчик намного лучше. Например, работа с наблюдением
"полочки" у графика температуры плавящегося вещества плохо получается с
обыкновенным датчиком и гораздо нагляднее- с усовершенствованным.
ДАТЧИК НАПРЯЖЕНИЯ DT-001 на ± 25 Вольт
ТОЧНОСТЬ ИЗМЕРЕНИЙ:
Точность измерений заявлена как " 3% во всём диапазоне измерений".
Результаты специально проведенных в нашей школе исследований работы четырех
датчиков на различных режимах ( К. Силина и С. Скрябин):
- При напряжениях до 0,1В датчики использовать нельзя. (Ошибка от 30% до 120%)
- При напряжениях 0,1-0,5 В погрешность может достигать 10%
- При напряжениях 0,5-1 В погрешность не превышает 3%
- При напряжениях 1-4 В погрешность не превышает 2%
- При напряжениях 4-25 В погрешность не превышает 1%
- При напряжениях выше 24,5В датчики выдают свое предельное значение, для
каждого датчика свое - от 24,8 до 25,1В.
ОСОБЕННОСТИ ДАТЧИКА НАПРЯЖЕНИЯ:
18
Иногда случаются странные вещи - приём датчиком напряжения "комнатной
наводки" с частотой 50 Гц, при этом амплитуда регистрируемых колебаний достигала
двух вольт!!! Не верите? Подключите выводы датчика напряжения к собственным
ушам!
Одновременное параллельное подключение нескольких датчиков напряжения к
исправному стабилизированному блоку питания вместо постоянного напряжения
показывало колебания с падением напряжения до нуля, с периодом в 2-3 секунды, да
ещё и со сдвигом по фазе для каждого датчика! И компьютер меняли и блок питания результат тот же. При этом от батареек всё работало нормально.
Случаются и компьютерные сбои. Например, в одном из опытов сразу два
датчика напряжения показывали заведомо неверный результат 0,4 В при напряжении 0
В, которое контролировалось подключенным параллельно датчикам вольтметром.
После перезагрузки компьютера показания стали верными.
СОВЕТЫ:
Не используйте датчик для измерения малых напряжений.
Помните о нестабильной работе датчика почти со всеми блоками питания.
Применяйте батарейки или очень "продвинутый" блок питания.
ДАТЧИК РАССТОЯНИЯ
ОСОБЕННОСТИ (по результатам наших исследований):
- Датчик хорошо «держит» даже маленькую движущуюся мишень. Например, мячик
для настольного тенниса, движущийся вдоль оси датчика на расстоянии до 1,2 метра.
- Датчик легко теряет большую, но неподвижную мишень. Для стабильной работы при
больших расстояниях с неподвижной мишенью требуется угловой размер порядка 15
градусов. Также требующийся размер зависит от расстояния до датчика и
расположенных позади и сбоку от мишени предметов.
- Точность датчика - порядка 1% на всем диапазоне от 0,2 м до 6 м.
- Датчик измеряет расстояние до предмета от своей ТЫЛЬНОЙ стороны.
- Этот датчик хорошо "держит" даже маленькую движущуюся мишень, например,
теннисный шарик на расстоянии 1 метр, но требует больших размеров для
неподвижной мишени. Датчик перемещения использует ультразвуковую локацию,
поэтому не может работать на высоких частотах измерения (максимум - 50 - 100 Гц
на малых расстояниях), что затрудняет его применение для исследования, например,
ускорения тела во время упругого удара. Датчик может работать на расстоянии 1-1,2 м
с частотой до 50-100 Гц .
- При движении объекта на расстояниях порядка 50 см датчик даёт показания
"рывками" в 2-3 миллиметра, что укладывается в заявленную погрешность 3% от
измеряемого расстояния, но может привести к заметным ошибкам при измерении
изменения дальности до объекта при небольших перемещениях.
ДАТЧИК УГЛА ПОВОРОТА не работает при частоте вращения более двух оборотов в
секунду.
СЧЕТЧИК ГЕЙГЕРА - не определяется компьютером NOVA-5000.
19
ОБЗОР РАБОТ ДЛЯ УЧИТЕЛЯ
В методических пособиях места, рекомендованные для проверки учителем, для удобства
выделены курсивом.
Работа № 1
ИЗУЧЕНИЕ ЛАБОРАТОРИИ "АРХИМЕД"
Работа требуется, чтобы человек почувствовал уверенность при работе с компьютером.
Делается ДО выполнения прочих работ, занимает 2 - 4 часа. Содержит много интересных
дополнительных (не обязательных) заданий, которые можно изменить по выбору учителя.
Лучше подобрать задание, понятное и "актуальное" для ученика, например, девятиклассника
неплохо попросить определить, как зависит сила трения от веса тела и от поверхности (стол,
пол).
Работа № 5
ИЗУЧЕНИЕ НАСОСА
Небольшая несложная работа, где насос делается из шприца, шлангов, клапанов от
аквариума. Получается неплохо, при условии аккуратного выполнения соединений (это
трудно!) Проверяйте герметичность опусканием соединений в воду.
Используемые клапаны также текут при давлениях 200 кПа, они же предназначены для
аквариума… Больших давлений достичь не удастся. А вот продемонстрировать,
"почувствать" работу насоса для восьмиклассника (и заодно решить задачу по этой теме- для
десятиклассника) – вполне.
Работа № 6
ИЗУЧЕНИЕ ПРОЦЕССА КРИСТАЛЛИЗАЦИИ И ПЛАВЛЕНИЯ ВЕЩЕСТВА
Отличная работа для 8 (можно и для 10) класса. Хорошо получается.
Работа № 8
ИЗУЧЕНИЕ ИЗОХОРНОГО ПРОЦЕССА
Неплохо получается. Значение абсолютного нуля получается хорошо, если установка и
датчик давления не "текут", а датчик температуры усовершенствован добавлением
термопасты.
Желательно начать опыт с очень холодной воды (зимой добавьте в воду снега), нагревать
медленно. Можно и охлаждать, начиная с горячего состояния, но это будет намного дольше.
Впрочем, нагревание в обычном полностью заполненном чайнике даёт хороший результат
при использовании "быстрого" датчика температуры.
ОСТОРОЖНО- ГОРЯЧИЙ ЧАЙНИК!
ВНИМАНИЕ:- датчики давления, (не знаю, как Ваши, а все наши 16 датчиков- определённо
точно) требуют калибровки или пересчета снятых данных! Впрочем, эта работа получается и
с неточно работающими датчиками, получается абсолютный ноль- около – 350-280K.
Работа № 9
ИЗМЕРЕНИЕ ДАВЛЕНИЯ НАСЫЩЕННЫХ ПАРОВ АЦЕТОНА
Хорошо получается. Основные
избежать проблем, тщательно
шприца поставьте заглушку
переливании крови), опустите
убедитесь, что нет пузырьков.
внутрь бутылки воды нальют!)
проблемы- утечка в крышке бутылки или датчике. Чтобы
проверьте герметичность перед началом работы. Вместо
(сделайте её, например, из переходничка от системы
установку пробкой в воду, сильно сожмите бутылочку,
Учитель, помогите детям, а то они и датчик искупают, и
20
ВНИМАНИЕ:- датчики давления, (все наши 16 датчиков- точно) требуют калибровки или
пересчета снятых данных! Эта работа нормально получается с неточно работающими
датчиками,
Возможно (и интересно) дополнительное исследование поглощающей способности
активированного угля.
Работа № 9 А
ИЗМЕРЕНИЕ ДАВЛЕНИЯ В ГАЗОВОЙ ЗАЖИГАЛКЕ
Под наблюдением и на риск учителя можно (просто и более эффектно) взять баллончик для
заправки зажигалок. Получается 3- 3,5 кПа при комнатной температуре. Не давайте детям
самостоятельно подключать баллончик, строго следите за пожарной безопасностью!
ВНИМАНИЕ:- датчики давления, (все наши 16 датчиков- точно) требуют калибровки или
пересчета снятых данных! Эта работа нормально получается с неточно работающими
датчиками,
Работа № 10
ИЗУЧЕНИЕ ЗАВИСИМОСТИ
ТЕМПЕРАТУРЫ
ДАВЛЕНИЯ
НАСЫЩЕННГО
ПАРА
ВОДЫ
ОТ
Работа требует специально подготовленного сосуда, подключаемого к датчику. Причины
непонятны, проверено многократно. Возможно, получится намного лучше с
усовершенствованными датчиками.
ВНИМАНИЕ:- датчики давления, (все наши 16 датчиков- точно) требуют калибровки или
пересчета снятых данных! Эта работа плохо получается с неточно работающими датчиками,
Давление паров воды получается значительно (на 30%) заниженным по сравнению со
справочными данными, при 100 0С составляет около 70 кПа. А вот если выполнить
корректировку показаний датчиков, получится гораздо лучше- 93 кПа.
Не исключено также влияние конденсации пара в шланге датчика. Тем не менее, прекрасно
видна динамика роста давления в "сухом" и "влажном" сосудах.
Работа № 11
ИЗУЧЕНИЕ ИЗОТЕРМИЧЕСКОГО И АДИАБАТНОГО ПРОЦЕССА
Хорошо получается. Небольшая несложная работа.
В работе – довольно высокие давления, при этом ВСЕ наши датчики давали утечку!!! После
устранения утечек (см. соответствующий раздел) всё получается прекрасно.
ВНИМАНИЕ:- датчики давления, (наши 16 датчиков- определённо точно) требуют
калибровки или пересчета снятых данных! Эта работа неважно получается с неточно
работающими датчиками, и прекрасно- после корректировки показаний.
Работа № 12
ИССЛЕДОВАНИЕ ПРОЦЕССА ЗАРЯДКИ И РАЗРЯДКИ КОНДЕНСАТОРА
Не делайте эту работу по описанию из синей методички, прилагающейся к "Архимеду" – оно
не приведёт к победе!
Хорошо получается. Частая трудность- дети неверно собирают схему. Помогайте.
В Multilab при аппроксимации полученной зависимости экспонентой ОБЫЧНО возникают
проблемы (как на NOVA-5000, так и на компьютере), устранить их не удаётся. При этом
получившиеся зависимости- действительно экспоненты (проверяли).
Не требуйте от ребят получения результата- показателя экспоненты, если Multilab, как
обычно, "глючит". Ограничьтесь общим видом зависимости.
Работа № 13
21
ИЗМЕРЕНИЕ ЁМКОСТИ ГАЛЬВАНИЧЕСКОГО ЭЛЕМЕНТА (АККУМУЛЯТОРА)
Простая работа, вполне доступная восьмикласснику. Даёт красивый результат, особенно для
аккумулятора - в какой-то момент ток резко падает.
Следите за тем, чтобы работа завершилась в момент резкого падения тока - иначе
аккумулятор может выйти из строя.
Работа № 14
ИЗУЧЕНИЕ ЗАВИСИМОСТИ ВНУТРЕННЕГО СОПРОТИВЛЕНИЯ ИСТОЧНИКА ТОКА
ОТ ТОКА В ЦЕПИ
Красивая работа с неожиданным результатом. Внутреннее сопротивление гальванического
элемента весьма заметно уменьшается при увеличении тока до тока короткого замыкания.
В варианте без "Архимеда" с традиционными приборами, это работа получается трудно,
"капризничает", результаты меняются с каждым опытом. С "Архимедом"- стабильный
результат. Следите, чтобы школьники не подвергали батарейку длительной разрядке
большими токами.
Работа № 15
ИЗМЕРЕНИЕ ИНДУКЦИИ МАГНИТНОГО ПОЛЯ ЗЕМЛИ
Получается и многократно проверен странный результат- поле в школе 179 г. Москвы
направлено круто вниз – под углом 700 к горизонту. Чувствительность датчика маловата. Зато
есть уникальная возможность- "почувствовать" магнитное поле вокруг нас.
Работа № 17
ИЗУЧЕНИЕ ТЕМПЕРАТУРЫ И ЯРКОСТИ СВЕЧЕНИЯ НИТИ НАКАЛА ЛАМПОЧКИ
Захватывает дух от того, что мы узнаем в этом исследовании . Интересная большая работа.
Работа № 18
ИЗМЕРЕНИЕ МАГНИТНОГО ПОЛЯ СОЛЕНОИДА
Простая хорошо получающаяся работа. Только надо иметь или намотать на пластиковой
трубе соленоид.
Работа № 19
ПРОВЕРКА ЗАКОНА ЭЛЕКТРОМАГНИТНОЙ ИНДУКЦИИ
Красивая простая работа. Часто получается необъяснимый сдвиг фаз (видимо, проблема
связана с NOVой), которого не должно быть, иногда всё нормально.
РАБОТА № 20 (пробная)
ИЗУЧЕНИЕ СТОЯЧЕЙ ВОЛНЫ В ТРУБЕ
Работа получается, наблюдается "красивая" стоячая волна. С той стороны, где напротив открытого
конца трубы расположен динамик, выраженный узел давления получается далеко не всегда.
Протянув датчик через трубу, получим зависимость амплитуды колебаний от времени.
Проблема- в определении расстояния. Датчик расстояния приспособить не удалось, он теряет
мишень. Мы пользовались датчиком угла поворота. При этом непросто собрать установку правильно
и аккуратно.
Хорошим решением может быть протягивание датчика на нитке равномерно, например, с
использованием моторчика от LEGO.
Также нужен кабель повышенной длины, его нетрудно изготовить из двух обыкновенных.
РАБОТА № 22
ИЗУЧЕНИЕ РАСТЯЖЕНИЯ ПРОВОЛОКИ
Простая работа для 7-9 классов. Школьников радует неожиданно большое растяжение
проволоки - несколько сантиметров при длине 1,5 метра.
22
РАБОТА № 23
ИЗУЧЕНИЕ СВОБОДНОГО ПАДЕНИЯ ШАРИКА
Большая интересная и полезная работа со многими исследованиями, минимум - на 6 часов.
Хорошо получается с частотой 50- 100 измерений в секунду. Данное описание существенно
дополняет и уточняет то, что написано в "синей книжке" и на сайте.
РАБОТА № 24
ИЗУЧЕНИЕ КОЛЕБАНИЙ МАТЕМАТИЧЕСКОГО МАЯТНИКА
С применением датчика угла поворота.
Прекрасно качается маятник с весьма малым затуханием. Отлично получается соответствие
силы натяжения в нижней точке с теорией. Можно регулировать и исследовать затухание.
Можно на этой же установке исследовать колебания физического маятника.
Симпатичная, но большая работа, для "продвинутых" ребят.
Проблема при исследовании затухания- в Multilab при аппроксимации полученной
зависимости экспонентой ОБЫЧНО возникают проблемы (как на NOVA-5000, так и на
компьютере), устранить их не удаётся. При этом получившиеся зависимости- действительно
экспоненты (проверяли).
ПРИЛОЖЕНИЕ в "бумажном" виде – отчеты школьников о некоторых работах, находится в
школе 179.
23
ПРАВИЛА ВЫПОЛНЕНИЯ ЛАБОРАТОРНЫХ РАБОТ
Надо выполнить работу, аккуратно, правильно и понятно оформить отчет и подойти с ним
к учителю, чтобы обсудить результаты, и, возможно, побеседовать по всей теме. Разбросанный на
учительском столе отчет о работе не проверяется и не считается выполненной работой.
Помните о целях исследования и достигайте их в первую очередь!
Отчет о работе - это результаты вашего небольшого научного исследования. Он должен
содержать название, цели работы, оборудование, схемы, краткое, но понятное описание ваших
действий, вывод всех нужных формул, аккуратную и понятную запись полученных в
экспериментах данных, обработку их, нужные графики, сравнение того, что получилось, с тем, что
ожидалось в соответствии с теорией, выводы, интересные замечания. Отчет должен легко читаться
и быть понятным без дополнительных пояснений. Каждый график должен быть подробно
подписан и понятен, масштаб – правильно подобран, графики должны быть построены на
миллиметровке (в редких случаях- на бумаге в клеточку) или напечатаны с помощью компьютера.
Всегда приветствуются конструктивные предложения по улучшению описания работы. Ваш отчет
будет бережно храниться и может использоваться другими, поэтому сдайте его на сколотых
степлером (книжечкой!) листах формата А4 (не в тетради!) аккуратно написанным или
напечатанным. Обязательно оставьте слева(!) поля для скрепок. Пишите на одной стороне листа.
Если отчет печатается на компьютере, целесообразно взять для него описание работы в
электронном виде.
Работу надо сдать в день ее выполнения, а если данные требуют обработки дома конкретную дату защиты работы надо обсудить с учителем и записать в журнал. Тихо исчезнуть,
не сдав работу или не договорившись, нельзя (учитель обидится).
Если сложную работу выполняют два человека и сдают её в день выполнения, можно
сделать только один хороший отчёт.
При выполнении работы не забывайте постоянно оценивать полученные данные на
соответствие здравому смыслу.
В процессе выполнения работы полезно спросить учителя, насколько ему нравятся
полученные данные, обсудить ваши предварительные выводы (то, что не понравится, придется
переделывать, будет жалко, если учитель испишет ваш красивый отчет многими замечаниями…)
В описаниях работ места, выделенные курсивом, будут проверены учителем, позаботьтесь, чтобы
соответствующие данные были в вашем отчёте.
Каждая работа (или раздел работы) завершается написанием ВЫВОДОВ. Это НЕ
заклинания, которые умилостивят учителя , а краткий отчет - обобщение полученных
результатов (вспомните, для чего вы делали работу) и ваши замечания, озарения, идеи.
Прежде чем загрузить учителя какой-нибудь проблемой, подумайте, может быть, на этот
вопрос вам легко ответят товарищи, уже делавшие работу (они могут помочь найти что-то,
показать, как сбирается установка и дать полезные советы).
Делайте работу БЫСТРО. Очень быстро! Работы довольно большие, успеть сделать и сдать
работу за два часа можно, только если вы действуете, не отвлекаясь. Не забывайте, однако, что
святая обязанность каждого – помогать ближнему . Если вас двое - придумайте, как сделать всё
быстрее. Глупо, когда один делает, а другой ждет.
Очень аккуратно уберите оборудование на место. Каждая мелочь, которую сломали,
потеряли, оставили на столе, положили не в тот ящик, может вызвать серьезную проблему при
выполнении этой работы следующей группой. Если что-то не в порядке с оборудованием,
справьтесь с этим, а если не можете, скажите учителю.
При работе с компьютером NOVA не забудьте выключить его длительным (более 4
секунд) нажатием на черную кнопку, иначе он перейдет в спящий режим, но не выключится.
Если компьютер неисправен, напишите, в чем конкретно состоит неисправность, в
специальную тетрадь. На каждый компьютер ведется "досье", все компьютеры пронумерованы.
Нельзя уносить из школы описания работ. Если хотите взять их домой - снимите копию.
Соблюдайте правила безопасности (подробно - в каждой работе), поддерживайте порядок
на рабочих столах и в кабинете.
Создавайте рабочую обстановку, не делайте лишнего шума и лишнего движения в
кабинете, не ешьте в классе, заботьтесь об окружающих.
24
О ГРАФИКАХ И ПРАВИЛЬНЫХ МАСШТАБАХ
Выведенный на экран или бумагу график должен быть наглядным, красивым, легко читаемым.
Для этого график (или интересный в данной задаче его участок) должен занимать почти всё
отведенное поле. Ниже приведены три варианта масштабирования одного и того же графика:
Y
Y
X
X
ПЛОХО - неверно
подобран масштаб
по оси у
ХОРОШО
Y
X
ПЛОХО - неверно подобран
масштаб по оси х
Типичная ошибка:
Ось не имеет масштаба.
Надо понять, чему соответствует одна клеточка или сантиметр на графике и ясно указать это
на оси.
Типичная ошибка:
На оси вместо "красивых" чисел (например, 0, 2, 4, 6, 8, 10) пишут значения,
соответствующие нанесённым на график точкам (например: 1,25 2,73 6,08 9, 53)
Типичная ошибка:
На оси мало чисел.
Типичная ошибка:
Когда ось начинается "с нуля", забывают написать, где 0.
Типичная ошибка:
На осях не написаны единицы измерения.
Должно быть, например:
Х, см или
U, В
или
t, c
Типичная ошибка:
При построении графика упущены важные точки.
Там, где функция изменяется "быстрее", требуется наносить точки чаще!
Типичная ошибка:
График или рисунок не подписан или подписан так, что непонятно, зачем вы его сделали чи
что он показывает.
Рядом с каждой картинкой должен быть поясняющий текст, например:
Координата теннисного шарика при первом отскоке от поверхности (график маркирован экспериментальными
точками) и аппроксимация участка свободного падения параболой. Частота 100 измерений в секунду. Видно хорошее
соответствие полученной в опыте зависимости и параболы. По уравнению параболы определено ускорение
свободного падения g = 9,72м/с2 . По графику выполнена оценка времени взаимодействия шарика с поверхностью,
которое составило около 0,005 секунды.
25
О ПОСТРОЕНИИ ГРАФИКОВ
С ПОМОЩЬЮ ПРОГРАММЫ "MULTILAB"
При выводе графика компьютер "NOVA" не особенно "заботится" о правильном
масштабе. Нетрудно выделить интересующий фрагмент, чтобы он занял всё поле графика, но
посмотрите, какой ужас при этом будет выведен в качестве чисел на осях! Например,
получится время - 2.21, 3.21, 4.21, 5.21 и т.д. секунды. Это ещё можно использовать для
рабочих вариантов - но никак не для итоговых отчётных графиков. Не поленитесь сделать
"хорошие" числа на осях – например, время – 2, 3, 4, 5 и т. д. секунд.
Для этого нужно зайти в пункт меню "свойства графика", нажав на одну из иконок в нижнем
ряду (два кружочка с черточками справа от них). Там имеются две вкладки: "Ось Х" и
"Линии".
Для задания масштаба "вручную" по выбранной оси нужна вкладка "Линии". (Однако из-за
особенностей программы "Multilab", установленной на NOVA-5000, во время печати всё
равно могут быть выведены неудачные числа. При печати графика с нормального
компьютера всё работает хорошо.).
Иногда очень полезно задать правильные единицы измерения, например, выводить
время не в миллисекундах, а в секундах, ток в миллиамперах, расстояние в миллиметрах.
(Инструменты-Единицы измерения.)
Внимательно следите за данными, выводимыми на ось ординат. Иногда случается, что
туда выводятся сплошные нули… А кому он нужен, график без масштаба? Если на ось
совсем никак не выводятся нормальные числа - напишите их ручкой!
Если график получился состоящим из заметных кусочков прямых линий –
недостаточна частота записи при испытаниях.
Если на одном листе выведено несколько зависимостей - будет полезно "раскрасить"
графики цветными карандашами. Кроме того, перед печатью можно сделать один график
чёрным, а другой - серым. В программе Multilab для обычного компьютера (не на старой
"NOVе") есть возможность вывести на график экспериментальные точки. - Иконка в нижнем
ряду меню ("Свойства графика"- два кружочка с черточками справа от них). - "Ось Х""Линии". Здесь можно задать и цвет линии, и маркировку точками.
При печати графиков с компьютера "NOVA" не забудьте убрать галочку "Черновик",
иначе качество печати будет плохим.
Каждый график должен быть подробно подписан. Весьма желательно, чтобы было не
только ясно, что измерялось, но и как и зачем измерялось, и какие выводы следуют из этого
графика.
Постарайтесь сделать на графике поля, чтобы его можно было скрепить с остальными
листами работы в "книжечку"
Не забывайте, что, кроме принтера и компьютера, существует голова, линейка,
карандаши и миллиметровая бумага. Некоторые зависимости проще и нагляднее построить
"вручную" на миллиметровке.
26
НЕОБХОДИМО ЗНАТЬ ПРИ РАБОТАХ С КОМПЬЮТЕРАМИ "NOVA"
До начала других работ с компьютером "NOVA" надо изучить данный материал и выполнить
лабораторную работу "Изучение компьютера "NOVA".
КАБЕЛИ
 На NOVE крайне неудачные разъёмы для подключения кабелей, связывающих компьютер с
датчиками. Если применить даже небольшую силу, пытаясь вставить разъём в неправильном
положении, штырьки погнутся и замкнутся друг с другом. Затем такой кабель сжигает порты
всех компьютеров, в которые его впоследствии включают. ПЕРЕД ВКЛЮЧЕНИЕМ ЛЮБОГО
КАБЕЛЯ ИЛИ ДАТЧИКА В КОМПЬЮТЕР ПОСМОТРИТЕ И УБЕДИТЕСЬ, ЧТО ШТЫРЬКИ
РАЗЪЁМА НЕ ПОГНУТЫ И НЕ ЗАМКНУТЫ.
ЭТО ОЧЕНЬ ВАЖНО!
Если выявлен неисправный кабель, обязательно пометьте его (можно завязать узлами, можно
приклеить бумажку с надписью) и сообщите учителю.
 "NOVA" имеет четыре гнезда для подключения датчиков. Датчики подключают, начиная с
гнезда №1 (ближе к разъёму питания). Если сделать не так - датчик может не работать.
ПЕРЕД НАЧАЛОМ РАБОТЫ
 Установите компьютер в удобном безопасном месте, исключающем возможность падения его
на пол, заливания водой. Сделайте так, чтобы блоки питания не лежали на полу, чтобы за
провода никто не зацепился.
 Обеспечьте порядок на столе и вокруг него. Уберите лишние вещи, оставьте нужные.
 Если надо, подключите компьютер к блоку питания.

Если не используется блок питания, не забывайте контролировать индикатор заряда
аккумулятора.

Помните, что некоторые датчики (например, датчик расстояния) потребляют большой ток и
очень быстро разряжают аккумулятор.

Если опыт длительный - также требуется блок питания.

Перед началом опыта с записью множества данных убедитесь, что на компьютере достаточно
свободного места. Если это не так - результаты вашего опыта будут утеряны.

Не поленитесь подключить клавиатуру и мышку.
ПРОБЛЕМЫ С КОМПЬЮТЕРОМ, КАБЕЛЯМИ, ДАТЧИКАМИ
К сожалению, довольно часто (особенно при подключении многих датчиков или при записи
большого количества данных) с компьютером NOVA-5000 происходят странные вещи.
Например, "зависает" программа Multilab, иногда не "виден" датчик, не выполняется какоенибудь действие по обработке результатов или показания датчика явно противоречат здравому
смыслу. В этом случае:
- Подумайте, правильно ли вы действуете, может быть, ошибка произошла из-за ваших неверных
действий или неверного подключения чего-либо. Проверьте себя, посоветуйтесь с товарищами,
уже выполнявшими эту работу.
27
- Может помочь перезагрузка "Multilab" или компьютера.
- Если датчик даёт явно неверные показания, (а такое часто случается), перезагрузите компьютер,
а если это не поможет, замените датчик. Сообщите о неисправном датчике учителю, напишите, в
чём состоит неисправность (карандашом на корпусе датчика).
- Если ничего не помогает, замените компьютер.
При этом проверьте в первую очередь кабели подсоединения датчиков. Не берите другой
компьютер, пока не убедитесь, что на используемых кабелях нет погнутых и замкнутых
штырьков, а то сожжете новый компьютер тоже.
- Если выявлена неисправность компьютера, обязательно сообщите учителю и напишите, в чем
конкретно она состоит, в специальную тетрадь. Также напишите на бумаге, в чём состоит
проблема, приклейте эту бумагу к компьютеру. На каждый компьютер ведется "досье", все
компьютеры пронумерованы. Таким образом, неисправные компьютеры не будут попадаться вам
снова и снова.
- Помните, что обработка данных на домашнем компьютере или на школьном "обычном"
компьютере легче и быстрее, чем на NOVА, также при этом будет меньше "глюков". Некоторые
действия по обработке данных (а также иногда и вывод графиков с "красивым" масштабом)
вообще не получаются на NOVA-5000, зато прекрасно выполняются на обычном компьютере.
Если вы хотите установить Multilab на свой компьютер - возьмите загрузочный диск у учителя и
сделайте это.
ПРИ ВЫПОЛНЕНИИ РАБОТЫ
В описаниях работ места, рекомендованные для проверки учителем, для удобства выделены
курсивом. Подготовьте заранее ответы на все выделенные курсивом вопросы.
ЗАПИСЬ РЕЗУЛЬТАТОВ ОПЫТА
Помните, что программа обработки нередко даёт сбои, а NOVA-5000 часто "зависает", а иногда
даже выходит из строя навсегда. Поэтому, как только получен ценный экспериментальный
материал, его надо сразу же сохранить. Наиболее ценные данные рекомендуется сохранять на
свою "флешку". Компьютеры не защищены от вирусов - не используйте "грязные" "флешки"!
Сохраняя файл на компьютере, присвойте ему понятное имя, обязательно используйте свою
фамилию, например: Ivanov-1 padenie sharika.mlp. Обратите внимание на то, какой папке
сохранён файл. Сохранение промежуточных результатов тоже важно.
ЗАВЕРШЕНИЕ РАБОТЫ
Когда закончите работу и сдадите её, удалите с компьютера сохраненные вами файлы. Не надо
устраивать там мусорку.
В то же время, если работа не доделана и вам надо сохранить какие-либо файлы, напишите
записку, например: "Не удаляйте файлы ivanov_1 и ivanov_2 до 20.10.2010" и прикрепите её к
компьютеру. Обязательно запишите, на каком компьютере вы делали работу, чтобы не спутать
его с другим.
ВНИМАНИЕ! Если нажать кратковременно на чёрную кнопку, компьютер перейдет в ждущий
режим, но не отключится, а батареи будут разряжаться. Чтобы выключить его, надо удерживать
кнопку около 4 секунд!
Уберите аккуратно компьютер, стилус, блок питания, датчики и другое оборудование в
соответствующие коробки. Перед тем, как уйти, продемонстрируйте порядок учителю.
Проконтролируйте, чтобы он отметил ваши заслуги по выполнению работы в журнале.
А.Е. Тарчевский, шк. 179 МИОО, апрель 2010
28
ЛАБОРАТОРНАЯ РАБОТА № 1
ИЗУЧЕНИЕ ЛАБОРАТОРИИ "АРХИМЕД"
ОБОРУДОВАНИЕ:
компьютер NOVA-5000, датчик температуры, другие доступные датчики (например, силы,
перемещения, расхода воздуха и пр.), принтер.
Материалы : "Необходимо знать при работах с компьютерами "NOVA", "О графиках и
правильных масштабах", "О построении графиков с помощью программы "Multilab".
ЦЕЛИ РАБОТЫ:
Научиться работать с компьютером NOVA и программой Multilab.
Приобрести основные навыки, требующиеся для дальнейшей работы с компьютером,
почувствовать себя уверенно при работе с ним.
ВНИМАНИЕ!
Будьте осторожны с компьютером и датчиками. Это - не игрушки, а весьма дорогие вещи, за
сохранность которых вы отвечаете.
Работа с компьютером обычно осуществляется через сенсорный экран при помощи стилуса.
НЕЛЬЗЯ прикасаться к экрану авторучками, руками и другими неподходящими предметами!
Рекомендуется также подключить к компьютеру мышь и клавиатуру.
Кроме того, через порт USB можно подключить принтер (годится не всякий) и выводить на
печать полученные данные и графики.
ВЫПОЛНЕНИЕ РАБОТЫ:
1. Установите компьютер в удобном безопасном месте, исключающем возможность падения
его на пол, заливания водой, задевания за провода проходящими людьми и другие
неприятности.
Обеспечьте порядок на столе и вокруг него. Уберите лишние вещи, оставьте нужные.
Подключите компьютер к блоку питания. (Разряженный компьютер должен заряжаться 4
часа без перебоев, в это время с ним можно работать.)
2. Включите компьютер длительным нажатием на черную кнопку.
После загрузки вы можете изучить рабочий стол и панель задач системы. Обратите внимание
на иконку в правом нижнем углу экрана: нажатием на нее можно вызвать (и убрать)
экранную клавиатуру.
3. Запустите ПУСК - ПРОГРАММЫ-НАУКА-MULTILAB
Откроется программа, которая позволит записывать, обрабатывать и распечатывать данные
экспериментов.
Иконка с зелёным человечком в верхнем ряду иконок - это «кнопка» запуска эксперимента;
во время опыта на её месте изображается красная рука - это остановка эксперимента.
"NOVA" имеет четыре гнезда для подключения датчиков. Датчики подключают, начиная с
гнезда №1 (ближе к разъёму питания). Если сделать не так - датчик может не работать.
ВНИМАНИЕ!
Лаборатория имеет крайне неудачные разъёмы для подключения кабелей,
связывающих компьютер с датчиками. Если применить даже небольшую силу,
пытаясь вставить разъём в компьютер или в датчик в неправильном положении,
29
штырьки погнутся и замкнутся друг с другом. Затем такой кабель сжигает порты
всех компьютеров, в которые его впоследствии включают.
ПЕРЕД ВКЛЮЧЕНИЕМ ЛЮБОГО КАБЕЛЯ ИЛИ ДАТЧИКА В КОМПЬЮТЕР
ПОСМОТРИТЕ И УБЕДИТЕСЬ, ЧТО ШТЫРЬКИ РАЗЪЁМА НЕ ПОГНУТЫ И НЕ
ЗАМКНУТЫ. ЭТО ОЧЕНЬ ВАЖНО!
Если выявлен неисправный кабель, сообщите учителю и обязательно пометьте
его (можно завязать узлами, можно приклеить бумажку с надписью).
Перед началом эксперимента откройте и изучите меню РЕГИСТРАТОР-НАСТРОЙКА –
здесь можно увидеть, какие датчики определил компьютер, задать частоту замеров в
эксперименте и их количество (а значит, и время записи, которое компьютер вычислит
автоматически). Здесь же можно выбрать возможность измерять "непрерывно", в этом случае
опыт закончится, когда вы нажмёте иконку остановки эксперимента.
4. ИЗМЕРЕНИЕ ТЕМПЕРАТУРЫ
4.1 Подключите к компьютеру датчик температуры.
Установите НАСТРОЙКИ - частота 10 замеров в секунду, число измерений 2000. (Легко
посчитать время эксперимента, 200 секунд, через которое запись автоматически остановится,
однако никто не мешает вам остановить опыт досрочно, нажав на иконку с красной рукой)
4.2 Запустите эксперимент. (Иконка с зелёным человечком.)
4.3 Используйте датчик температуры для измерения температуры своего тела, как обычный
градусник. (Если это неудобно, просто зажмите и держите кончик датчика температуры
между ладонями, но тогда вы получите не 36,6 градусов, а более низкую температуру.)
4.4 Наблюдайте, как выводится на экран график зависимости температуры от времени.
4.5 Сохраните полученный файл (научитесь делать это).
4.6 Попробуйте изменить масштаб графика по одной и по другой оси, двигая вдоль
соответствующей оси.
4.7 Попробуйте выделить на графике фрагмент (используйте инструменты, иконки которых
располагаются под графиком), выделенный фрагмент "растянется" на весь экран.
4.8
Масштаб иногда придётся задавать и "вручную", например, чтобы во время
предстоящего опыта на экран сразу выводился "красивый", легко читаемый график. Для
этого нужно зайти в "свойства графика", нажав на одну из иконок в нижнем ряду (два
кружочка с черточками справа от них). Там имеются две вкладки: "Ось Х" и "Линии".
Для изменения масштаба по какой-либо оси нужна вкладка "Линии". Для выбранного
параметра можно задать масштаб и цвет, которым он будет показан на графике. Таким
образом, вы можете получить "красивую" и наглядную картинку.
Потренируйтесь делать это. Задайте перед опытом "удобный" диапазон температур,
например, 32-37 градусов. Проведите измерение ещё раз, посмотрите, каким получится
график.
Заметим, что во вкладке "Ось Х" можно изменить ось абсцисс: по умолчанию там
стоит время, но можно выбрать и другой параметр. ( Например, есть опыт, в котором
одновременно измеряют температуру и давление газа в сосуде. Чтобы построить график
30
зависимости давления от температуры, а не от времени,
"температура" в качестве оси абсцисс. )
надо выбрать параметр
4.9 Задайте имя графику (меню ИНСТРУМЕНТЫ).
4.10 Подключите компьютер USB- кабелем к принтеру. ("Квадратный" разъём включите в
компьютер", "плоский"- в принтер). Распечатайте полученный подписанный график
температуры с правильным масштабом. Применяйте данные, описанные в материалах : "О
графиках и правильных масштабах", "О построении графиков с помощью программы
"Multilab". Если Вы ещё не знакомились с этими материалами, попросите их у учителя.
4.11 Попробуйте посмотреть полученные данные в виде таблицы (используйте иконки,
которые располагаются в одном ряду с "зеленым человечком").
4.12 По графику оцените время установления теплового равновесия тела с датчиком.
Имеется в виду тот момент, когда разумно "вынимать" градусник, потому что дальнейшее
"нагревание" его практически не изменяет результат. На графике, однако, таких тонкостей
почти не видно. Поможет иконка под ним (линия с крестиком) – вывод курсора на график.
После "нажатия" на иконку рядом с графиком появится стрелочка, также вы увидите
значения координат, соответствующие точке, на которую показывает эта стрелочка.
Стрелочку можно двигать стилусом вдоль графика.
Запишите (можно на листе с графиком) время, за которое установилось тепловое
равновесие тела и датчика температуры. Объясните, почему вы решили, что время именно
такое. Отметьте результат на графике. Обсудите результат с учителем.
4.13 Изучите, как работают другие иконки.
4.14 Используя меню ФАЙЛ- ОЧИСТИТЬ ВСЁ, освободите компьютер от данных
проведенного эксперимента (это надо делать после обсуждения результатов с учителем).
В этой работе не нужен подробный отчёт - только распечатанный "красивый" график,
ваша оценка времени установления теплового равновесия и заявление о том, что
требующиеся действия у вас получились. Будьте готовы рассказать о том, как
устанавливать различные частоты и продолжительность опыта, как масштабировать
график, задавая минимум и максимум, "растягивать" его стилусом, делать на графике
надписи, выбирать нужный фрагмент, считывать численные данные с графика, удалять
файлы и сохранять их, распечатывать графики.
Покажите результаты учителю. Затем, если осталось время - выполните задание 6.
5. Закройте программу “Multilab” и выключите компьютер.
ВНИМАНИЕ! Если нажать кратковременно на чёрную кнопку, компьютер перейдет в
ждущий режим, но не отключится, а батареи будут разряжаться. Чтобы выключить его, надо
удерживать кнопку не отпуская более 4 секунд!
Уберите аккуратно стилус, компьютер, блок питания и датчики туда, откуда вы их брали.
7. Познакомьтесь с материалами : "Необходимо знать при работах с компьютерами "NOVA",
"О графиках и правильных масштабах", "О построении графиков с помощью программы
"Multilab".
6. ДОПОЛНИТЕЛЬНОЕ ЗАДАНИЕ:
ЭТОТ РАЗДЕЛ МОЖЕТ ВЫПОЛНЯТЬСЯ В КАЧЕСТВЕ ОТДЕЛЬНОЙ РАБОТЫ.
31
Цель – научиться работать с различными датчиками.
Отчётность- обсуждение с учителем проведенных вами измерений в свободной форме.
По согласованию с учителем вы можете вместо описанных опытов или в дополнение к ним
сделать свои. Можно распечатать понравившиеся графики. Экономить бумагу тоже надо,
поэтому явно ненужные вещи не печатайте.
6.1 ИЗМЕРЕНИЕ РАСХОДА ВОЗДУХА
Подключите датчик расхода воздуха (трубку обмотайте чистым платком или кусочком
чистой бумаги) и дышите через него нормально в течение 50 секунд. Постройте график
зависимости расхода воздуха от времени. Можно проинтергрировать график и получить
зависимость объёма воздуха от времени. Можно только выдыхать через датчик, а вдыхать
мимо него, затем, проинтергрировав график, определить выдыхаемый за минуту объём
воздуха.
6.2 ИЗМЕРЕНИЕ РАССТОЯНИЙ.
Работа с датчиком расстояния требует обязательного подключения компьютера к блоку
питания!
Датчик расстояния излучает ультразвуковые импульсы, они отражаются от препятствия и
возвращаются обратно. По времени, которое "путешествовал" звук, рассчитывается
расстояние.
Подключите датчик. Закрепите его в штативе на высоте 10 см. от поверхности стола,
направьте вдоль стола. Расположите на расстоянии не менее 40 см. от него книгу
(перпендикулярно столу). Запустите опыт. Отодвигайте книгу. Положите метровую линейку
или рулетку на стол. Проверьте, правильно ли датчик измеряет расстояния.
6.3 ИЗМЕРЕНИЕ СИЛЫ
Подключите к компьютеру датчик силы, подвесьте датчик на штативе. Проверьте,
показывает ли датчик 0, если нагрузки нет. Подвешивая грузы известной массы (можно
вешать их на крючок или в полиэтиленовом пакетике) проверьте, правильно ли датчик
измеряет силу тяжести.
Укрепите груз на «мягкой» пружине, заставьте его качаться вверх-вниз, постройте
график зависимости силы, регистрируемой датчиком, от времени.
Девятиклассникам будет интересно определить, как зависит сила трения от веса тела и
от поверхности (стол, пол). Для этого рекомендуется прицепить картонную коробочку
проволочкой к датчику, затем положить в эту коробочку грузы 1 кг, 2 кг, и т.д. и "возить" её
датчиком по столу, по полу. Тянуть надо медленно, движение должно быть равномерным!
Не забудьте записать результаты и поделиться ими с учителем!
6.4 МОЖНО ПОПРАКТИКОВАТЬСЯ И С ДРУГИМИ ДАТЧИКАМИ
Описание различных датчиков, их пределов измерений и принципа работы есть в конце
руководства к лаборатории "Архимед"
А.Е. Тарчевский, шк. 179 МИОО, 10 апреля 2011
32
ЛАБОРАТОРНАЯ РАБОТА 5 для 8 класса
ИЗУЧЕНИЕ ПРОЦЕССА КРИСТАЛЛИЗАЦИИ И ПЛАВЛЕНИЯ
ВЕЩЕСТВА
ОБОРУДОВАНИЕ
Кипяток, калориметр, (при выполнении работы
большим количеством учащихся желательно
иметь на одном из столов металлический стакан с
постоянно кипящей на плитке водой).
NOVA, датчик температуры (улучшенный),
пробирка со стеариновой кислотой.
Для улучшения теплообмена внутрь пробирки
необходимо поместить спутанную тонкую
медную проволоку. Эта проволока должна быть
"пушистой", равномерно заполнять
приблизительно 2 сантиметра пробирки и
находиться в расплавленном изучаемом веществе.
Она имеет высокую теплопроводность и
обеспечит быстрое установление одинаковой
температуры внутри пробирки.
Подставка для пробирки или штатив с лапкой для
закрепления пробирки.
Датчик температуры не должен касаться дна и
стенок пробирки. Чтобы датчик хорошо держался
в пробирке, рекомендуется надеть на него шайбу
из пенополиуретана.
Закрепляя пробирку в штативе, обмотайте её
полоской бумаги, не раздавите!
ЦЕЛИ РАБОТЫ
Построить график зависимости температуры охлаждаемого вещества от времени.
Изучить и объяснить полученную зависимость. Определить температуру кристаллизации.
Дополнительное задание - выполнить аналогичное исследование для процесса плавления.
ВЫПОЛНЕНИЕ РАБОТЫ
1. Подготовьте компьютер, подключите датчик температуры. Установите частоту записи одно измерение в секунду, продолжительность - непрерывно.
2. Поставите пробирку в стакан с кипятком, кристаллическое вещество через некоторое
время расплавится.
Внутрь пробирки аккуратно поместите датчик температуры, добейтесь, чтобы датчик
располагался по центру пробирки, не касался дна и стенок. Подождите 30 секунд, чтобы
датчик пришел в тепловое равновесие с веществом. Важно нагреть пробирку не менее 85-90
градусов, иначе не увидим графика изменения температуры для остывания жидкости.
Внимание! Типичная ошибка- пока несли пробирку от учительского стола, пока
включали установку, температура опустилась ниже 80 oC и опыт пришлось переделывать
заново.
ОСТОРОЖНО! КИПЯТОК!
3. Поставьте пробирку на подставку или укрепите в штативе, включите запись результатов.
33
Опыт длится 15-20 минут. Если мало времени, допускается охлаждать пробирку в стакане с
холодной водой, в этом случае время проведения опыта сократится до 3-4 минут, качество
графика будет немного хуже.
4. Наблюдайте за пробиркой, Когда начнется кристаллизация, на графике будет характерная
"полочка". Посмотрите на пробирку - меняется ли в это время внешний вид вещества?
Запишите результат.
5. При охлаждении до температуры около 45 - 40 градусов остановите опыт.
6. Позаботьтесь об удобных масштабах, подпишите график зависимости температуры от
времени. Распечатайте график.
7. Объясните письменно (можно непосредственно на графике), что и в какой момент
происходило.
На графике должна быть видна "полочка". Пробирка была горячей (следовательно,
отдавала тепло), а температура вещества не изменялась. Почему? Что происходило в это
время?
8. Укажите на графике и запишите температуру кристаллизации.
9 Если вы не делаете дополнительного задания, датчик температуры можно будет осторожно
вынуть, держа его за металлический корпус.
НИКОГДА НЕ ТЯНИТЕ ДАТЧИК ЗА ПРОВОД!
Протрите датчик салфеткой.
10 ДОПОЛНИТЕЛЬНОЕ ЗАДАНИЕ
Аналогичным способом изучите, что происходит при нагревании жидкости. Для этого
погрузите пробирку с установленным в ней термометром в калориметр с крутым кипятком
чтобы вещество расплавилось, Рекомендуется закрыть калориметр листом картона, бумаги.
Сравните температуру плавления и кристаллизации вещества.
Подпишите и распечатайте график процесса кристаллизации.
А.Е. Тарчевский, шк. 179 МИОО, Москва, 10 апреля 2011
34
ЛАБОРАТОРНАЯ РАБОТА 6
ПРОБНАЯ
ИЗУЧЕНИЕ НАСОСА
ОБОРУДОВАНИЕ
Насос, изготовленный из шприца на 10-50 мл с
резиновым поршнем, трубок, клапанов, пропускающих
газ только в одном направлении, жесткая ёмкость,
подключенная к насосу (объём ее должен в несколько
раз превышать объём шприца), датчик давления,
NOVA-5000.
Для стеклянной бутылки необходим плотный тканевый
мешок, обеспечивающий безопасность.
Насос можно собрать в двух вариантах- для
накачивания воздуха в бутылочку и для откачивания.
ОСТОРОЖНО!
СТЕКЛЯННАЯ БУТЫЛКА ВЗРЫВООПАСНА ПОД
ДАВЛЕНИЕМ, ПОМЕСТИТЕ ЕЁ В ТРЯПОЧНЫЙ МЕШОК!
Внимание! Много трудностей в этой работе может вызвать утечка в соединениях трубочек.
Следите, чтобы шланги были надеты плотно. Можно обмотать места стыковки проволокой. Можно
пользоваться резиновым клеем для герметизации соединений. Клапаны от аквариума обязательно
смочите водой перед началом работы, чтобы не было утечки.
Соберите установку, подключите к сосуду датчик давления, установите частоту замеров -10 в
секунду, количество- непрерывно.
ЗАМЕЧАНИЕ: ОСОБЕННОСТИ ДАТЧИКОВ ДАВЛЕНИЯ, ИСПОЛЬЗУЕМЫХ В НАШЕЙ ШКОЛЕ:
Датчики давления лаборатории "Архимед" работают со значительными ошибками. Если они
до сих пор не исправлены (спросите учителя), необходимо пересчитать данные по формуле
P = 1,21*Pдат ч - 22,6 (Здесь Рдатч- измеренное давление, Р- верное). При помощи этой
функции можно скорректировать данные непосредственно в программе Multilab. (Следует
отметить, что через мастер анализа выполнить это простое линейное преобразование на
наших NOVA-5000 не получалось!) В этой работе без корректировки данных ошибки будут
значительными, особенно при откачивании воздуха. (График можно вывести и без
корректировки, а при решении задачи данные надо скорректировать.)
ЦЕЛИ РАБОТЫ: Часть 1
1. Исследуйте динамику установления давления при накачивании воздуха в бутылку.
При этом делайте достаточные паузы после каждого качания, график должен показывать, что
в этот момент давление установилось и неизменно (или, наоборот, есть утечка)
Если давление "не держится", найдите и устраните причину утечек (если "текут" места
прикрепления трубок, обмотайте их медной проволокой) Проверить, есть ли утечка, можно,
поместив соответствующее место в воду. (Не намочите датчик!)
Узнайте максимальное давление, которое можно создать в бутылке данным насосом.
Получите "красивый", "интересный" график и распечатайте его.
2. Задание для 10 класса :
Решите задачу. В бутылку объёмом … накачивают воздух насосом объёма…
35
Какое давление установится после … качаний? Выберите данные самостоятельно (для того
участка опыта, где точно нет утечки). Проведите расчёты, сравните полученные данные с
результатами опыта.
Запишите вывод.
Часть II
Выполните аналогичные действия, откачивая воздух из бутылки.
Представьте график зависимости давления от времени для откачивания воздуха. Запишите
выводы.
Задача расчёта давления с откачиванием сложнее, спросите учителя, надо ли вам делать
её.
А.Е. Тарчевский, шк. 179 МИОО, Москва, 10 апреля 2011
36
ЛАБОРАТОРНАЯ РАБОТА № 8
ИЗУЧЕНИЕ ИЗОХОРНОГО ПРОЦЕССА
ПРЕДВАРИТЕЛЬНЫЕ ТРЕБОВАНИЯ:
- знание темы "Газовые законы и изопроцессы".
ОБОРУДОВАНИЕ:
- NOVA, датчик давления, датчик температуры, (желательны усовершенствованные датчики),
– холодная вода, а лучше- ледяная вода. (В холодную погоду поставьте ёмкость с водой на
мороз заранее или принесите снега с улицы, чтобы начать опыт с нулевой температуры),
- электрический чайник,
- рукавица, бумажное полотенце,
- штатив с лапками, проволока тонкая,
- сухой герметичный сосуд, подключаемый к датчику давления
(медный баллончик с уже припаянной тонкой трубкой, снятый со старого холодильника (Со второго, широкого конца надо удалить
содержащиеся внутри мелкие шарики и герметично закрыть отверстие, например, нарезать резьбу и завернуть винт с прокладкой).
Также можно взять баллончик от сифона и впаять в него медную трубку.
В худшем случае это может быть стеклянная колба с узким горлышком (чтобы избыточное давление не выбило пробку), резиновая пробка
для колбы, проколотая иглой от шприца и оборудованная переходником для подключения к датчику давления).
НАКОНЕЧНИК
ОТ СИСТЕМЫ
ПЕРЕЛИВАНИЯ
КРОВИ
ПАЙКА
БАЛЛОНЧИК ОТ
ХОЛОДИЛЬНИКА
ИЛИ ОТ СИФОНА
КЛЕЙ
ЦЕЛИ РАБОТЫ:
- Проверить выполнение закона Шарля (линейную зависимость давления от абсолютной
температуры)
- Определить значение абсолютного нуля температуры.
ВНИМАНИЕ!
- Осторожно обращайтесь с горячим
Датчик температуры
чайником! Не оставляйте чайник с
на фото расположен
кипятком без присмотра, если
неверно, надо
кипяток не нужен – сразу вылейте
БАЛЛОНЧИК С
примотать его
его.
Выливая
кипяток
в
ВОЗДУХОМ
проволочкой
к
керамическую
раковину,
лейте
баллочнчику
медленно, при этом откройте кран с
водой,
иначе
раковина
может
треснуть.
- Чайник работает от сети 220В! Не
трогайте провода, имейте сухие
руки, не ставьте чайник в раковину и
на мокрые поверхности.
- Если используется стеклянная
колба, пробку в неё вставляйте плотно, как бы завёртывая. При этом пользуйтесь рукавицей или
тряпкой, чтобы не порезать руки, если горлышко колбы вдруг развалится!
37
ЗАМЕЧАНИЕ: ОСОБЕННОСТИ ДАТЧИКОВ ДАВЛЕНИЯ, ИСПОЛЬЗУЕМЫХ В НАШЕЙ ШКОЛЕ:
Датчики давления лаборатории "Архимед" работают со значительными ошибками. Если они до
сих пор не исправлены (спросите учителя), необходимо пересчитать данные по формуле P =
1,21*Pдат ч - 22,6 (Здесь Рдатч- измеренное давление, Р- верное). При помощи этой функции
можно скорректировать данные непосредственно в программе Multilab. (Следует отметить, что
через мастер анализа выполнить это простое линейное преобразование на наших NOVA-5000 не
получалось!) В этой работе без корректировки данных ошибки измерения давления составят до 4%,
если не получается легко их исправить, можно игнорировать их.
ПОДГОТОВКА:
- При выполнении ЛЮБЫХ работ необходимо проверять датчики (иначе работы не будут
получаться!) Для этого можно подсоединить шприц объёмом 2-5 мл с резиновым поршнем,
сильно, до упора, сжать воздух, подержать 10-20 секунд в таком состоянии, затем
посмотреть, вернется ли поршень в первоначальное положение (можно чуть "помогать" ему).
Если поршень получил новое "устойчивое" положение - была утечка. (Давление при этом
должно достигать 3-6 атмосфер.) Также можно запустить регистрацию данных, создать
давление около 400 кПа, убедиться, что оно не падает со временем в течение 10-20 секунд.
Небольшое падение давления в начальную секунду вызвано охлаждением газа после его
сжатия и является нормальным.)
- Убедитесь, что колба или баллончик внутри СУХИЕ, иначе к давлению воздуха добавится
сильно изменяющееся с температурой давление паров воды.
- Подключите датчики давления и температуры,
- Установите частоту 10 измерений в секунду, количество измерений - непрерывно.
- Налейте почти полный чайник холодной, а если есть возможность - ледяной воды. (Зимой
не поленитесь сходить за снегом и добавить достаточное(!) количество снега в воду.) Чем
больше изменение температуры в опыте, тем интереснее результаты.
- Колбу и датчик температуры поместите в электрический чайник.
- Обеспечите, чтобы датчик давления и нагреваемый сосуд НЕ висели на шланге.
Пользуйтесь штативом.
- Добейтесь, чтобы датчик температуры внутри чайника был рядом с нагреваемой колбой. И,
конечно, НЕ касался дна или нагревательного элемента чайника (важно!) Удобно
прикрепить датчик проволочкой к колбе. Чувствительный элемент находится у конца трубки
датчика.
- Если Вы пользуетесь стеклянной колбой, прикройте крышку чайника так, чтобы колба не всплывала. Убедитесь, что провода, пробка и
шланг не попали в воду, в то же время колба должна быть почти полностью под водой.
- Перед включением установки ещё раз убедитесь, что на столе порядок, нет лишних
предметов, никто не толкнёт горячий чайник и не запутается в ваших проводах, а на
компьютер не попадёт вода.
Покажите установку и порядок на столе учителю. Будьте готовы ответить на вопросы о мерах
безопасности при выполнении работы.
ДОПОЛНИТЕЛЬНО (не обязательно).
Желающие могут исследовать влияние скорости нагревания на результаты работы, почёт будет им за
это. Для этого надо сделать опыт дважды - при нагревании, а затем (не выключая записи) – при охлаждении.
(Тогда надо запастись терпением - чайник остывает почти час, подключить компьютер к блоку питания,
установить частоту одно измерение в секунду). В результате на одном поле получится два графика, совпадение
которых покажет нам отсутствие утечки и влияние возможного запаздывания установления температуры в
колбе и в датчике температуры. В этом случае сначала всё же следует сделать простой вариант эксперимента
(только для нагревания), а затем, приобретя нужный опыт, выполнить длительный опыт с нагреванием и
последующим охлаждением.
ВЫПОЛНЕНИЕ ОПЫТА
- Запустите измерение, включите чайник. При закипании воды выключите чайник и
остановите запись.
ВНИМАНИЕ! Не пропустите этот момент, а то вода начнет выплёскиваться.
38
ОСТОРОЖНО!
- Будьте весьма осторожны с кипятком. По окончании измерений сразу вылейте кипяток из
чайника.
- Для стеклянной колбы: Всё горячее, а колба под давлением! Прежде чем вынуть колбу,
отсоедините её от датчика давления.
ОБРАБОТКА РЕЗУЛЬТАТОВ ОПЫТА
- Сохраните файл, присвойте ему имя, чтобы возможные сбои программы не уничтожили
данные Вашего эксперимента.
- На экране получатся графики зависимостей температуры и давления от времени.
- Посмотрите, линейна ли зависимость температуры от времени. Вероятно, при низких
температурах - почти линейна, а при более высоких температурах наклон графика
уменьшается. Подумайте и напишите, почему это так.
- Получите на экране график зависимости давления от температуры.
Для этого используйте иконку внизу - "Свойства графика"- ЛИНИИ - для графика давления в
качестве "оси x" надо указать температуру. Здесь же (если надо) выберите "вручную"
хороший масштаб (так, чтобы график занимал почти полное поле).
- Получите уравнение аппроксимирующей прямой с помощью компьютера.
Инструменты - Анализ - Линейное приближение. Поставьте курсор на полученный график.
Внизу вы увидите уравнение полученной прямой. ( Только оно в странном виде: f(x) = …x
+… На самом деле f(x) - это P(t) , х - это t.)
Запишите это уравнение P(t) =….
- Распечатайте график P(t) и его линейное приближение в таком масштабе, чтобы графики
занимали почти всё поле. Не забудьте подписать графики. Позаботьтесь о читаемых
числах на осях. Линеен ли экспериментальный график? Выполняется ли закон Шарля?
Можно и выводы напечатать на графиках.
ОПРЕДЕЛЕНИЕ ЗНАЧЕНИЯ АБСОЛЮТНОГО НУЛЯ
- При помощи графика P(t) для изохорного процесса легко определить значение абсолютного
нуля температуры. Оно соответствует той точке, где давление газа равно нулю.
Аппроксимируем график прямой линией. Если сделать это "на глаз", будет плохая точность,
т.к. имеющийся участок графика короткий. Поэтому воспользуемся уравнением линейного
приближения P(t) =…. Рассчитаем t, при котором P(t) = 0. Это и есть значение абсолютного
нуля.
Для графика зависимости давления от температуры выберите "вручную"
соответствующий масштаб. (Давление - от нуля, температура – от получившегося
"абсолютного нуля" в градусах Цельсия до 100 0С). Добейтесь правильной картинки на
экране.
Распечатайте
график,
приложите
прозрачную
линейку,
нарисуйте
аппроксимирующую прямую, укажите на графике значение абсолютного нуля (то самое,
которое рассчитали по уравнению аппроксимирующей прямой.)
Запишите выводы по работе. Проанализируйте, полученные результаты. Возможно, чтото не полностью соответствует теории, постарайтесь объяснить, почему.
39
ТИПИЧНЫЕ ОШИБКИ ПРИ ВЫПОЛНЕНИИ ЭТОЙ РАБОТЫ:
- неосторожное обращение со стеклянными колбами и, особенно, стеклянными трубками
предоставляет широкие возможности для травмирования рук;
- датчик температуры касается дна чайника (нагревательного элемента) и показывает завышенную
температуру;
- не проверили датчик давления в начале опыта, а он даёт утечку.
- установка не герметична. Неплотные соединения, плохо закрыта пробка.
- колба не полностью погружена в воду - нет гарантии теплового равновесия датчика температуры
и газа в колбе;
- датчик температуры и колба находятся в разных местах чайника, где температура воды разная;
- датчик висит на шланге, шланг портится, особенно при высоких температурах;
- вода выплескивается из чайника при кипении - это плохо и опасно;
- на столе беспорядок – лишние предметы вредны при работе с кипятком.
А.Е.Тарчевский, шк.179 МИОО, 10 апреля 2011
40
ЛАБОРАТОРНАЯ РАБОТА № 9
ИЗМЕРЕНИЕ ДАВЛЕНИЯ ПАРА АЦЕТОНА
ПРЕДВАРИТЕЛЬНЫЕ ТРЕБОВАНИЯ:
Знание газовых законов, знание темы "Пар".
ОБОРУДОВАНИЕ: - Вариант А
Обычная прозрачная пластиковая бутылка объёмом 1 литр, герметично закрывающаяся
пластиковой пробкой. В эту пробку вставлен переходничок от системы для переливания
крови для подключения датчика давления (и тщательно приклеен, чтобы не было утечки), а
также пропущена (и тоже герметично) игла от шприца для подключения шприца, через
который в бутылку добавляется ацетон.
УСТРОЙСТВО
ПРОБКИ
ШПРИЦ С
АЦЕТОНОМ
ДАТЧИК
ДАВЛЕНИЯ
МЕСТА
СОЕДИНЕНИЙ
ТЩАТЕЛЬНО
ПРОКЛЕЕНЫ
БУМАГА, НА
КОТОРОЙ ВИДНО,
ИСПАРИЛСЯ ЛИ
ЖИДКИЙ АЦЕТОН
- Вариант Б (менее удобный)
прозрачная колба (а лучше и безопаснее – пластиковая
бутылка) объёмом 0,4 - 1 литр, герметично закрывающаяся
резиновой пробкой. Сквозь эту пробку пропущено две
иглы от шприца – одна - для подключения датчика
давления (через переходничок от системы для переливания
крови), другая - для подключения шприца, через который в
бутылку добавляется ацетон. Пробка должна плотно
держаться и выдерживать избыточное давление в бутылке
не менее 30кПа. В пробке не должно быть никаких лишних
отверстий, в том числе от прокалывания иглами, чтобы не
возникло утечки.
41
На дно бутылки (в то место, куда будет попадать ацетон из шприца) рекомендуется положить
кусочек белой шероховатой бумаги, чтобы капелька ацетона испарялась быстрее, а также
чтобы видеть, присутствует ли жидкость в бутылке или всё уже испарилось. (Это не
обязательно, но работа " с бумагой" получается лучше и нагляднее. На результаты наличие
бумаги не влияет.)
- шприц с пластиковым (не резиновым) поршнем на 1-2 мл с ацетоном (в опыте используем
около 0,5-1 мл ацетона)
- NOVA, датчик давления, датчик температуры или термометр,
- справочник "Физические величины" (желательно),
- ацетон в герметично закрывающемся пузырьке.
ЦЕЛЬ РАБОТЫ:
- наблюдение установления давления пара ацетона в бутылке при постоянной комнатной
температуре.
- измерение давления насыщенного пара ацетона при комнатной температуре.
ВЫПОЛНЕНИЕ РАБОТЫ:
ПОДГОТОВКА:
- Перед началом работы проверьте датчик давления. Иногда неплотно надет шланг, и есть
утечка. Рекомендуется подсоединить к датчику сухой шприц на 2-10 мл. с резиновым
поршнем, включить регистрацию данных, создать давление 200- 400 кПа, убедиться, что оно
не падает со временем в течение 10-20 секунд.)
- Измерим температуру в комнате термометром или датчиком температуры.
Запишем температуру.
- Подключим к NOVA датчик давления.
ОСТОРОЖНО!
- Пары ацетона ядовиты и весьма пожароопасны. Категорически запрещается зажигать
спички и зажигалки рядом с местом проведения опыта или там, где сушится бутылка.
- Бутылку с парами ацетона можно открывать и сушить только в вытяжном шкафу или вдали
от людей в проветриваемом помещении. Спросите учителя, где. Также не надо отключать от
бутылки датчики в кабинете- после опыта в бутылке избыточное давление, поэтому часть
вонючих паров выйдет из бутылки.
- Соберём установку. Осторожно, но плотно подключим шланг датчика давления.
Обеспечим, чтобы установка стояла удобно, датчик НЕ висел на своём шланге, ацетон
попадал на вложенную в бутылку бумажку. Можно пользоваться штативом.
- В маленький шприц наберём около 0,5 мл ацетона (для бутылки 0,5 литра) и подсоединим
шприц (ацетон до начала опыта не должен попадать внутрь бутылки – важно!) Запишите,
сколько жидкости было в шприце - в конце опыта потребуется узнать, сколько ацетона
испарилось в бутылке.
- В опыте давление будет повышаться от атмосферного до чуть большего. Если ничего не
сделать, масштаб давления на графике "начнется" от нуля. Чтобы увидеть что-то интересное
на графике, надо сразу отказаться от автоматического масштабирования и "вручную" задать
масштаб, например, от 99 до 129 кПа. Выводим на экран, разумеется, график зависимости
давления от времени.
42
- Установим частоту 10 замеров в секунду, время эксперимента – непрерывно.
ЭКСПЕРИМЕНТ:
Запустим эксперимент.
В ходе опыта не надо держать бутылку руками и нагревать или охлаждать её. Постоянная
температура важна. Не надо открывать окно рядом с установкой во время опыта.
- Подождём 20-30 секунд, чтобы убедиться, что давление в бутылке постоянно.
("Постоянно"- значит, меняется мало по сравнению с изменениями давлений из-за испарения
ацетона, с которыми мы будем иметь дело.) Возможно, за бутылку подержались рукой и она
остывает - тогда давление будет понижаться.
Аккуратно (не нагревая бутылку руками и дыханием) капнем внутрь совсем немного (одну
или несколько капель) ацетона. Для чистоты эксперимента можно каждый раз возвращать
поршень шприца в исходное положение, чтобы не изменять объём, (хотя это почти не влияет
на результат).
Наблюдаем график давления. Давление должно расти, пока весь ацетон не испарится. Это
легко отследить по "мокрому" пятну на бумаге внутри бутылки. Когда давление перестанет
расти, капнем ещё немного ацетона, затем ещё и ещё пока ацетон не "перестанет" испаряться,
а давление не выйдет на "полочку". В этом случае устанавливается динамическое равновесие
- число испарившихся за некоторое время молекул равно числу сконденсировавшихся.
Установившееся избыточное давление - это давление насыщенного пара ацетона. (При этом
на дне должна обязательно находиться лужица жидкости или бумага должна быть мокрой.)
Упрощенный вариант: Получим 2-3 явных полочки, и, когда иссякнет терпение, выльем в
бутылочку весь ацетон из шприца.
Остановим запись результатов.
Сохраним полученный файл.
Унесём закрытую бутылку (НЕ ОТСОЕДИНЯЯ ОТ БУТЫЛКИ ДАТЧИК!) сушиться в
проветриваемое место или в вытяжной шкаф, и только там её откроем. Не заставляйте себя и
окружающих вдыхать вредные вонючие пары.
Если надо повторить опыт, хорошо высушите бутылку или возьмите другую.
ОБРАБОТКА РЕЗУЛЬТАТОВ:
1. График давления.
Распечатаем график, (сделаем масштаб хорошим, единицы измерения - правильными,
количество знаков - верным, подпишем график). Будем готовы объяснить, что, где и почему
происходило.
2. Сравнение получившегося давления насыщенного пара с данными справочника.
Запишем давление насыщенного пара ацетона и температуру, при которой выполнен опыт.
Справочник "Физические величины" (таблица "давление насыщенных паров органических
веществ") на стр. 268 предлагает таблицу давлений насыщенного пара ацетона при
различных температурах. (Откройте справочник! Здесь таблица приведена для тех, у кого нет
справочника!)
давление, кПа
температура 0С
5
-10
10
3
20
17
50
37
100
55
43
По этим данным построим на миллиметровке или бумаге в клеточку график зависимости
давления насыщенного пара ацетона от температуры. По графику определим, каким
должно быть давление пара в вашем опыте.
Сравним получившееся давление со справочными данными. Сделаем выводы.
3. Сравнение испарившегося объёма ацетона с расчётным.
Запишем объём бутылки и количество (объём) испарившегося жидкого ацетона.
Оценим (рассчитаем), какая масса и объём жидкого ацетона может испариться в бутылке.
Аккуратно запишем вывод формулы и расчёты. (Просто применим уравнение МенделееваКлапейрона. Формула ацетона C3H6O, отсюда легко узнать его молярную массу. Давление
насыщенного пара известно. Плотность ацетона 791 кг/м3.) Сравним с тем количеством
ацетона, которое вы залили в бутылку до момента установления динамического равновесия,
запишем вывод.
4. Не забудьте поставить на место высушенную бутылку.
ТИПИЧНЫЕ ОШИБКИ В ЭТОЙ РАБОТЕ:
- Установка не герметична. Неплотные соединения, плохо закрыта пробка. Если есть сомнения в
герметичности установки, не поленитесь проверить, нет ли утечки. Можно заглушить отверстие,
куда вставляют шприц (специальной пробочкой или пальцем), поместить пробку под воду и, сжав
пластиковую бутылку, посмотреть, не выходят ли пузырьки.
Если при выполнении работы давление в бутылке, установившись начинает снижаться, есть
утечка.
Подсказка - в конце опыта должно получиться давление около 120 кПа.
ДОПОЛНИТЕЛЬНОЕ ИССЛЕДОВАНИЕ:
Можно в бутылку, в которую понемногу капаем ацетон, положить таблетку (или несколько
таблеток) активированного угля (уголь продаётся в аптеке). Как известно, это вещество,
имеющее огромную площадь поверхности, будет поглощать пары ацетона, поэтому давление
должно постепенно падать. (Кстати, это свойство активированного угла позволяет применять
его в противогазах). Только добейтесь, чтобы жидкий ацетон не попадал на таблетку.
Попробуйте получить красивый результат, в частности, дойти до состояния, когда уголь
перестанет поглощать пары ацетона. Вероятно, потребуется некоторое время на поглощение
паров ацетона. Постройте график зависимости давления от времени, показывающий
поглощение паров ацетона. Запишите выводы.
ЗАМЕЧАНИЕ
Не надо брать спирт вместо ацетона. Он содержит воду и вместо давления паров спирта
получается … не сумма парциальных давлений, а … давление, значительно меньшее
давления паров чистого спирта из-за взаимодействия его с водой. Как обычно, водка не
приводит к хорошим результатам.
А.Е.Тарчевский, шк.179 МИОО, 10 апреля 2011
44
ЛАБОРАТОРНАЯ РАБОТА № 9 -А
ИЗМЕРЕНИЕ ДАВЛЕНИЯ В ГАЗОВОЙ ЗАЖИГАЛКЕ
Эта небольшая работа может быть выполнена вместе с работой
"Измерение давления насыщенных паров ацетона"
ОБОРУДОВАНИЕ
- Обычная прозрачная одноразовая зажигалка, с которой снят металлический колпачок
вместе с колёсиком и кремнем (то есть сама зажигалка поджечь выходящий газ не может).
Подключение датчика давления к зажигалке
осуществляется с помощью отрезка пластиковой
трубки от использованного стержня обычной
авторучки. На один конец стержня надевается
шланг
датчика,
другой
конец
полотно
прижимается к выходящей из зажигалки
металлической трубочке во время опыта. Выпуск
газа осуществляется простым нажатием на рычаг
зажигалки.
Зажигалка имеет регулировочный рычаг. Чтобы
опыт выполнялся быстрее, надо повернуть рычаг
в положение максимального расхода газа.
Желательно
иметь
две
подготовленных
зажигалки, чтобы сразу охладить одну из них и измерить давление при пониженной
температуре.
- NOVA, датчик давления.
ВНИМАНИЕ!
Не должно быть рядом никакого открытого огня.
С зажигалки должно быть обязательно снято колёсико и кремень.
Делайте опыт в проветриваемом помещении или в вытяжном шкафу.
НЕ ДОПУСКАЙТЕ УТЕЧКИ ГАЗА.
Опыт делается в присутствии учителя.
ЗАМЕЧАНИЕ: ОСОБЕННОСТИ ДАТЧИКОВ ДАВЛЕНИЯ, ИСПОЛЬЗУЕМЫХ В НАШЕЙ ШКОЛЕ:
Датчики давления лаборатории "Архимед" работают со значительными ошибками. Если они до
сих пор не исправлены (спросите учителя), необходимо пересчитать данные по формуле P =
1,21*Pдат ч - 22,6 (Здесь Рдатч- измеренное давление, Р- верное). При помощи этой функции
можно скорректировать данные непосредственно в программе Multilab. (Следует отметить, что
через мастер анализа выполнить это простое линейное преобразование на наших NOVA-5000 не
получалось!) В этой работе без корректировки данных ошибки измерения давления -до 20%, и их
легко исправить, пользуясь формулой.
0. ПОДГОТОВКА:
- В конце работы потребуется охлажденная зажигалка, поэтому сразу до выполнения работы
положите одну зажигалку в морозильник, там -180С.
(Вариант- в холодную воду, в которой тает снег- там 00С).
- Перед началом работы проверьте датчик давления. Иногда неплотно надет шланг, и есть
утечка. Рекомендуется подсоединить к датчику сухой шприц на 2-10 мл. с резиновым
поршнем, включить регистрацию данных, создать давление не менее 300-400 кПа,
убедиться, что оно не падает со временем в течение 10-20 секунд.)
45
1. Измерьте давление в зажигалке при комнатной температуре (её надо измерить и
записать). Для этого плотно подсоедините датчик давления, запустите измерения, нажатием
на рычаг откройте клапан. ВНИМАНИЕ! Не должно быть утечки газа. Если есть утечка,
немедленно прекратите опыт и добейтесь плотного соединения зажигалки с переходником,
затем повторите опыт. Когда давление полностью установится (для этого может
потребоваться десяток-другой секунд - отследим нужную длительность опыта по графику
давления и по тому моменту, когда капельки жидкости в трубке перестанут двигаться)
закончите опыт, потом отсоедините датчик.
Запишите полученное установившееся давление.
2. Выполните такой же опыт при пониженной температуре для зажигалки, которая полежала
в морозильнике или другом холодном месте. Чтобы зажигалка не нагрелась во время
проведения опыта, теплоизолируйте её (обмотайте салфетками и т.п.), выполняйте опыт
быстро! Запишите полученное установившееся давление.
3. Ниже приведена часть таблицы из справочника "Физические величины"- "Давление паров
органических веществ" для БУТАНА С4H10 (для тех, у кого нет справочника):
50 кПа - 254К
100 кПа - 270К, (стр. 269 справочника)
148 кПа - 10 0С, (стр. 281 справочника)
207 кПа - 20 0С
283 кПа - 30 0С
378 кПа - 40 0С
…………….
1529 кПа - 1000С
Сравните результаты измерений с имеющимися справочными данными.
Поскольку температуры ваших зажигалок, вероятно, не совпадают с теми температурами, для
которых в справочнике приведены давления паров, постройте график зависимости
давления паров от температуры по данным справочника, потом возьмите данные для ваших
температур из этого графика.
Можно ли утверждать по результатам опыта, что в зажигалке находится чистый бутан?
4. ОТВЕТЬТЕ НА ВОПРОСЫ:
- Могло ли и как наличие воздуха в шланге датчика значительно повлиять на результаты
опыта? Почему?
- Сколько времени вы охлаждали зажигалку и какую температуру, как вы считаете, имел
охлаждённый газ?
- Почему газовая плитка плохо работает на морозе?
- В некоторых аэрозолях жидкий бутан используют для поддержания давления внутри
баллона. Изменяется ли давление в баллоне по мере использования газа?
- Почему нельзя нагревать аэрозольный баллончик? Приведите соответствующие
табличные данные.
ТИПИЧНЫЕ ОШИБКИ В ЭТОЙ РАБОТЕ:
- Забыли охладить зажигалку заранее- на это потребуется минут 20!
- Охладив зажигалку в морозильнике, не теплоизолировали её (например, обмотав салфетками),
или же долго держали в тепле, вместо того, чтобы немедленно провести измерение.
46
- плохое подсоединение зажигалки к датчику давления. При этом НЕДОПУСТИМА утечка газа.
Даже не пытайтесь проводить измерения, если ядовитый и горючий газ, шипя, вырывается
наружу!
А.Е.Тарчевский, шк.179 МИОО, март 2010
47
ЛАБОРАТОРНАЯ РАБОТА № 10
ИЗУЧЕНИЕ ЗАВИСИМОСТИ ДАВЛЕНИЯ
НАСЫЩЕННГО ПАРА ВОДЫ ОТ ТЕМПЕРАТУРЫ
ОБОРУДОВАНИЕ
- вода, электрический чайник, NOVA, датчик давления.
- Вариант А- герметичный металлический сосуд с небольшим количеством воды, подключаемый к
датчику давления.
БАЛЛОНЧИК С
ВОЗДУХОМ И ВОДОЙ
(воды -около 2 мл).
НАРЕЗАНА РЕЗЬБА,
ОТВЕРСТИЕ ПЛОТНО
ЗАКРЫТО ВИНТОМ С
РЕЗИНОВОЙ
ПРОКЛАДКОЙ
БАЛЛОНЧИК С
СУХИМ
ВОЗДУХОМ
- Вариант В (лучше) - Два герметичных металлических сосуда, оборудованные трубками для
подключения к датчику давления. Хорошо подойдёт баллончик с уже подсоединённой медной
трубкой, который находится рядом с насосом выброшенного на свалку холодильника. Один сосуд - с
небольшим количеством воды, другой - строго сухой. Сосуды одновременно подключают к двум
датчикам давления и опускают в чайник с водой.
(Сосуды должны надёжно выдерживать давление 200 кПа, не быть стеклянными (опасно!), не деформироваться от
нагревания в кипятке. Можно попробовать использовать алюминиевые сосуды от использованных баллончиков с
распыляемыми лекарствами типа "Каметон". В выходной пластиковой трубке такого баллончика вдоль её оси
просверливается отверстие, то есть ликвидируется клапан, а сам сосуд остается крепким и стойким к нагреванию выше
100°С. Однако возникает вопрос- как отмыть баллончик от остатков масла изнутри)
ПРЕДВАРИТЕЛЬНЫЕ ТРЕБОВАНИЯ
Знание газовых законов, знание темы "пар".
ЦЕЛИ РАБОТЫ:
Получить зависимость давления насыщенного пара воды от температуры в диапазоне
от 0-150С до 100 0С. Сравнить давления в "сухом" и "мокром" сосудах.
Сравнить полученные значения давлений насыщенного пара со справочными данными.
МЕРЫ БЕЗОПАСНОСТИ:
Установите компьютер в удобном безопасном месте, исключающем возможность попадания на него
воды.
Осторожно обращайтесь с горячим чайником! Не оставляйте чайник с кипятком без присмотра, если
кипяток уже не нужен – сразу вылейте его. Выливая кипяток в керамическую раковину, лейте
медленно, при этом откройте кран с водой, иначе раковина может треснуть.
Чайник работает от сети 220В! Не трогайте провода, имейте сухие руки, не ставьте чайник в
раковину и на мокрые поверхности, не лейте воду куда не надо.
ТЕОРИЯ
Прочитайте и поймите, затем прочитайте ещё раз и выполните:
48
Герметичный сосуд, содержащий воздух и немного воды, подключенный к датчику давления,
поместим внутрь чайника с холодной (лучше- ледяной) водой, включим чайник, вода вместе с
сосудом будет нагреваться. Будем измерять давление в сосуде и температуру воды в чайнике (она
равна температуре в сосуде). Затем построим зависимость давления от температуры.
Но всё не так просто… Проблема в том, что изначально в сосуде есть воздух, давление которого
будет изменяться с увеличением температуры и добавляться к давлению пара. Можно было бы
избавиться от воздуха, заставив воду в сосуде кипеть некоторое время, (тогда воздух был бы вытеснен
паром), но, к сожалению, датчики давления не умеют измерять низкие (ниже 30 кПа) давления.
Поэтому (вариант А) придется сделать опыт с водой и воздухом вместе, затем рассчитать давление
воздуха. (Это несложно, процесс для воздуха изохорный, начальное атмосферное давление измерено,
начальная температура – тоже.) Возможно использование результатов работы "Изучение изохорного
процесса" для нахождения давлений сухого воздуха при различных температурах.
Затем из общего давления в сосуде надо вычесть это расчетное давление воздуха.
ВАРИАНТ В - более "честный"- вместо расчета давления сухого воздуха будем в ходе опыта
измерять его во втором сосуде с сухим воздухом. Сосуды могут быть и разного размера - ведь
давление не зависит от этого, но важно, чтобы было тепловое равновесие между ними. Затем из
графика суммарного давления вычтем график давления сухого воздуха. (Вариант В не опробован
экспериментально. Вариант А в наших опытах давал заниженные значения давлений пара, особенно при приближении температуры к 100оС,
что, возможно, вызвано конденсацией пара в шланге датчика давления.)
ОПЫТ:
Если надо, подключим компьютер к блоку питания.
Подключим к компьютеру датчики давления и температуры, установим частоту 10 измерений в
секунду, продолжительность опыта не менее 10 минут.
Перед началом работы проверьте датчики давления. Иногда неплотно надет шланг, и
есть утечка. Рекомендуется подсоединить к датчику сухой шприц на 2-10 мл. с резиновым
поршнем, включить регистрацию данных, создать давление 200- 400 кПа, убедиться, что оно
не падает со временем в течение 10-20 секунд.)
В сосуд (он герметично закрыт, единственное отверстие- трубочка для подсоединения датчика
давления) шприцем с иглой нальём 3 - 5 мл воды, поместим сосуд в чайник. Вода в чайнике должна
быть холодной (а лучше-ледяной) - это даст возможность пренебречь давлением водяного пара в
начале опыта, Если на улице зима, не поленитесь сходит за снегом для охлаждения воды. Плотно
подсоединим шланг датчика давления. Позаботимся, чтобы шланг не соскакивал и не перегибался,
чтобы датчик не висел на шланге, чтобы пар при кипении воды не попадал на датчик. Можно
пользоваться штативом.
Вариант В - в тот же
чайник опустим и сосуд с
"сухим"
воздухом,
подключим
к нему
другой датчик давления.
Следите, чтобы в "сухой"
сосуд не попала вода!
Обеспечим, чтобы сосуды
и датчик температуры не
касались нагревательного
элемента внутри чайника.
Запустим
регистрацию
данных.
Включим чайник (чтобы
он нагревался медленнее,
желательно включить его
через
регулятор
напряжения или диод,
понижающий мощность
49
вдвое).
Доведем воду в чайнике до кипения и секунд 30 аккуратно покипятим воду, заботясь о том, чтобы она
не выплёскивалась. (Если вода слишком сильно кипит, можно периодически кратковременно
выключать чайник.)
Остановим измерения, аккуратно выльем кипяток.
Сохраним полученный файл.
ЗАМЕЧАНИЕ: ОСОБЕННОСТИ ДАТЧИКОВ ДАВЛЕНИЯ, ИСПОЛЬЗУЕМЫХ В НАШЕЙ ШКОЛЕ:
Датчики давления лаборатории "Архимед" работают со значительными ошибками. Если они до
сих пор не исправлены (спросите учителя), необходимо пересчитать данные по формуле P =
1,21*Pдат ч - 22,6 (Здесь Рдатч- измеренное давление, Р- верное). При помощи этой функции
можно скорректировать данные непосредственно в программе Multilab. (Следует отметить, что
через мастер анализа выполнить это простое линейное преобразование на наших NOVA-5000 не
получалось!) В этой работе без корректировки данных ошибки измерения давления будут
значительными, НЕОБХОДИМО скорректировать данные, хотя бы даже и пересчитав их по
формуле "вручную".
Давление насыщенных паров воды – фрагмент таблицы из справочника "Физические величины", стр. 254.
t, 0C
0
6
10
20
30
40
50
60
Р, кПа
0,61
0,94
1,22
2,33
4,24
7,37
12,3
19,9
t, 0C
Р, кПа
70
31,1
80
47,3
86
60,1
90
70,1
94
81,5
96
87,7
98
94,3
100
101,3
Вариант А: По данным опыта заполним таблицу со столбцами:
Температура, Общее давление, Расчетное давление воздуха (его найдём по уравнению P/T=P0/T0),
Искомое давление пара, Табличное значение давления пара. Данные надо снимать через 5-10 градусов
(а на некоторых участках чаще). Для снятия данных используйте курсор.
Выбрав правильный масштаб, по этой таблице построим на миллиметровке
"экспериментальный" график зависимости давления насыщенного пара воды от температуры и на
этом же листе - "теоретический" график по табличным данным.
(Попытки заставить "NOVА" построить график за вас, вероятно, приведут к огромным потерям
вашего времени).
Вариант В:
Получим графики зависимости давления в сосуде "с водой" и давление в сосуде "без воды" от
температуры. (Выберем в качестве аргумента температуру.)
Вычтем из давления в сосуде "с водой" давление в сосуде "без воды". Получим зависимость давления
насыщенного водяного пара от времени. Обеспечим хороший масштаб, подпишем график и выведем
его на печать.
На этом же листе построим (нарисуем) график по табличным значениям давления.
Запишем выводы по работе. Проанализируем, возможно, результаты не полностью соответствуют
теории, постараемся объяснить, почему.
А.Е. Тарчевский шк.179 МИОО, 10 апреля 2011
50
ЛАБОРАТОРНАЯ РАБОТА № 11
ИЗУЧЕНИЕ ИЗОТЕРМИЧЕСКОГО И АДИАБАТНОГО ПРОЦЕССОВ
ПРЕДВАРИТЕЛЬНЫЕ ТРЕБОВАНИЯ –
знание газовых законов, изопроцессов, адиабатного процесса.
ОБОРУДОВАНИЕ:
NOVA, датчик давления, термометр или датчик температуры, принтер, шприц
приблизительно на 2-3, 5- 10, 20-25 мл с поршнями, имеющим резиновые уплотнения (не
изношенные).
ЦЕЛИ РАБОТЫ:
- Оценить быстроту выравнивания температуры после быстрого сжатия газа в шприце с
температурой окружающей среды.
- Проверить выполнение закона Бойля-Мариотта.
ЗАМЕЧАНИЕ: ОСОБЕННОСТИ ДАТЧИКОВ ДАВЛЕНИЯ, ИСПОЛЬЗУЕМЫХ В НАШЕЙ ШКОЛЕ:
Датчики давления лаборатории "Архимед" работают со значительными ошибками. Если они до
сих пор не исправлены (спросите учителя), необходимо пересчитать данные по формуле P =
1,21*Pдат ч - 22,6 (Здесь Рдатч- измеренное давление, Р- верное). При помощи этой функции
можно скорректировать данные непосредственно в программе Multilab. (Следует отметить, что
через мастер анализа выполнить это простое линейное преобразование на наших NOVA-5000 не
получалось!) В этой работе без корректировки данных ошибки измерения значительны,
обязательно надо пересчитать давления!
ОПЫТ 1.
ОЦЕНКА ВРЕМЕНИ ВЫРАВНИВАНИЯ ТЕМПЕРАТУРЫ после быстрого сжатия газа с
температурой окружающей среды.
1. При выполнении этой работы необходимо сначала проверить датчики давления. Для этого
можно подсоединить шприц объёмом 2-3 мл с резиновым поршнем, сильно, до упора, сжать
воздух, подержать 10-20 секунд в таком состоянии, затем посмотреть, вернется ли поршень в
первоначальное положение (можно чуть "помогать" ему). Если поршень получил новое
"устойчивое" положение - была утечка. (Давление при этом должно превышать 400 кПа.)
Второй вариант- можно подсоединить к датчику шприц на 5-10 мл. с резиновым поршнем,
включить регистрацию данных, создать давление 300- 400 кПа, убедиться, что оно не падает
со временем в течение 15-20 секунд. Поршень шприца, конечно, надо удерживать
неподвижно. Если есть утечка - работа не получится.
2. Подключите шприц на 2 - 10 мл к датчику давления.
Все соединения шлангов должны быть плотными, чтобы не допустить утечки сжатого газа.
3. Установите частоту 200 - 500 замеров в секунду, длительность- 4 - 5 секунд (на такой
частоте график будет выведен на экран только после выполнения опыта!)
Запустите измерение, через 1 секунду очень резко нажмите на поршень шприца.
Удерживайте поршень нажатым строго неподвижно около 4 секунд.
Добейтесь, чтобы на графике был явно виден небольшой "пик" давления.
4. Выберите "интересный" фрагмент графика, где видно, что давление после сжатия
достигает пикового значения, а затем быстро снижается до некоторого постоянного уровня,
что связано с установлением теплового равновесия с окружающей средой.
51
Выделите из графика "интересную" часть, сделайте хороший масштаб, подпишите и
напечатайте график.
5 Оцените время установления теплового равновесия с окружающей средой. Укажите это
время на вашем графике.
6 По полученным данным оцените максимальную температуру, до которой нагрелся воздух
в шприце в ходе опыта. Измерить ее не получится, а вот рассчитать с помощью газовых
законов легко. Нужные данные возьмите с графика. Подсказка - после того, как газ был сжат
(почти адиабатически), с ним произошел изохорный процесс, конечная температура известна
(комнатная).
7 Запишите выводы по этой части исследования, объясните, почему газ нагревается при
сжатии.
ОПЫТ 2
ИЗУЧЕНИЕ ИЗОТЕРМИЧЕСКОГО ПРОЦЕССА
Частота 10 измерений в секунду, количество измерений - непрерывно.
Будем ступенчато уменьшать объем шприца на 25 мл с целью получить значения давления
для объёмов, например, 25, 20, 15, 10, 5 мл., а лучше - чаще (например, 25, 22, 19, 16, 13 мл. и
т. д.) и обратно для тех же значений объёма.
Соединения должны быть плотными, чтобы не допустить утечки сжатого воздуха. Поршень
держите ровно, без перекоса.
При каждом значении объёма надо задержать поршень в неподвижном состоянии на
несколько секунд, чтобы установилось тепловое равновесие с окружающей средой.
Пользуйтесь ранее полученными данными о времени выравнивания температуры.
Чтобы посмотреть, была ли хотя бы небольшая утечка, проведем опыт при сжатии газа,
затем, не прекращая измерений, при расширении (для тех же значений объёма). Если график
для расширения имеет меньшие значения давления, чем график для сжатия - значит, была
утечка воздуха. ("Обратно" - можно (но нежелательно) "схалтурить", получить только
давление при значении объёма 25 мл.)
После построения графика на экране считаем с него нужные данные (для этого вызовем
курсор, перетащим его в нужную точку его стилусом, под графиком появится значение
давления для данной точки). Составим в тетради таблицу зависимости давления
(установившегося) от объёма, затем на бумаге (миллиметровке) аккуратно построим
изотерму P(V), подпишем график, позаботимся о правильном масштабе.
НЕ ЗАБУДЬТЕ О НЕОБХОДИМОСТИ ПЕРЕСЧЁТА ДАВЛЕНИЙ, ЕСЛИ ВАШИ ДАТЧИКИ ТРЕБУБТ
КАЛИБРОВКИ!
Возьмём начальный объём, при котором давление было атмосферным (оно известно измерено как давление несжатого газа) и, используя закон Бойля-Мариотта, рассчитаем
"теоретические" давления для всех значений объёма, для которых мы проводили измерения.
Построим другим цветом получившийся "теоретический" график на этом же листе.
Запишем выводы по работе.
Возможно, графики не полностью совпали, постарайтесь объяснить, почему.
52
Типичные ошибки в этой работе:
- Установка не герметична. Неплотные соединения. При больших давлениях некоторые датчики травят.
Датчик не усовершенствован – тогда он травит наверняка. Делайте указанные проверки.
- Имеется незаметная трещина в пластмассовом переходничке датчика давления.
- Использование старого шприца или шприца с пластиковым поршнем. Будет утечка.
- Перекашивание поршня во время опыта может привести к утечке.
- Попытки применять что-то без понимания того, что человек делает… незнание газовых законов…
- Попытки заставить компьютер построить график изотермы вместо честного построения его "вручную".
- При построении изотермы забыли учесть первую точку для атмосферного давления (когда газ ещё не
начали сжимать)
- не сделали опыт (№2) для сжатия, затем расширения газа
- Сильный юноша заставил слабую девушку сжимать газ в шприце. Это тяжело!
А.Е. Тарчевский, шк.179 МИОО, 10 апреля 2011
53
ЛАБОРАТОРНАЯ РАБОТА № 12
ИССЛЕДОВАНИЕ ПРОЦЕССА ЗАРЯДКИ И РАЗРЯДКИ КОНДЕНСАТОРА
ОБОРУДОВАНИЕ
NOVA, батарейка 4,5 В, датчики напряжения и тока (250мА), два конденсатора
электролитических (ёмкость порядка 470 мкФ), резистор (около 1,5 кОм), провода, ключ
ПРЕДВАРИТЕЛЬНЫЕ ТРЕБОВАНИЯ
знание темы "Конденсаторы", "Электрический ток".
ЦЕЛИ РАБОТЫ
- построить график зависимости напряжения на конденсаторе от времени.
- понять, за какое время заряжается конденсатор от батарейки,
- сравнить время зарядки и разрядки конденсатора через тот же самый резистор,
- исследовать зависимость времени зарядки от ёмкости конденсатора - проверить, является ли
график U(t) экспонентой U=U0e-t/(RC) и действительно ли показатель экспоненты -t/(RC),
- сравнить график ∫idt и график заряда, накопленного конденсатором q = U/C
ТЕОРИЯ
Выводится формула для напряжения на разряжающемся через сопротивление R конденсаторе
U=U0e-t/(RC) (Из условия равенства напряжений на R и С получается дифференциальное
уравнение для заряда, и оно просто решается, но для этого надо уметь брать интегралы.
Вывод формулы можно попросить у учителя - он написан на отдельном листочке. Весьма
полезно разобраться с этим выводом, а ещё лучше- вывести формулу самостоятельно. )
Здесь U0- начальное напряжение на конденсаторе, t- время, прошедшее с момента начала
разрядки.
Ход работы:
1. Познакомимся с теорией
2. Посмотрим, не разряжается ли конденсатор через датчик напряжения или сам по себе. Для
этого зарядим его. НЕ ЗАБУДЬТЕ ПРАВИЛЬНО ПОДКЛЮЧИТЬ + и – КОНДЕНСАТОРА К
БАТАРЕЙКЕ! Затем подсоединим конденсатор к датчику напряжения, например, на 100
секунд. Частота замеров может быть невысокой, (10 в секунду) главное- чтобы график
выводился на экран во время опыта).
Запишем результаты. Разряжается ли конденсатор на заметную величину? Если да- при
проведении дальнейших опытов время зарядки и разрядки должно быть существенно меньше
времени, за которое конденсатор разряжается через датчик.
3. Соберём установку (см. схему ниже), в качестве амперметра и вольтметра подключим
датчики тока и напряжения.
ВНИМАНИЕ : НЕ ЗАБУДЬТЕ ПРАВИЛЬНО ПОДКЛЮЧИТЬ + и – КОНДЕНСАТОРА!
Электролитический конденсатор может иметь большой ток утечки или выйти из строя при
неправильной полярности подключения!
ВНИМАНИЕ: Типичная ошибка- неверно собрана схема.
54
+
V
A
R
С
+
4. Сделаем пробный опыт. Частота- 25 измерений в секунду, количество- непрерывно. Цельполучить график U(T), на котором хорошо виден процесс зарядки. Оценим время зарядки.
(При рекомендуемых параметрах время зарядки составляет 2-5 секунд)
5. Теперь сделаем опыт "начисто". Установим частоту, например, 100 измерений в секунду,
(при такой частоте график появится на экране только после завершения опыта). Будем
заряжать конденсатор, например, 10 секунд, затем разряжать такое же время. (Время
подбираем так, чтобы конденсатор успел полностью зарядиться и разрядиться,
руководствуясь данными пробного опыта.)
4. По графику определим время зарядки и разрядки, считая конденсатор заряженным, когда
напряжение достигнет 98% от установившегося значения. (Не "на глаз"! Используем курсор.)
Запишем результаты. Сравним время зарядки и разрядки. Подберём хороший масштаб,
подпишем график. Распечатаем график U(t).
5. Увеличим вдвое ёмкость, соединив два конденсатора параллельно. Запишем время зарядки
и разрядки. (По теории должно быть t2 = t1C2/ C1 )
Теория: Пусть конденсатор ёмкостью С1 разрядился от напряжения U0 до U, при этом
прошло время t1. Тогда U=U0e-t1/(RC) Вывод этой формулы есть в книгах или у учителя на
отдельном листе. (Понимание этого вывода требует хотя бы начального знания интегралов).
Аналогично для конденсатора С2, разряжающегося через то же самое сопротивление R, до
того же напряжения U, получим U= U0e-t2/(RC2).
U0e-t1/(RC1) =U0e-t2/(RC2) или t1/C1 =t2/C2, или t2 = t1C2/ C1
Проверим, так ли это, запишем вывод.
6. За время 1/RC напряжение на конденсаторе должно падать в e раз (это легко видно из
формулы) Посмотрите по графику, за какое время напряжение падает в
e = 2,718 281 828 459 045 235 360 287 471 352 662 497 757 :) раза, запишите результат.
Рассчитайте 1/RC, сравните с экспериментальными данными.
7. ДЛЯ ТЕХ, КОМУ ИНТЕРЕСНО, и кто не боится возможных "глюков" программы
Multilab:
Возьмём данные для одного конденсатора и проверим, является ли зависимость U(t)
экспонентой U=U0e-t/(RC)
Теория: Если прологарифмировать зависимость, получится: ln (U0e-t/(RC)) = lnU0 - t/(RC)
График получившейся функции- прямая с наклоном -1/RC. Проверим, так ли это.
- Для этого вызовем два курсора, вырежем участок графика U(t), соответствующий процессу
разрядки.
- Позаботимся, чтобы напряжение ни в одной точке графика не принимало отрицательных
значений, иначе будут проблемы с логарифмированием. Если отрицательные напряжения
есть (проверьте это курсором) - прибавим к исследуемой функции очень малую величину.
(Инструменты- Операция- Линейная, в поле В поставим нужную величину.)
- Прологарифмируем зависимость. (Инструменты- Анализ- Мастер анализа - Логарифм)
55
- Аппроксимируем её прямой. (Инструменты- Анализ- Мастер анализа- Линейное
приближение)
Сравним полученную зависимость с прямой линией. Сделаем вывод. Распечатаем графики.
Увидим на экране уравнение этой прямой. (х- это t) (Если уравнения под графиком нет,
поставьте курсор на эту прямую.) Запишем его.
- Сравним коэффициент при переменной t и расчётное значение -1/RC
Сделаем вывод.
ВНИМАНИЕ: В Multilab при логарифмировании или аппроксимации полученной зависимости
экспонентой часто возникают проблемы (как на NOVA-5000, так и на компьютере), устранить их
не удаётся. При этом получившиеся зависимости- действительно экспоненты (проверяли). Один
из возможных путей- экспорт таблицы из Multilab в Excel и дальнейшая обработка данных в Excel.
8. Теория:
Интеграл тока должен быть равен заряду, накопившемуся в конденсаторе.
Сравним график ∫idt и график заряда, накопленного конденсатором q = U/C.
Распечатаем соответствующие графики. Раскрасим их разным цветом. Сделаем выводы.
(Замечание. Датчик тока рассчитан на 250 мА. Если ток зарядки был менее 1-3 мА (а это
примерно так в опыте с R=1,5 кОм и С = 500 мкФ, конечно, измерения совсем неточны.
Поэтому для этого опыта рекомендуется взять сопротивление около 100 Ом (тогда ток
зарядки может быть измерен более точно). Но и в таком случае на NOVA-5000 мастер
анализа может неверно выполнить интегрирование- не пугайтесь!)
9. Оформим работу, запишем выводы, обсудим результаты с учителем, уберем
оборудование.
Тарчевский А.Е., шк. 179 МИОО, март 2011
56
ЛАБОРАТОРНАЯ РАБОТА 13
ИЗМЕРЕНИЕ ЁМКОСТИ ГАЛЬВАНИЧЕСКОГО ЭЛЕМЕНТА (ИЛИ
АККУМУЛЯТОРА)
ОБОРУДОВАНИЕ: NOVA, исследуемая батарейка, датчик тока 2,5А, реостат 6 Ом,
мультиметр или вольтметр на 1,5 В.
Можно вторым комплектом оборудования одновременно исследовать второй источник тока.
Как известно, источник тока характеризуется не "зайцами, которые работают в 10 раз дольше
обычных", а емкостью, измеряемой в ампер-часах. (Это время (в часах), в течение которого источник
может давать ток 1Ампер. Фактически это заряд, который может "перекачать" источник тока.)
ЦЕЛИ РАБОТЫ:
Изучить, как источник тока отдаёт энергию цепи, измерить ёмкость батареи [А*ч].
Измерить емкость источника тока.
Узнать, изменилась ли ЭДС источника тока после его разрядки.
ПРЕДВАРИТЕЛЬНЫЕ ТРЕБОВАНИЯ:
Тема "Закон Ома для цепи, содержащей ЭДС".
ХОД РАБОТЫ:
Внимание !
Типичная ошибка – нечаянно замкнули источник тока (подключили источник "крокодилами", а они
замкнулись), а если вынимали элементы из "квадратной" батареи - замкнули их корпусами
(Нельзя!!!)
Типичная ошибка – плохие контакты или неисправный реостат приведут к скачкам тока во время
опыта.
Помните, что батарейка довольно быстро "садится" от большого тока.
1. Возьмём новый гальванический элемент или заряженный аккумулятор.
Измерим вольтметром ЭДС нового источника тока и запишем её значение.
2. Соберём цепь с исследуемым источником тока, датчиком тока на 2,5 А,
(не перепутайте его с датчиком на 250 мА - сгорит!!!), ключом и реостатом (всё соединяем
последовательно), покажем ее учителю перед включением.
Внимание ! Типичная ошибка – плохие контакты в цепи. На графике видны колебания тока.
3. Установим в NOVA частоту 1 измерение в секунду и длительность - бесконечно.
Не забудем подключить NOVA к подзарядке.
Поскольку эксперимент займёт более часа, займемся чем-нибудь полезным. Рекомендуется
исследовать такой же установкой другой источник тока и сравнить их.
Позаботимся, чтобы никто не трогал установку во время эксперимента.
4 .С помощью реостата установим ток разряда (для элемента 1,5 В около 1- 1,5 ампера) и
запустим измерения. Когда ток упадет до определенного уровня – например, до 0,5 ампера
(обсудите этот вопрос с учителем), закончите опыт. Действительно, батарейка еще работает,
а низкий ток уже не позволяет использовать ее в приборах.
Внимание! Аккумулятор НЕЛЬЗЯ полностью разряжать - он выйдет из строя! Будьте
внимательны, отслеживайте ток, как только ток начнёт явно резко падать – пора заканчивать
опыт.
57
5. Разберём цепь, отключим источник тока. Снова измерим его ЭДС вольтметром, запишем
результат и сравним с начальным значением.
6. У нас есть график зависимости I(t) . Проинтегрируем его (Анализ→Интеграл).
Получится график зависимости перекачанного заряда от времени в [А*с].
Значение этой функции в наивысшей точке - это и есть весь прошедший через источник тока
заряд. Переведём результат в [А*ч] и запишем его.
Сохраним файл.
Распечатаем графики I(t) , q(t).
7. Выбросим использованный гальванический элемент. (Аккумулятор выбрасывать не надо!)
Положим оборудование на место.
Внимание! Типичная ошибка - путать старые и неиспользованные источники. НЕ пометить
оставшиеся после разборки батареи элементы, как "новые", не выбросить использованный
гальванический элемент после работы.
8. Проанализируем зависимость I(t), q(t). Сделаем выводы. Составим краткий отчет о
работе и сдадим его.
9. Кстати:
В сети Интернет можно найти множество интересных данных об испытаниях различных
источников тока. Можно при желании исследовать достаточное количество различных
батареек и написать статью о них. (Цена 1 ампер-часа на рынке батареек.)
Не забудьте, что емкость батареек значительно зависит от тока разряда!
А.Е. Тарчевский, шк. 179 МИОО, 2009
58
ЛАБОРАТОРНАЯ РАБОТА 14
ИЗУЧЕНИЕ ЗАВИСИМОСТИ ВНУТРЕННЕГО
СОПРОТИВЛЕНИЯ ИСТОЧНИКА ТОКА ОТ ТОКА В ЦЕПИ
ПРЕДВАРИТЕЛЬНЫЕ ТРЕБОВАНИЯ: Знание темы: Источник тока, ЭДС, короткое
замыкание, закон Ома для полной цепи.
Школьные учебники предполагают, что источник тока имеет некоторое постоянное
внутреннее сопротивление, однако здесь кроется страшный обман ...
В работе предлагается исследовать этот вопрос.
ОБОРУДОВАНИЕ
Исследуемый источник тока например, гальванический элемент на 1,5 В (не старый!) или
"мизинчиковый" аккумулятор, реостат на 6-10 Ом, провода, ключ, цифровой мультиметр,
компьютер "NOVA", датчик напряжения, датчики тока 2,5А (может понадобиться два
одинаковых датчика тока, если ток короткого замыкания превышает 2,5А)
Хороший вариант - батарея типа "Крона" на 9В, реостат на 30-50 Ом. В этом случае ток не
превысит 1 - 1,2А.
ПОДГОТОВКА:
1. Возьмем гальванический элемент на 1,5 вольта, (например, пальчиковый).
Убедимся, что элемент новый. Проверим годность источника тока амперметром (можно
взять мультиметр на пределе измерений, например, 20А) ток при кратковременном (!) –
(одна- две секунды) коротком замыкании должен быть 2- 5 А.
- Запишем ток короткого замыкания.
Если ток короткого замыкания превышает 2,5 А, в следующих исследованиях целесообразно
подключить два датчика тока параллельно, таким образом можем измерять токи до 5А.
2. - Измерим и запишем ЭДС источника тока.
Для этого надо просто подключить вольтметр к источнику без нагрузки.
3. Соберем цепь из источника тока, ключа,
датчиков тока и напряжения, реостата.
Внимание!
Большой
ток
быстро
разряжает батарейку! Нельзя держать
ключ замкнутым дольше нескольких
секунд, которые необходимы для снятия
показаний приборов.
ВНИМАНИЕ! У реостата обычно три вывода два крайних и средний. Если подключить
провода
к
двум
крайним
выводам,
сопротивление
будет
невозможно
регулировать.
ВНИМАНИЕ! не перепутайте датчик на 2,5 А с
датчиком на 250мА, а то датчик сгорит!
R
‫ع‬,r
V
rA
A
NOVA
Покажите собранную
цепь учителю,
расскажите, что собираетесь делать
дальше.
ТЕОРИЯ:
59
4. В цепи (см. рисунок) датчик напряжения покажет напряжение U на сопротивлении R+ rA.
Зная ЭДС источника ‫ع‬, ток в цепи I и напряжение U, легко найти внутреннее сопротивление
r источника тока.
Нарисуйте схему. Выведите самостоятельно формулу для r и покажите вывод формулы
учителю.
Должно получиться : r = ( ‫ ع‬- U)/I
ЭКСПЕРИМЕНТ:
5. Установим частоту 25 измерений в секунду для пробного опыта. Если график получается
"рваным", можно установить частоту, например, 100 измерений в секунду. (При такой
частоте на NOVA 5000 график выведется на экран только после остановки опыта).
Установим максимальное сопротивление в цепи.
Замкнём цепь.
По возможности равномерно изменим сопротивление от максимума до нуля, затем опять до
максимума и так несколько раз. Рекомендуется сделать весь опыт секунд за десятьпятнадцать. Разомкнём цепь.
Сохраним результаты опыта.
ОБРАБОТКА РЕЗУЛЬТАТОВ:
6. Построим на компьютере график зависимости I от времени и график r = ( ‫ ع‬- U)/I от I
Рекомендации по построению графика. (Коломеец Иван, 11Б, 2009)
а) Нахождение зависимости тока от времени.
Если использовалось два датчика тока одновременно, сложим их показания. "Мастер анализа" –
"Операция"- "Сложение", в качестве аргумента выберем I1 , I2
б) Нахождение зависимости ‫ ع‬- U от времени.
"Мастер анализа" – "Операция"- "Линейная" - в качестве аргумента выберем U, коэффициент при U
возьмём -1, а свободный член зададим равным ‫ ع‬.
в) Нахождение зависимости r = ( ‫ ع‬- U)/I от времени.
"Мастер анализа" – "Операция"- "Деление"- первый аргумент- график, полученный в б), второй аргументграфик из а) или ток.
г) Нахождение r = ( ‫ ع‬- U)/I от тока в цепи.
К сожалению, старая NOVA упорно давала ошибку при попытке сделать это действие, поэтому в качестве
аргумента придётся выбрать r, а в качестве функции I. Хотелось бы сделать наоборот, но это получается не
в каждой версии программного обеспечения. (Попробуйте, конечно, сделать правильный график.)
"Свойства графика"- в качестве оси x выберем ( ‫ ع‬- U)/I из в)
д) Выведем I из а), получим требующийся график, правда, с "перепутанными" осями.
Сохраним результаты обработки данных.
7. Подпишем график, позаботимся, чтобы на осях были понятные обозначения и легко
читаемые числа. Не забудем записать тип исследуемого источника тока. Отметим на
графике начало и конец опыта.
8. Сравним графики для уменьшающегося сопротивления нагрузки и для увеличивающегося
Есть ли существенная разница между ними? Что происходит? Может быть, нагревание
батарейки влияет на внутреннее сопротивление?
9. Запишем выводы. Какова зависимость внутреннего сопротивления источника тока от
тока в цепи?
10 ДОПОЛНИТЕЛЬНО:
а) Хорошо бы дать батарейке остыть несколько минут, затем повторить опыт и
посмотреть, стабильны ли полученные зависимости, влияет ли разрядка батарейки на
результаты.
А.Е. Тарчевский, шк. 179 МИОО, 10 апреля 2011
60
ЛАБОРАТОРНАЯ РАБОТА 15
ИЗМЕРЕНИЕ ИНДУКЦИИ МАГНИТНОГО ПОЛЯ ЗЕМЛИ
ОБОРУДОВАНИЕ компас, чистый лист бумаги, скотч, датчик индукции магнитного поля,
NOVA-5000, секундомер или часы, транспортир, угольник, отвес.
ПРЕДВАРИТЕЛЬНЫЕ ТРЕБОВАНИЯ: знание основ тригонометрии, векторы, проекции
вектора на ось, понятие о магнитном поле.
ЦЕЛЬ РАБОТЫ
Определить направление и модуль вектора индукции магнитного поля Земли.
Понять, что мы находимся в поле и приобрести навык измерения его.
ВСПОМОГАТЕЛЬНЫЙ ОПЫТ
Чувствительный элемент датчика находится в его конце и помечен тонкой линией на
корпусе.
На измеряемое магнитное поле могут значительно повлиять железные детали стола,
узлы крепления, шурупы. Снимем с себя и уберём далеко часы, компас и другие железные
вещи, которые могут влиять на магнитное поле, если находятся близко к датчику.
Что такое "близко?" Неплохо бы проверить. Закрепим датчик неподвижно (можно
приклеить скотчем к столу). Установим высокую чувствительность переключателем на
корпусе датчика. Частота измерений -10 в секунду, продолжительность- бесконечно. Будем
приближать издалека к датчику часы или другой маленький железный предмет. Оценим
расстояние, начиная с которого присутствие предмета изменяет показания датчика.
Проверим то же самое для компаса, для массивного предмета, например, для
пятикилограммовой железной гири. Проверим влияние включенного блока питания для NOVA
на показания датчика. Запишем полученные расстояния.
ИДЕИ РАБОТЫ
Датчик показывает значение проекции вектора магнитной индукции (в той точке, где
находится чувствительный элемент датчика) на ось датчика. Вращая датчик в
горизонтальной плоскости, получим график зависимости проекции вектора Вгор на каждое
направление.
Найдем направление, где проекция Вгор наибольшая. (Оно должно бы совпасть с
направлениями на север и на юг.)
Затем установим в вертикальной плоскости лист из немагнитного материала,
(например, большой картонный ящик) так, чтобы направление север-юг лежало в этой
плоскости. Вращая датчик параллельно установленной плоскости, проведем аналогичные
действия, в результате получим максимальное значение В и определим угол наклона вектора
индукции по отношению к горизонтали.
Зная максимальные значения Вгор и В, найдем угол наклона вектора магнитной
индукции к горизонту. (Это легко, нарисуйте соответствующий треугольник – вектор В и его
проекции. То же самое можно, но менее удобно, получить прямым измерением угла наклона
оси датчика к горизонту в тот момент, когда показания датчика, вращаемого в вертикальной
плоскости, достигнут максимального значения.)
УСТАНОВКА
Найдём горизонтальную поверхность, располагающуюся вдалеке от железа, магнитов,
компасов, блоков питания, проводов, стен, пола, штативов. Неплохо делать опыт на лежащей
61
между двух столов широкой доске или фанере. При этом стальные элементы конструкций
столов будут находиться далеко от датчика.
НЕ забудьте, что датчик после вращения в горизонтальной плоскости придется
вращать в вертикальной, при этом чувствительный элемент (конец датчика с чёрточкой)
должен находиться в одном и том же месте.
МАЛЕНЬКАЯ ХИТРОСТЬ
Зависимость Вгор получится от времени, а нам хочется – от азимута. (Азимут-угол
между направлением на север и выбранным направлением, отсчитывается от направления на
север по часовой стрелке, например, востоку соответствует 90о, а западу- 210о).
Поэтому на приклеенном скотчем к столу листе отметим центр вращения и
направления на север и юг (по компасу) и аккуратно разметим лист углами по 12 о, в ходе
опыта будем стараться равномерно поворачивать датчик так, чтобы угол 12о проходился за 2
секунды, тогда на 360 градусов повернем ровно за минуту. Продолжительность опыта тоже
установим одну минуту - тогда вид графика зависимости от времени и от угла будет почти
одинаков. Можно сделать один аккуратно размеченный лист и оставить его тем, кто будет
делать эту работу после вас.
Если в комплекте лаборатории есть датчик угла поворота (скорее всего, в единственном экземпляре), можно
подсоединить его (немагнитными материалами) к центру вращения. Датчик поворота должен быть достаточно удалён от
датчика магнитного поля.
ХОД РАБОТЫ
1) Подготовим место для опыта, приклеим размеченный лист скотчем к столу.
2) Установим высокую чувствительность. Запустим опыт и начнём вращать датчик в
горизонтальной плоскости, контролируя поворот на каждые 6 градусов за секунду. Один
человек вращает, другой - диктует время каждые две секунды. Потренируйтесь, прежде чем
запустить измерения.
Не забудем точно определить положение точки, вокруг которой вращали датчик- оно
необходимо для следующего опыта.
Внимание: Типичная ошибка вращать датчик вокруг неверной точки, например,
середины или места, где выходят провода.
Почему важно вращать датчик вокруг точки, где находится чувствительный элемент?
3)Получим график зависимости Вгор(t)и распечатаем его. (Внимание! Позаботьтесь о том,
чтобы на оси ординат выведенного графика были читаемые цифры. По- видимому, чтобы не
терять точность, надо указать перед опытом число выводимых знаков.)
Подпишем ручкой на оси времени соответствующие значения азимута. Отметим на
графике точки, где модуль значения Вгор наибольший (таких точек должно быть две, одна когда датчик "смотрит" на север, другая- когда на юг).
Почему именно в этих положениях датчика получились наибольшие по модулю значения?
4) Найдём горизонтальную составляющую магнитной индукции поля: Сумму модулей Вгор в
двух точках, где значение максимально (соответствующих направлению на север и юг),
поделите на 2.
Заодно сравним, одинаковы ли по модулю эти значения.
Запишем Вгор макс=
5) Теперь, при помощи большого угольника установим вертикальную поверхность так, чтобы
направление север-юг (точнее то, где Вгор максимально), лежало в этой плоскости.
Убедимся, что чувствительный элемент датчика находится в той же точке, что и в первом
опыте.
62
Проведем эксперимент аналогичным образом, с поворотом датчика на 180о (под стол датчик
опускать неудобно) за 30 секунд, приклеив лист с разделением на 12-градусные углы к
вертикальной плоскости .
Распечатаем этот график.
Определим и запишем модуль вектора В (максимальное значение на полученном графике).
6) Найдём угол, который образует вектор В с горизонтом. Во- первых, его можно увидеть
на графике. а более точно угол можно рассчитать как арксинус отношения модуля
индукции магнитного поля к горизонтальной составляющей. (Нарисуйте картинку,
объясните почему!)
7) ПРОСТОЙ ВАРИАНТ ОПЫТА
К тому месту датчика, где выходит провод, прикрепим отвес. Расположим датчик
горизонтально на весу, при этом конец датчика надо прислонить к какому-нибудь
выступающему немагнитному предмету, например, к торцу вертикально стоящей прочно
закреплённой доски. Зафиксируем конец датчика в определенной точке и не будем позволять
ему сдвигаться (датчик может только вращаться).
Конечно, хорошо бы сделать устройство, позволяющее при этом не держать датчик в руках. Можно попробовать
приспособить для этого пластмассовые детали конструктора.
Алгоритм действий:
Медленно поворачиваем датчик в горизонтальной плоскости (вокруг вертикальной оси).
Если при выбранном направлении вращения показания увеличиваются - продолжаем,
уменьшаются - вращаем в другую сторону. Продолжаем вращать датчик, пока не будет
достигнут максимум показаний. Как только "проскочим" максимум и показания начнут
уменьшаться, возвратим датчик обратно в положение "максимума". Теперь сделаем то же
самое, вращая датчик в вертикальной плоскости. Затем, уже с меньшими углами поворота снова вокруг вертикальной оси, затем вокруг горизонтальной. В конце концов, остановимся в
том положении, где проекция поля на ось датчика наибольшая. Тут-то и измерим угол между
датчиком и вертикалью по отвесу, а также узнаем значение В.
Запишите результат, полученный таким способом. Какой способ, на ваш взгляд, лучше?
Какой способ быстрее, какой нагляднее?
8) Нарисуем Землю и линии индукции магнитного поля Земли, объясните, почему угол их
наклона на широте вашей школы таков.
9) Если бы опыт проводился в южном полушарии, что изменилось бы?
10) Дополнительное задание. Известно, что, поскольку магнитное поле направлено явно не
горизонтально, железные предметы, постоянно находящиеся в одном вертикальном
положении (ножки столов, отопительные батареи, и др.) намагничиваются. Проверьте
правильность этого утверждения, измерив индукцию поля вверху и внизу такого предмета
каждый раз направляя датчик строго горизонтально на север. Проверьте для нескольких
предметов. Кратко напишите, что получилось. (То же самое получается и с компасом)
11) Дополнительное задание. Исследуйте поле разных местах класса. Датчик ориентируйте
все время одинаково на север. Что получилось? Есть ли места с повышенной индукцией
поля?
А.Е. Тарчевский, шк. 179 МИОО, апрель 2010,
при участии учащихся 10 Б класса Ивана Коломейца и Ильи Сухорукова
63
ЛАБОРАТОРНАЯ РАБОТА 17
ИЗУЧЕНИЕ ТЕМПЕРАТУРЫ И ЯРКОСТИ СВЕЧЕНИЯ
НИТИ НАКАЛА ЛАМПЫ
ЦЕЛИ РАБОТЫ:
1. Получить вольт - амперную характеристику лампочки накаливания.
2. Получить зависимость температуры нити накаливания от потребляемой лампочкой
мощности.
3. Получить зависимость освещённости от потребляемой лампочкой мощности.
4. Исследовать, при какой температуре нити накаливания лампочка перегорит.
ОБОРУДОВАНИЕ:
- несколько одинаковых лампочек накаливания.
(Хорошо использовать лампочки от старой ёлочной гирлянды), напряжение питания лампочек- не выше 12 В!
УЧИТЕЛЮ:
В том случае, если будет исследоваться лампочка, питающаяся от сети 220В, помните, что датчик тока НЕ развязывает цепи и сеть,
следовательно, все проводники, а также компьютер оказываются подключенными к сети! Это очень опасно! Если же вы все же делаете
такое исследование под руководством учителя, не забудьте обязательно ОТКЛЮЧИТЬ компьютер от сетевого блока питания, иначе
компьютер и датчики выйдут из строя! Одновременное непродуманное использование при работе с высокими напряжениями датчика тока и
напряжения может также привести к пробою датчиков!
- батарея или несколько последовательно соединённых батареек, при этом напряжение
должно быть достаточно (с запасом), чтобы сжечь исследуемую лампочку.
(Удобно взять регулируемый (плавно) блок питания, но с ним обычно бывают проблемы у "Архимеда")
- реостаты, например, на 500 и 50 Ом, если работаем с нерегулируемым блоком питания,
- провода, ключ, мультиметр,
- NOVA - 5000,
- датчик тока, напряжения, освещённости (2 шт.),
- коробка или труба, в которой располагаются лампочка и датчик освещённости,
обеспечивающая неподвижность их положения и не допускающая доступа света извне.
ИССЛЕДУЕМАЯ
ЛАМПОЧКА
NOVA-5000
R
ТРУБА
Источник
питания

Датчики
тока, напряжения,
освещенности




L
АI
U
P
И С
R – реостат
А – датчик тока
U – датчик напряжения
L –датчик освещённости
ИС – исследуемый источник света
64
Выясните у учителя, можно ли в ходе работы довести лампочку до перегорания и спросите,
какое максимальное напряжение разрешается подавать на эту лампочку, чтобы она не
перегорела.
ПОДГОТОВКА
ПОЛУЧЕНИЕ ЗАВИСИМОСТИ СОПРОТИВЛЕНИЯ ИССЛЕДУЕМОЙ ЛАМПОЧКИ ОТ
ТЕМПЕРАТУРЫ НИТИ НАКАЛИВАНИЯ (По данным справочника и измеренному
сопротивлению лампочки при комнатной температуре).
(Девятиклассникам: Т- абсолютная температура, измеряется в Кельвинах (К), a t- температура, измеренная в градусах
Цельсия. Т = t + 273 )
В Справочнике "Физические величины" приведена зависимость сопротивления
отрезка вольфрамовой проволоки длиной 1 см и диаметром 1 см от температуры. Ниже
приведён фрагмент таблицы 21.27:
T, К R*10-6,Ом
273 6,37
300 7,20
500 13,45
Т, К
293
400
600
R*10-6, Ом
6,99
10,26
16,58
700 20,49
900 27,94
1100 35,58
800
1000
1200
24,19
31,74
39,46
1300 43,40
1500 51,40
1700 59,58
1400
1600
1800
47,37
55,46
63,74
1900 67,94
2100 76,49
2300 85,22
2000
2200
2400
72,19
80,83
85,65
2500 94,13
2700 103,22
2900 112,51
2600
2800
3000
98,66
107,85
117,24
3100 121,95
3300 131,60
3500 141,42
3200
3400
3600
126,76
136,49
146,40
3655 149,15
Температура плавления вольфрама при нормальном давлении 3420 °С = 3693 К (из таблицы
12.1 того же справочника.)
В лампочке нить накаливания тоже вольфрамовая, но другого размера, поэтому данные для
лампочки будут пропорциональны справочным данным.
1. Измерьте сопротивление лампочки "при комнатной температуре" мультиметром и
запишите его значение.
(Замечание: При этом придётся пропустить ток через лампочку, и нить накаливания нагреется. Поэтому ток должен быть
минимально возможным. Для проверки влияния тока мы брали самый чувствительный из имеющихся микроамперметров, и,
подобрав очень малое напряжение, проводили измерение сопротивления, включив цепь на короткое время. Измерения
сопротивления лампочки при токах 30 и 90 мкА, а также измерение сопротивления мультиметром показали хорошее
совпадение результатов, поэтому можно считать, что измеренное сопротивление соответствует "комнатной" температуре.
Так что можно пользоваться мультиметром. Желающие могут повторить наш опыт.
ВНИМАНИЕ! Микроамперметр выйдет из строя при большом токе! Если есть сомнения, включите сначала
малочувствительный миллиамперметр, (или мультиметр на соответствующем пределе измерений), чтобы определить
примерное значение тока. Амперметры включают последовательно и никогда не подключают непосредственно к источнику
65
тока! Прежде чем подключать микроамперметр, покажите учителю, как и куда вы собираетесь его включить! (До
включения, а не после того, как он перегорит!)
2. Постройте зависимость сопротивления ВАШЕЙ ЛАМПОЧКИ от температуры на
миллиметровке в хорошем масштабе. Предположим, что измеренное сопротивление R
соответствует 300К. Надо умножить данные таблицы на R/(7,20*10-6), затем построить
нужный график.
(Добрый совет: там, где это уместно, берите данные таблицы через одно.)
3. Известно, что сопротивление чистых металлов можно считать с неплохой точностью
пропорциональным абсолютной температуре [1], а коэффициент пропорциональности примерно
составляет 1/273 град-1, т.е. R(t) = R0(l+t/273) или, то же самое, R(T) = R0(T/273), где R0 сопротивление при нуле градусов Цельсия. Проверьте по полученному графику, является ли
такое линейное приближение (график проходит через точку (0,0)) достаточно точным для
рассматриваемой задачи, где температура будет подниматься от комнатной до 3500К.
Запишите вывод. Покажите построенный график учителю.
ЭКСПЕРИМЕНТАЛЬНАЯ УСТАНОВКА
4. В опыте потребуется плавное непрерывное изменение тока, проходящего через лампочку,
поэтому блок питания, изменяющий напряжение "скачками" не подходит. ( Более того,
подключение нескольких датчиков напряжения с NOVA-5000 к исправному стабилизированному блоку питания вместо
постоянного напряжения показывало колебания с падением напряжения до нуля, с периодом в 2-3 секунды, да ещё и со
сдвигом по фазе для каждого датчика! И компьютер меняли и блок питания - результат тот же. При этом от батареек всё
работало нормально.)
Неплохое решение - использовать реостаты, включенные последовательно с лампочкой –
например, один - на 500Ом, а последовательно ему - на 50 Ом, что позволит провести
измерения сначала на малых токах (плавно изменим при этом сопротивление реостата от
500Ом до 0), затем – более точно, на больших токах (плавно изменим сопротивление второго
реостата от 50 Ом до разрешенного напряжения или до перегорания лампочки).
Подключите датчики тока и напряжения, установите блоком питания или реостатами
минимальное напряжение, попросите учителя проверить схему перед включением.
ПРОБНЫЕ ОПЫТЫ
5. Убедитесь, что яркость свечения лампочки изменяется от нуля до яркого свечения.
(ВНИМАНИЕ! НЕ СОЖГИТЕ ЛАМПОЧКУ!)
ВНИМАНИЕ:
Если в цепи плохие контакты, опыт не получится!
6. Установите частоту 10 измерений в секунду.
ЗАМЕЧАНИЕ:
На графиках, которые мы будем строить, должна быть важная точка - соответствующая нулю тока и
нулю напряжения на лампочке. Чтобы не потерять её, сначала включите регистрацию данных, а сразу
после этого – ток в цепи (разумеется, минимально возможный).
Медленно увеличивайте напряжение до яркого свечения.
Убедитесь, что датчики тока и напряжения не "зашкаливают".
Убедитесь, что установка работает нормально.
ОПЫТ
7. Выполните опыт "начисто". Получите график зависимости тока и напряжения от времени.
Получите график зависимости тока от напряжения на лампочке - он называется вольт амперной характеристикой. Распечатайте вольт- амперную характеристику лампочки. Не
забудьте про "хороший" масштаб. График надо подписать.
Не забудьте обеспечить, чтобы на графике была точка, где ток и напряжение равны нулю.
66
Неплохо построить график, соответствующий увеличению тока (до яркого свечения
лампочки), а затем, не прерывая записи - в сторону уменьшения тока. Если всё хорошо,
графики должны "наложиться" друг на друга. А если нет, попробуйте объяснить результат.
Объясните, почему график не линеен, ведь закон Ома говорит, что I=U/R.
Сохраните экспериментальные данные.
ПРОБНЫЙ ОПЫТ
8. Подберите чувствительность и расположение датчика освещённости так, чтобы он не
"зашкаливал" при максимальной яркости. В то же время, он не должен показывать "слишком
мало". Просто подвигайте датчик, посмотрите, при каком расстоянии показания выйдут на
"полочку".
Установите лампочку в трубу. Убедитесь, что обеспечена неподвижность лампочки и
датчика. Убедитесь, что датчик освещённости показывает ноль, когда лампочка не светится
(проверьте!) и показывает что-то разумное (и не выходит на ограничение!), когда
лампочка светится ярко. Потребуется подобрать взаимное расположение датчика и
лампочки. Рекомендуется установить в коробке два датчика освещённости с разной
чувствительностью, тогда можно точно "отследить" и очень слабое и сильное свечение
лампочки.
Внимание:
Будет обидно, если при перегорании лампочки датчик освещённости выйдет на предел измеренийпридётся переделывать опыт.
Должна быть обеспечена неподвижность лампочки и датчиков во время опыта!
ОПЫТ
9. Установите запись данных с частотой 25 измерений в секунду.
Включите запись данных, затем включите ток.
постепенно (вначале - быстрее, к концу опыта - медленно) и всегда-плавно увеличивайте
ток в цепи, чтобы достичь максимального свечения секунд за 50.
Если учитель разрешил сжечь лампочку - сделайте это в только конце работы (в том случае,
если вы уверены, что всё получится и опыт не придётся переделывать).
Внимание!
Если лампочка перегорела, а опыт надо переделать – придётся повторно выполнить пункты 1 и 2!
Сохраните экспериментальные данные.
ОБРАБОКА РЕЗУЛЬТАТОВ
10. Если лампочку довели до перегорания, вероятно, получилась более качественная
вольт-амперная характеристика, чем в опыте из пункта 7. Если это так, распечатайте
и этот график.
11. Выведите зависимость тока от напряжения на экран и при помощи курсора
снимите данные (например,15-20 точек, в "интересных" местах точки надо брать
чаще), нужные для построения хороших графиков. Заполните этими данными таблицу со
столбцами:
Ток I - измеренное значение, мА,
Напряжение U - измеренное значение, В,
Сопротивление лампочки R = I/U (расчёт), Ом,
Потребляемая мощность P= IU, (расчёт), Вт
Температура нити накаливания Т, К (Зная сопротивление, легко определить значение
температуры по графику из пункта 2.)
67
12. Постройте на миллиметровке график зависимости температуры нити накала от
потребляемой мощности. Отметьте на графике температуру, при которой перегорела
лампочка и температуру плавления вольфрама. Сделайте выводы.
13. Постройте при помощи компьютера график зависимости освещённости от
потребляемой мощности. Для этого надо построить график мощности, умножив (при
помощи мастера анализа) I на U, затем выбрать IU в качестве аргумента, а
освещенность - в качестве функции. (Освещённость - что нам нужно от лампочки, а
мощность- то, за что мы платим деньги. Сделайте вывод.
14. Постройте при помощи компьютера график зависимости освещённости от
напряжения на лампочке. При каком напряжении надо использовать эту лампочку?
15. ЗАМЕЧАНИЕ.
Согласно теории (закон Стефана-Больцмана), мощность излучения нагретым
телом
пропорциональна разности четвёртых степеней температуры тела и температуры
окружающей среды. При высокой температуре нити накала можно пренебречь обратным
потоком излучения от окружающей среды к лампочке, т.е. освещённость должна быть
пропорциональна четвёртой степени абсолютной температуры нити накала. Но наш датчик
воспринимает не всё излучение, а только видимую его часть (при этом большая часть излучения
лампочки находится в инфракрасном диапазоне), поэтому проверить этот закон не получится 
16. Запишите выводы по работе.
17. ДОПОЛНИТЕЛЬНО:
Можно аккуратно разбить стекло у лампочки и исследовать, при какой температуре она
перегорит на воздухе.
В работе использованы идеи из книги:
[1] Варламов С. Д., Зильберман А. Р., Зинковский В. И.
Экспериментальные задачи на уроках физики и физических олимпиадах. — М.: МЦНМО, 2008.
А.Е. Тарчевский, шк. 179 МИОО, 10 апреля 2011
68
ЛАБОРАТОРНАЯ РАБОТА 18
ИЗМЕРЕНИЕ МАГНИТНОГО ПОЛЯ СОЛЕНОИДА
пробная
ОБОРУДОВАНИЕ:
Соленоид (его можно изготовить самостоятельно, намотав медную изолированную
проволоку на кусок пластиковой трубы), батарейки, ключ, реостат 10-20 Ом, провода, датчик
магнитного поля, штатив с алюминиевой лапкой, NOVA, изолента, скотч, две линейки 40-50
см, датчик тока 2,5 А.
Внимание! Работа с блоком питания не получается - "глючит" программа Multilab, даже на обычном
компьютере. Пользуйтесь батарейками.
ЦЕЛЬ РАБОТЫ:
Измерить величину магнитного поля соленоида в различных точках на его оси.
Посмотреть, как зависит магнитное поле от тока в цепи.
Оценить, однородно ли поле в центре соленоида (зависит ли от расстояния от оси)
УСТАНОВКА:
ДАТЧИК МАГНИТНОГО
ПОЛЯ РАСПОЛОЖЕН
ПО ОСИ СОЛЕНОИДА
ШТАТИВ ПЕРЕМЕЩАЕМ
ВДОЛЬ ПРКЛЕЕННОЙ
СКОТЧЕМ ЛИНЕЙКИ
ЗАКРЕПЛЯТЬ
АККУРАТНО!
СОЛЕНОИД
метка или кусочек
изоленты для точного
определения расстояния
Соберите установку. Обеспечьте отсутствие железных предметов рядом с катушкой. Будьте
осторожны при закреплении датчика, не раздавите его при затягивании лапки! Рекомендуется
обмотать закрепляемый конец датчика изолентой, чтобы не поцарапать его. Чувствительный
элемент датчика находится на его конце и помечен тонкой линией на корпусе. В начале
опыта это место должно находиться строго в центре соленоида. Затем будем постепенно
выдвигать датчик, измеряя значение индукции магнитного поля. Линейка не должна
болтаться, штатив должен легко двигаться вдоль неё, недопустимы перекосы (проверьте),
датчик должен быть расположен строго на оси соленоида во время всего опыта.
69
ТЕОРИЯ:
Солено́ид — разновидность электромагнитов.
Характеризуется значительным соотношением длины
намотки к диаметру, что позволяет создать внутри
катушки относительно равномерное магнитное поле.
Магнитная индукция внутри соленоида:
,
n = N / s — число витков на единицу длины,
I — ток в обмотке.
Магнитная постоянная — константа
Н/А²
ПОЛЕ СОЛЕНОИДА
ХОД РАБОТЫ
Соберите цепь из батареи на 4,5 В, ключа, соленоида, реостата, амперметра. Включайте ток
только на короткое время, необходимое для проведения измерения, затем выключайте. Токи
большие, батарейка быстро "садится".
ОПЫТ 1 ЗАВИСИМОСТЬ ПОЛЯ В ЦЕНТРЕ СОЛЕНОИДА ОТ ТОКА В НЁМ
Расположите чувствительный элемент датчика строго в центре соленоида.
Включите регистрацию данных (частота- 10 замеров в секунду, время - непрерывно).
Регулируя реостатом ток в цепи, измеряйте ток и поле в центре катушки. Можно сначала
увеличить ток до максимума, затем уменьшить до минимума. Не забудьте выключить ток
после выполнения опыта.
Посмотрите на графики зависимости тока и поля от времени. Не "зашкаливает" ли датчик? А
если сделать его чувствительность выше? (на корпусе датчика есть переключатель).
Подходит ли датчик тока 2,5А для вашего опыта? (Есть ещё датчик с чувствительностью 250
мА).
Сделайте опыт "начисто" и постройте график зависимости индукции магнитного поля от
тока в катушке.
На этом же листе постройте график по результатам расчётов, можно - "вручную", (А
если вы - любитель компьютера, рассчитайте коэффициент, на который надо умножить I и
сделайте это в "Мастере анализа".
Запишите выводы.
ОПЫТ 2 ЗАВИСИМОСТЬ ПОЛЯ ОТ РАССТОЯНИЯ ОТ ОСИ В ЦЕНТРЕ СОЛЕНОИДА
Датчик находится в центре соленоида. Включим ток и не будем изменять его. Перемещая
датчик от оси соленоида (перпендикулярно оси!) к краю, посмотрим на графике, как
изменяется поле. Запишем вывод.
70
ОПЫТ 3 ИЗУЧЕНИЕ ПОЛЯ НА ОСИ СОЛЕНОИДА
Расположим чувствительный элемент датчика на оси в центре соленоида.
Будем постепенно выдвигать датчик из соленоида, измеряя значение магнитного поля.
Линейка не должна болтаться, штатив должен легко двигаться вдоль неё, недопустимы
перекосы (проверьте), датчик должен быть расположен строго на оси соленоида во время
всего опыта.
- Установим достаточный ток, чтобы датчик в центре соленоида показывал значение,
сравнимое с пределом его измерений (если это не так, при выходе датчика из соленоида он
будет показывать "слишком мало")
- Не забывайте выключать ток, когда не проводятся измерения.
Вариант 1:
- Установим режим снятия данных "по нажатию кнопки".
- Будем заполнять таблицу со столбцами: "расстояние от центра, см", "поле, Тл".
Рекомендуемый шаг снятия данных - 1см там, где поле меняется слабо, 0,5 см - где сильно.
- Построим на "миллиметровке" график зависимости магнитной индукции от расстояния,
расстояние отсчитываем от центра соленоида.
Вариант 2 :
Для любителей автоматизации.
Если вы хотите рискнуть, а учитель готов дать вам датчик расстояния, можно прикрепить к
штативу вертикальную "мишень", а сам датчик расстояния закрепить в другом штативе,
причём от него до мишени должно быть не менее 20 см. и в этот промежуток не должны
попадать посторонние предметы, а мишень должна быть достаточно большой (порядка 15
градусов, если смотреть от датчика), чтобы датчик её не терял. При работе с датчиком
расстояния необходимо подключить компьютер к блоку питания.
Установите частоту 10 измерений в секунду, включите ток, затем запустите измерения.
Медленно выдвигайте датчик из катушки, пока поле не снизится почти до нуля.
- Постройте нужный график (поле от расстояния) в программе Multilab. Не забудьте при этом
точно отметить (во время опыта и затем- на графике), какому расстоянию (по датчику
расстояния) соответствует центр обмотки, а какому - край (не трубы, а обмотки, разумеется).
Если вариант 2 не получается, придётся сделать работу по варианту 1.
Результаты опыта 3:
Покажите учителю график зависимости индукции магнитного поля на оси соленоида от
расстояния от центра. На графике обязательно должно быть чётко указано, где
находится центр соленоида и где край обмотки!
Запишите выводы.
А.Е. Тарчевский, шк. 179 МИОО, 11 апреля 2011
71
ЛАБОРАТОРНАЯ РАБОТА 19
ПРОВЕРКА ЗАКОНА ЭЛЕКТРОМАГНИТНОЙ ИНДУКЦИИ
ОБОРУДОВАНИЕ:
Сильный магнит, катушка- соленоид с железным сердечником, датчик напряжения, датчик
магнитного поля, штатив с двумя алюминиевыми лапками, NOVA.
Дополнительно – генератор звука, или источник переменного тока 50 Гц, ещё одна катушка,
ещё одна лапка штатива.
ЦЕЛЬ РАБОТЫ:
Проверить выполнение закона электромагнитной индукции: ‫ =ع‬- dФ/dt
УСТАНОВКА:
штатив
магнит
датчик
напряжения
катушка
датчик
магнитного
поля
72
Закрепим в лапке штатива катушку (ось вертикальна) так,
чтобы сверху к ней можно было подносить магнит, подключим датчик напряжения. (Магнит
должен быть достаточно сильным, а катушка - иметь довольно много витков и, желательно,
железный сердечник, чтобы возникающая ЭДС могла быть корректно измерена
малочувствительным датчиком напряжения (25 вольт).
Снизу (по оси катушки) расположим (закрепим в другой лапке на том же штативе)
датчик магнитного поля. Подберем опытным путём расстояние от низа катушки до датчика
так, чтобы показания датчика во время приближения магнита к верхней части катушки были
в разумных пределах и корректно отражались на графике.
Внимание :
типичная ошибка: Датчик магнитной индукции выходит за пределы измерений или наоборот,
работает около предела чувствительности и поэтому даёт большую погрешность.
Не надо допускать "зашкаливания" (при этом график выйдет на "полочку"). С другой стороны,
если поле слишком слабое, датчик ничего не покажет, или даст результат "скачками", с большими
погрешностями, что тоже плохо. Помните, что у датчика на корпусе есть переключатель
чувствительности. Также надо отрегулировать удаление датчика от катушки до начала основного
опыта.
ПОДГОТОВКА:
Установим частоту 10 измерений в секунду, чтобы график выводился на экран в реальном
масштабе времени. "Положим" магнит сверху на катушку. Перемещая вверх или вниз датчик
магнитного поля добьёмся, чтобы его показания были правильными, желательно близкими к
верхнему пределу измерения. Закрепим датчик в таком положении.
ОПЫТ 1
Установим достаточную частоту, например, 200 измерений в секунду, число измерений 500.
Будем быстро подносить и уносить магнит к верхней части катушки. (Вариант - вращать
магнит).
73
Построим график напряжения на катушке. Магнитный поток через катушку
пропорционален измеренному значению магнитной индукции В. Поэтому ЭДС в катушке
должна быть пропорциональна dВ/dt .
Построим график dВ/dt. Можно пользоваться иконкой "Производная"
в верхнем меню.
(Если график "шумный"- выполните сглаживание - надо выделить график курсором, нажать
иконку +, отмена - иконка – )
Выведем на печать оба графика в правильном масштабе. Сделаем выводы.
Дополнительно- ОПЫТ 2
Можно создать переменное магнитное поле в исследуемой
катушке при помощи другой катушки, (её расположим
сверху от катушки, на которой измеряем напряжение; обе
катушки и датчик расположены на одной вертикальной
оси). Закрепим верхнюю катушку в лапке штатива.
Подключим верхнюю катушку
к генератору звука
(выберите частоту пониже!) или к сети переменного тока с
частотой 50 Гц.
При этом позаботьтесь о правильной частоте измерений.
(Для 50 Гц рекомендую выбрать максимальную частоту
записи – около 3000 в секунду, число измерений – 100 ,
тогда продолжительность опыта составит приблизительно
0,03с, т.е. около полутора периодов колебаний), Разумеется,
надо включать запись данных после подачи переменного
тока на катушку, создающую поле.)
Внимание: Типичная ошибка- делать измерения слишком
долго. Колебания гармонические, достаточно записать
несколько периодов. Если запись получилась слишком
длинной- придётся вырезать из неё фрагмент разумной длины,
чтобы графики были наглядными.
Обработка данных эксперимента аналогична описанной в предыдущем пункте. Здесь можно
более тщательно сравнить графики.
ЗАМЕЧАНИЕ. Иногда в этой работе получается сдвиг фаз между графиками dВ/dt и напряжением, хотя его не
должно быть. Иногда сдвига фаз нет. Причины не выяснены. Возможно, сдвиг получается из-за разного
запаздывания датчиков, поэтому низкие частоты в опыте предпочтительны. Вполне вероятно, проблема связана
с компьютером NOVA-5000
А.Е. Тарчевский, шк. 179 МИОО, март 2011
74
ЛАБОРАТОРНАЯ РАБОТА № 20 (пробная)
ИЗУЧЕНИЕ СТОЯЧЕЙ ВОЛНЫ В ТРУБЕ
ОБОРУДОВАНИЕ:
Генератор звука (должен быть оборудован частотомером, также можно взять мультиметр,
"умеющий" определять частоту), динамик, микрофонный датчик, NOVA, прочная нитка,
картонная или пластмассовая труба длиной 1,5 метра, диаметром 7- 15 сантиметров, заглушка
для
свободного
конца
трубы,
рулетка
или
длинная
линейка,
нитка, удлиненный кабель (2,5 метра) для подключения микрофонного датчика.
Протянув датчик через трубу, получим зависимость
амплитуды колебаний от времени.
Проблема - в определении расстояния. Датчик расстояния
приспособить получается плохо, он теряет мишень. Мы
пользовались датчиком угла поворота (см. рисунок с
общим видом установки). При этом непросто собрать
установку правильно и аккуратно. Хорошим решением (и
это рекомендуется) может быть протягивание датчика на
нитке равномерно, например, с использованием моторчика
от LEGO (см рисунок).
ЦЕЛИ РАБОТЫ:
- Наблюдать резонанс в трубе при различных частотах звука,
- Определить скорость звука;
- Исследовать зависимость амплитуды колебаний давления в трубе от координаты (стоячую
волну);
- Проверить соответствие теоретических предсказаний резонансных частот
и
экспериментальных данных для трубы с обоими открытыми концами и с одним открытым
концом.
ТЕОРИЯ
Стоячие волны возникают вследствие интерференции волн, распространяющихся во взаимно
противоположных направлениях. Практически стоячие волны возникают при отражении волн
от преград и неоднородностей в результате наложения отражённой волны на прямую. Различные
участки стоячей волны колеблются с различными амплитудами (см. рисунок). Такие волны
возникают, например, в упругой системе - стержне или столбе воздуха, находящегося внутри
трубы, закрытой с одного конца, при колебаниях поршня в трубе. Бегущие волны
отражаются от границ системы, и в результате наложения падающих и отражённых волн в
системе устанавливаются стоячие волны. При этом по длине воздушного столба образуются узлы
смещений (скоростей) - плоскости, перпендикулярные к оси столба, на которых смещения
частиц воздуха отсутствуют, а амплитуды давлений максимальны, и пучности смещений плоскости, на которых смещения максимальны, а давления равны нулю. Узлы и пучности
смещений располагаются в трубе на расстояниях четверти длины волны друг от друга, причём у
твёрдой стенки всегда образуются узел смещений и пучность давлений. (Очевидно, у стенки
частицы не могут смещаться.)
Подобная же картина наблюдается, если убрать твёрдую стенку в конце трубы, но тогда
пучность скорости и узел давлений находятся приблизительно на плоскости отверстия. (Около
открытого конца трубы давление равно атмосферному.)
Распределение давлений и скоростей в стоячей волне при открытом и закрытом конце трубы
показано на рисунке. Амплитуды колебаний давления в разных точках вдоль трубы показаны
сплошной линией, амплитуды скорости - пунктирной.
75
ЗАКРЫТЫЙ КОНЕЦ ТРУБЫ
>
ОТКРЫТЫЙ КОНЕЦ ТРУБЫ
>
МОДА - вид собственных колебаний системы, при которой все параметры системы колеблются
с одной и той же частотой, а разница фаз колебаний выбранных точек остаётся неизменной во
времени. Обычно каждой моде соответствует определённая собственная частота.
ОРГАННАЯ ТРУБА (с одним открытым концом).
Стоячие волны в таком резонаторе возможны лишь для тех случаев, когда на длине трубы
укладывается нечетное число четвертей длин волн. (На открытом конце трубы будет узел
давления, на закрытом – пучность).
ОПРЕДЕЛЕНИЕ СКОРОСТИ ЗВУКА.
Определение скорости звука в газе в данной работе выполняется методом стоячих
звуковых волн. Стоячая волна в трубе с открытыми концами возникает, когда длина трубы L
равна целому числу полуволн: L = n λ/ 2 - открытая с двух концов труба,
где λ -длина волны звука, а п - любое целое число.
Подбор условий возникновения стоячей волны можно проводить двумя способами:
• При постоянной длине трубы L можно изменять частоту звуковых колебаний, наблюдая
стоячие волны. Если мы знаем порядок резонанса (изменяя частоту от 20 Герц (длина
волны при этом примерно λ = VT = V/ν = 330 / 20 (м) = 16,5 метров; очевидно, половина
длины волны много больше длины трубы. Повышая частоту, можем "поймать" первый
резонанс, L = 1 λ /2, λ = VT = V/ν, 2L= V/ν, V = 2Lν - рассчитаем скорость звука.
Если же порядок резонанса для более высоких частот неизвестен, можно воспользоваться таким
приёмом:
Плавно изменяя частоту звукового генератора, и, следовательно, длину волны, будем
наблюдать несколько последовательных резонансов.
В резонансе имеем: n λ n = 2L, учитывая λ п = VT = V/νn, для
резонансов порядка п и k получаем
2L νn= nV, 2L νk= kV Отсюда получаем:
V = 2L(νn - νk)/(n - k) где: L - длина трубы, νn - частота для n-ого узла. Зная частоты двух
последовательных резонансов (n - k = 1), рассчитаем скорость звука.
ДАТЧИКИ
Микрофонный датчик измеряет давление в момент времени, когда проводится
измерение. Очевидно, при частоте 200 Гц давление в данной точке совершает 200 колебаний в
секунду. Поскольку в работе используются меньшие частоты записи данных, например, 100 в
секунду, датчик покажет "странные" данные (нарисуйте высокочастотную синусоиду и снимите с
неё данные с низкой частотой). Нам интересна ОГИБАЮЩАЯ полученного графика (не та,
что рассчитана компьютером, а та, что видим "на глаз").
Внимание! Если громкость слишком велика, датчик будет всё время показывать
максимум. Подберите громкость. На слух – должно звучать заметно, но негромко.
76
Не забывайте вовремя отключать звук, когда вы не проводите измерения, звуки мешают
окружающим.
Подчеркнём, что датчик покажет ДАВЛЕНИЕ, соответственно на открытом конце
трубы оно близко к атмосферному. (Там узел давления.)
(Датчик уровня звука, который, как кажется, можно применить вместо микрофонного, показывает некий
"средний уровень" звука. Мы пробовали делать эту работу с датчиком уровня звука- не получилось.)
ХОД РАБОТЫ
1. Расположим трубу "лёжа" на
столе, обеспечим её неподвижность. На
расстоянии около двух диаметров
трубы
(можно
подобрать
это
расстояние, но важно обеспечить
"открытый" конец трубы) от торца на
оси трубы расположим динамик,
подключенный к генератору звука.
Сделаем
небольшую
на
слух
громкость.
Научимся
измерять частоту
колебаний.
(Большой
мультиметр
позволяет измерять частоту, его можно
подключить и использовать, если на генераторе нет хорошего указателя частоты).
2. Повышая частоту от минимальной (20 Гц), найдём на слух частоты, при которых
возникает резонанс. (Труба явно "загудит" на этих частотах. Можно также определить
резонанс, слегка касаясь трубы рукой.). Нетрудно предсказать минимальную частоту, при
которой возникнет резонанс - в этом случае в трубе "поместится" половина длины волны.
Скорость звука должна быть около 330 м/с.
Если частоту, при которой возникает резонанс, трудно точно определить на слух,
поместите в середину трубы микрофонный датчик и, изменяя частоту генератора, смотрите,
когда показания будут максимальны. (Частота записи для такого опыта 25 измерений в
секунду, иначе не получится графика в "реальном" времени. Разумеется, график будет весьма
"рваным"- ведь частота колебаний много выше частоты записи. Но нас интересуют только
максимальные значения этой зависимости!)
Измерим частоту "первого" резонанса, измерим длину трубы (возможно точнее) и
рассчитаем скорость звука. Оценим погрешность измерения скорости звука.
3. Повышая частоту, определим на слух, при каких частотах возникают следующие
резонансы. Запишем эти частоты. Проверим, выполняется ли условие резонанса. Запишем
вывод.
4.Привяжем микрофонный датчик к нитке, чтобы "протащить" его вдоль всей трубы
и узнать, каков уровень звука (амплитуда колебаний давления) в разных местах трубы во
время резонанса.
При установившемся резонансе в трубе "протащим" через трубу привязанный к нити
микрофонный датчик, определим, где пучности, где узлы давления. Цель - получить график
давлений от расстояния (не времени!). Поэтому нужно обеспечить равномерное движение
датчика, например, с использованием моторчика от LEGO . тогда координата датчика в трубе
пропорциональна времени, и график будет "правильным".
(Если такого моторчика нет, можно поступить иначе. Можно было бы подключить датчик расстояния, а к концу нитки
привязать предмет немалых размеров, до которого датчик определял бы расстояние. Но, к сожалению, датчик
расстояния не умеет работать на высоких частотах. А чтобы получить хорошую "картинку" микрофонным
датчиком, нужна именно высокая частота записи, например, 500 в секунду. Сделать же разные частоты для разных
77
датчиков невозможно. Один из интересных, но требующих аккуратности вариантов - взять датчик угла поворота,
обмотать нить (к другому концу которой привязан микрофонный датчик) вокруг шкива (один - два оборота,
чтобы не было проскальзывания) и медленно тянуть нить. Угол поворота будет пропорционален пройденному
расстоянию. Но помните - датчик угла поворота НЕ работает на скоростях выше двух оборотов в секунду! (При
превышении скорости данные будут неверными. Поэтому после опыта проконтролируйте соответствующий
график!)
Построим графики амплитуды колебаний давления в трубе для двух разных частот рекомендуется минимальная частота и в четыре раза бОльшая.
ДОПОЛНИТЕЛЬНО
5. Получим резонанс в трубе с закрытым концом.
(Закроем заслонкой конец, противоположный динамику). Тогда первый резонанс будет при
условии равенства длины трубы четверти длины волны (см. рисунок выше), второй - три
четверти, и т.д. Проверим соответствие теории и практики, запишем выводы.
Получим и распечатаем график зависимости колебаний давления от положения датчика
в трубе. Датчик рекомендуется перемещать от крышки в сторону динамика. Опыт достаточно
выполнить только для одной частоты, например, второго резонанса.
6 Запишем выводы по работе.
А.Е. Тарчевский, шк. 179 МИОО, март 2011
78
ЛАБОРАТОРНАЯ РАБОТА № 22
ИЗУЧЕНИЕ РАСТЯЖЕНИЯ ПРОВОЛОКИ
(пробная)
ОБОРУДОВАНИЕ
Медная проволока диаметром 0,1- 0,3 мм и длиной около 1,5 м, скотч, линейка, NOVA,
датчик силы, тяжелый груз, например, 2 кг, штангенциркуль или микрометр, справочник
"Физические величины". Вариант В- датчик угла поворота.
ЦЕЛИ РАБОТЫ
- узнать, действительно ли металлическая проволока будет заметно растягиваться,
- узнать, как зависит растяжение от действующей на проволоку силы,
- познакомиться с понятием механического напряжения, упругой и неупругой деформации,
предела упругости и прочности.
- Наблюдать явление текучести металла.
ИДЕЯ РАБОТЫ
Будем нагружать проволоку постепенно возрастающей силой и измерять по линейке её
удлинение, затем построим график зависимости удлинения от силы, проанализируем его.
ЭКСПЕРИМЕНТАЛЬНАЯ УСТАНОВКА
Удобнее тянуть не за проволоку, а за
датчик силы, и, например, и через каждый
миллиметр удлинения отмечать, какая
была сила, и сразу заносить данные в
таблицу.
Вариант А: ВИД СВЕРХУ:
На краю стола закрепим проволоку.
Можно воспользоваться струбциной.
Обеспечим, чтобы на проволоке делались
"правильные" узлы. (Например, "восьмёрка",
используемая для альпинистских верёвок. Если
проволока в ходе опыта порвётся в узле, значит узел плохой, придётся переделать опыт.)
На другом краю стола
прикрепим
к
концу
проволоки датчик силы, а
сам датчик прикрепим к
тяжелому грузу, так,
чтобы
растянутая
проволока
не
могла
"утянуть" датчик обратно,
когда мы его отпустим.
Или же у вас получится
проволока
штатив с
грузом
датчик
силы
линейка,
прикреплённая к столу
79
аккуратно, прижимая датчик к столу, сдвигать его на малые расстояния- попробуйте. Чтобы
обеспечить большую длину проволоки, можно поставить два стола один за другим, но при
этом важно обеспечить, чтобы столы не сдвигались в ходе опыта.
Чтобы точно отмечать удлинение проволоки, и ровно передвигать штатив, приклеим скотчем
к столу линейку. Проволока в ходе опыта растянется, поэтому обеспечим нужный запас
длины линейки. Перемещение (удлинение проволоки) можно отслеживать по краю штатива,
движущегося вдоль линейки. Удобно сделать так, чтобы при очень малая но ненулевая) сила
натяжения проволоки соответствовала "нулю" линейки. Можно для этого приклеить к
нужному месту штатива около линейки штативу кусочек изоленты и отслеживать положение
по ней.
Вариант В
Если имеется датчик угла поворота и вы сможете жестко закрепить его на краю стола и обеспечить аккуратное
наматывание нити на вал датчика (нужен совсем маленький диаметр), а к этой нити будет прикреплён датчик
силы - получится автоматизировать процесс измерения расстояния и обработки результатов.
Датчик расстояния не годится из-за малой точности.
Ещё одна "красивая" возможность - использовать для измерения расстояния "ползунковый" реостат с возможно
большей длиной хода, блок питания и датчик напряжения.
Старые "школьные" проволочные реостаты малопригодны из-за перекашивания ползунка.
ПОДГОТОВКА
1. Перед началом опыта убедитесь, что разрывное усилие проволоки не приводит к
"зашкаливанию" датчика силы. Для этого на самом деле порвите датчиком короткий кусочек
проволоки и посмотрите, какова сила при разрыве. Помните- у датчика два разных предела
измерений, переключатель - на корпусе.
2. Соберите экспериментальную установку.
3. Включите компьютер так, чтобы можно было в любой момент снять показания силы
Есть режим снятия показаний прибора при нажатии иконки.
4. Установите "начальную точку"- проволока натянута, а сила - почти нулевая. В этой точке
удлинение будем считать равным нулю.
5. Выполните опыт, сдвигая датчик каждый раз, например, на 1 миллиметр ("правильный"
шаг зависит от длины и свойств взятой проволоки), и каждый раз записывайте данные в
таблицу: сила F, удлинение Δх.
Делайте это до тех пор, пока проволока не порвется.
6. Постройте на миллиметровке график зависимости силы упругости от удлинения
проволоки. Напишите выводы по графику.
Внимание! Типичная ошибка:
Вместо требующегося графика строят график зависимости удлинения от силы.
7. Посмотрите в справочнике "ПРЕДЕЛ ПРОЧНОСТИ" для материала проволоки.
Это механическое напряжение (силу делим на площадь поперечного сечения проволоки),
при котором образец разрушается.
(Из справочника стр. 67: Предел прочности меди σв = 240 МПа )
Для определения диаметра проволоки пользуйтесь микрометром или штангенциркулем.
Запишите результаты измерений, справочные данные, расчёты и вывод.
Соответствуют ли данные справочника полученным экспериментальным результатам?
80
8. По результатам работы рассчитайте:
Сколько (минимум) таких проволочек понадобилось бы, чтобы выдержать вес
экспериментатора?
9 ДОПОЛНИТЕЛЬНОЕ ЗАДАНИЕ
Проведите аналогичное исследование с ниткой.
10. ДОПОЛНИТЕЛЬНОЕ ЗАДАНИЕ:
Надо узнать, при какой силе проволока восстановит прежнюю длину после снятия нагрузки,
а при какой останется растянутой. Такая деформация, когда после снятия нагрузки тело
возвращает прежнюю форму, называется упругой.
Соберите установку, взяв новую проволоку.
Установите "начальную точку"- проволока натянута, а сила - почти нулевая (но есть
какая-то и её можно измерить!). Запишите это значение силы и показание "стрелки".
Нагрузите проволоку некоторой небольшой силой, запишите деформацию и силу, затем
снимите нагрузку до исходного значения. Восстановила ли проволока свою первоначальную
длину? (Конечно, в этом опыте изначально проволока должна быть нагружена маленькой
силой, иначе она будет провисать и опыт не получится).
Повторяя это действие и каждый раз увеличивая силу, определите силу, где деформация
перестала быть упругой. Запишите результаты опыта. Сравните полученную силу с силой
при разрыве образца.
А.Е. Тарчевский, шк. 179 МИОО, март 2011
81
ЛАБОРАТОРНАЯ РАБОТА № 23
ИЗУЧЕНИЕ СВОБОДНОГО ПАДЕНИЯ ТЕННИСНОГО ШАРИКА
ЦЕЛИ РАБОТЫ:
- Получить и исследовать графики координаты, скорости, ускорения тела при свободном
падении.
- Определить величину ускорения свободного падения.
- Исследовать влияние сопротивления воздуха на ускорение при падении шарика.
- Оценить время взаимодействия шарика с плитой и ускорение в момент отскока шарика.
- Исследовать, какая часть энергии переходит в тепло при каждом отскоке.
- Познакомиться с характером проблем, возникающих при регистрации данных с
недостаточной частотой измерений, и способами их решения.
ОБОРУДОВАНИЕ:
- шарик для настольного тенниса,
(можно взять и резиновый прыгающий мячик, но с шариком получается лучше)
- штатив для крепления датчика, желательно - струбцина для крепления штатива к столу
(если её нет, можно использовать в качестве груза стопку книг)
- NOVA-5000,
- датчик расстояния,
- ровная массивная доска (плита из ДСП), чтобы шарик хорошо и ровно отскакивал.
штатив
датчик
расстояния
y
груз
стол
шарик
плита
0
ТЕОРИЯ:
Свободным падением называется движение тела в инерциальной системе отсчёта только под
действием силы тяжести.
Ускорение тела g при таком движении называется ускорением свободного падения.
Пусть координата тела в точке начала движения y0, начальная скорость равна v0, ускорение a.
Тогда уравнения для координаты и проекции скорости на ось у имеют вид:
y(t) = y0 +v0yt + ayt2/2
vy (t) = v0y + ayt
82
В этой работе при помощи датчика расстояния мы будем измерять координату шарика при
движении его вниз и вверх (после отскока от пола). По зависимости y(t) можно определить
величину ускорения свободного падения.
ВНИМАНИЕ: Берегите шарик, не давайте ему кататься по кабинету!
ПОДГОТОВКА ОБОРУДОВАНИЯ:
1. Соберите экспериментальную установку в соответствии с рисунком.
- Закрепите датчик расстояния на штативе. (Не надо пользоваться лапкой, просто
укрепите стержень, к которому прикреплён датчик, в зажиме штатива вместо лапки.)
- Установите штатив на краю стола и прикрепите его струбциной или положите на штатив
стопку книг.
- Обеспечьте наличие ровной плиты под местом падения шарика.
- Сделайте так, чтобы на полу рядом с местом падения не было посторонних предметов,
которые могут помешать датчику правильно определять положение шарика.
- Установите датчик на высоте 100 – 140 см от поверхности пола. (Большие расстояния более
интересны, но если расстояние слишком велико, датчик перестаёт "видеть" шарик и даёт
странные результаты, тогда график y(t) получится "рваным". Датчик должен "смотреть"
строго вниз. (Проверьте это!)
- Подключите датчик расстояния.
- Подключите компьютер к блоку питания, иначе датчик расстояния быстро разрядит
аккумуляторы.
2. Для предварительной настройки экспериментальной установки попробуйте несколько раз
отпустить шарик без начальной скорости, не включая запись. Удобно отпускать шарик из
положения непосредственно под датчиком, но не забывайте, что все расстояния, меньшие 20
сантиметров, датчик покажет как 20 сантиметров. Потренируйтесь отпускать шарик так,
чтобы он падал строго под датчиком и успевал подпрыгнуть и снова упасть хотя бы 3-5 раз,
находясь под датчиком или почти под датчиком. Возможно, для этого потребуется чуть
изменить наклон плиты, от которой отскакивает шарик.
3.1 ПРИНЦИП РАБОТЫ ДАТЧИКА И ВЫБОР ЧАСТОТЫ ИЗМЕРЕНИЙ.
Датчик посылает импульс ультразвука, звук отражается от шарика и возвращается в датчик.
По времени задержки определяется расстояние. Скорость звука 330 м/с. Датчик рассчитан на
измерение расстояний до 10 метров. Звук пройдёт 10 метров до объекта и 10 обратно. На
этом расстоянии задержка составит 20/330 ≈ 0, 06(с). Тогда за секунду датчик мог бы
выполнить не более 16 замеров. Согласно паспорту на больших расстояниях датчик способен
делать 10 замеров в секунду, на малых- 25.
Очевидно, чем больше частота измерения, тем качественнее получатся наши графики.
Если расстояние 1,5 метра, время похождения сигнала 1,5*2/330 = 0.009 (c), тогда датчик
теоретически мог бы выполнить 110 замеров в секунду.
3.2 НУЖНАЯ ЧАСТОТА ИЗМЕРЕНИЙ ДЛЯ ВЫПОЛНЕНИЯ РАБОТЫ.
Шарик, падающий с высоты 1.25 метра, имеет скорость √2gh = √(2*10*1.25) = 5 (м/с). Если
частота измерений 25 в секунду, шарик успеет пролететь за время между соседними
измерениями его положения 5/25 = 1/5 метра! Частота 100 измерений в секунду не решила
бы всех проблем, но, по крайней мере, это расстояние снизится до 5 сантиметров.
3.3 ВЫБИРАЕМ ЧАСТОТУ
На самом деле все датчики должны надёжно работать на частоте 25 замеров в секунду (но
этого явно мало для нашей работы).
Стоит постараться сделать частоту 100 измерений в секунду – но, чтобы получить красивый
график без сбоев, вероятно, придётся набраться терпения и выполнить опыт много раз.
83
Не получится 100- попробуйте установить 50 замеров в секунду.
4. ВЫПОЛНЕНИЕ ОПЫТА
4.1 НАСТРОЙКИ: Длительность – непрерывно, в конце опыта надо вовремя остановить
запись, когда шарик "убежит" из- под датчика. Частота -25 записей в секунду.
4.2 Проведите пробный опыт с регистрацией данных на частоте 25 записей в секунду.
Убедитесь, что датчик нормально отслеживает положение шарика, что получается график, на
котором нормально получились хотя бы три-четыре отскока.
4.3 Если идут сбои, попробуйте:
- выполнить опыт ещё несколько раз,
- проверить, нет ли выступающих посторонних предметов, например, ног экспериментатора,
которые "ловит" датчик,
- проверить, "смотрит" ли датчик строго вниз,
- поставить датчик немного ближе к плите, на которую падает шарик (это хорошо помогает),
- уменьшить частоту записи (но не ниже 25 измерений в секунду),
- если ничего не помогает, взять другой датчик и другой компьютер.
4.4 Если опыт получается, попробуйте увеличить частоту записи до 100 или хотя бы 50
измерений в секунду, тогда ваши данные будут намного более качественными.
Этот опыт при должной аккуратности и терпении даёт "красивый" график, похожий на
приведённый ниже (график будет "перевёрнутым"- это нормально).
Обязательно сохраните результат удачного эксперимента, дайте ему имя. Можно провести
опыт ещё несколько раз, чтобы потом выбрать из данных наиболее удачный вариант. Не
забывайте сохранять результаты. Над зависимостью придётся много работать, а NOVA часто
"зависает" при сложной обработке результатов.
ОБРАБОТКА И АНАЛИЗ РЕЗУЛЬТАТОВ
ПОЛУЧЕНИЕ ГРАФИКА КООРДИНАТЫ
1. Мы получили зависимость координаты шарика от времени при свободном падении и
последующих отскоках от плиты. В идеальном случае получится что-то похожее на график
на этом рисунке.
Конечно, может быть и меньше отскоков.
84
2. Вероятно, ваш график получился перевёрнутым, но его легко привести к указанному виду.
Надо умножить функцию на минус 1 и что-то прибавить, чтобы в момент падения
координата была нулевой.
Прибавить надо самое "нижнее" значение, зафиксированное при различных отскоках шарика
(конечно, следите, чтобы в этом месте не было сбоя). Обычно эти значения достигаются при
невысоких скоростях (правая часть графика), что легко увидеть и объяснить. Конкретное
значение можно узнать, поставив на него курсор. Сделать это надо весьма аккуратно, т.к. на
основе этих данных мы скоро оценим длительность удара шарика о поверхность.
Другой, более "честный" способ - непосредственно измерить координату "самого нижнего
положения" шарика датчиком расстояния. Поскольку датчик плохо определяет расстояние
до неподвижных объектов, шарик надо положить сверху на тонкую проволочку (в конце
свёрнутую колечком) и, включив измерения, аккуратно опустить от датчика вниз, на место
его обычного падения.
Иконка fx – Операция- Линейная- будет предложено ввести коэффициенты для
преобразования AG1+B. Здесь G1- ваш график, А- то, на что вы его умножаете (надо ввести 1 в поле для А), в поле В - то, что к нему прибавляете. Надо прибавить такое В, чтобы график
в самой нижней точке приходил в ноль. Это соответствует выбору оси y, показанной на
рисунке.
3. Выберите "интересный" участок (включающий несколько отскоков без сбоев) лучшего из
полученных графиков. С ним мы дальше и будем работать.
Выберите хороший масштаб и правильные единицы измерения.
Инструменты- Единицы измерения - для оси "время"- лучше выбрать секунды, знаков - два.
Для функции по оси y желательно выбрать приставку "милли" (м) – тогда расстояние будет в
миллиметрах.
Подпишите этот график. (Рекомендуется подключить клавиатуру). Укажите также
частоту измерений и свою фамилию.) Нажмите иконку "Сохранить".
Распечатайте этот график (при печати с NOVA установите высокое качество печати),
покажите его учителю. Будьте готовы объяснить вид графика.
Замечание: Если в окне одновременно выведено несколько графиков и
преобразование, которое вы делаете, выполняется не над нужной функцией, надо в окне
слева сверху от графиков указать имя нужного графика!
ОПРЕДЕЛЕНИЕ УСКОРЕНИЯ
ОПРЕДЕЛЕНИЕ СООТВЕТСТВИЯ ГРАФИКА у(t) ПАРАБОЛЕ
4. Выберите с помощью двух курсоров (иконка внизу, сначала надо вызвать один курсор,
затем – два курсора) первый участок, соответствующий свободному падению шарика.
Участок, разумеется, не должен включать момента удара. Обратите внимание на искажения
графика около момента удара (точек минимальной высоты) из-за недостаточной частоты
записи, добейтесь, чтобы искаженные части графика НЕ попали в выделенный фрагмент.
Нажмите иконку "Сохранить"- график может пригодиться, чтобы показать учителю.
5. Аппроксимируйте выделенный график параболой. (Инструменты - Анализ - Квадратичное
приближение). Чтобы было видно, где какой график, выведите на исходный график
экспериментальные точки. Для этого нажмите иконку "Свойства графика" внизу, выберите
нужный график (не "время"), поставьте галочку в меню "значок - виден". Если опция не
работает, попробуйте одну из зависимостей вывести чёрным цветом, а другую- светлосерым.) Распечатайте выделенный фрагмент крупно с хорошим качеством. Посмотрите,
действительно ли график хорошо описывается параболой.
Распечатайте один из графиков с аппроксимирующей параболой в наглядном масштабе с
высоким качеством. Захватите небольшие кусочки соседних участков – они потребуются
для оценки длительности удара. Внизу вы увидите уравнение полученной параболы. (Только
оно в странном виде: f(x) = -4.86 x^2- 7.080x -1.89 Здесь ^ - символ возведения в степень.
Заменяем x на t, f(x) на у(t) и получаем искомое уравнение). Запишите его: y(t)=….
85
Соответствующий член уравнения - это ay/2. Запишите полученное значение g на
соответствующем(!) месте общего обзорного графика.)
Замечания:
- Если уравнение не выводится, поставьте курсор на график параболы.
- Относитесь к полученному уравнению критически, при аппроксимации часто получаются
явно абсурдные уравнения координаты - видимо, недостаток связан с программой Multilab.
- Если ошибка не устраняется, можно попробовать немного сдвинуть курсоры или
презапустить программу.
6. Исследуйте полученное приближение, оцените, насколько соответствует параболе
реальный график зависимости y(t).
ОЦЕНКА ДЛИТЕЛЬНОСТИ УДАРА
7. В пункте 2 мы не зря тщательно искали самую
нижнюю точку при падении шарика. Теперь на
распечатанном графике карандашом продолжим
соседние участки аппроксимирующих парабол до
пересечения с "нулём". Получим интервал времени, в
течение которого шарик "лежал" на поверхности
плиты. Запишем его значение. Понятно, что не
получится сделать это очень точно. Должно
получиться что-то около 0,005 секунды.
ОПРЕДЕЛЕНИЕ УСКОРЕНИЯ ДЛЯ ДРУГИХ
УЧАСТКОВ
8. Аналогично обработайте и несколько других
участков (отскоков шарика). Графики для них
строить не надо. Запишите полученные значения g
на соответствующих(!) местах общего обзорного
графика и в таблицу.)
9. Найдите среднее значение ускорения.
ИССЛЕДОВАНИЕ ВЛИЯНИЯ СОПРОТИВЛЕНИЯ ВОЗДУХА НА УСКОРЕНИЕ
10. Прочитайте:
На тело, движущееся в воздухе, кроме силы тяжести действует сила сопротивления,
направленная против скорости, тем большая, чем больше скорость тела. Если тело падает
вниз, ускорение из-за сопротивления должно получиться чуть меньше (сила тяжести и сила
сопротивления действуют в разные стороны), а при движении вверх - чуть больше т.к. сила
тяжести и сила сопротивления действуют в одну сторону. Тогда шарик должен бы взлетать
быстрее, чем падать. В этом случае наш график должен получится чуть "перекошенным"смещённым влево от аппроксимирующей кривой. Если это действительно таксопротивление воздуха влияло на ускорение заметно.
11. Конечно, заманчиво выбрать участок падения и рядом - участок взлёта (только не до
верхней точки, а от самого "низа" до "серединки" по высоте, чтобы скорость влияла
заметнее), выполнить квадратичное приближение и определить ускорение на каждом из
участков. Затем надо получить значения ускорений на многих других участках и потом
сделать вывод, действительно ли ускорение при движении вниз и вверх, заметно отличаются.
Если да, значит, что сопротивление воздуха влияет на ускорение шарика.
Но, к несчастью, NOVA-5000 упорно "глючит" при попытках сделать так. Но если вам
повезёт, всё же сделайте такое исследование и сравните значения ускорения при движении
вниз и вверх.
86
Если получается, найдите хотя бы по трём-четырём участкам подъёма среднее значение
ускорения. Найдите среднее значение ускорения по нескольким участкам падения.
Запишите вывод о зависимости ускорения шарика от направления движения.
ПОЛУЧЕНИЕ ГРАФИКА СКОРОСТИ.
12. Для этого проведите дифференцирование графика координаты.
(Можно пользоваться иконкой "Производная"
в верхнем меню.)
Вырежьте из графика "интересный" участок. Получится что-то похожее на рисунок внизу.
(Правда, здесь график скорости показан точками, а у вас точки могут быть соединены
линией.)
Убедитесь, что зависимость скорости шарика
от времени на участках свободного падения
близка к линейной.
Сделайте правильный масштаб, подпишите и
распечатайте этот график.
12. Выделите двумя курсорами один из линейных участков, аппроксимируйте его прямой
(в верхнем меню иконка - Линейное приближение). Внизу появится уравнение скорости.
Запишите его (в нормальном стандартном виде vy (t) = v0y + ayt. Соответствует ли оно
тому, что ожидалось?
13. Напишите в соответствующих местах графика значения скорости непосредственно
перед каждым ударом и после удара. Это просто – прикладываем линейку, карандашом
проводим линию, наилучшим образом аппроксимирующую линейный участок графика,
смотрим на оси значение скорости в точке, соответствующей моменту касания шариком
поверхности, записываем его на графике. Не надо "снимать" значение скорости курсором.
Умейте объяснить вид этого графика.
ПОЛУЧЕНИЕ ГРАФИКА УСКОРЕНИЯ
14. Получите график "ускорения". Для получения ускорения надо взять производную от графика
скорости (иконка
вверху).
Кавычки - потому что это, конечно, не график ускорения, а что-то похожее на него. Проблема в том,
что из-за недостаточной частоты записи программа "не знает", сколько времени на самом деле длился
удар, и существенно завышает это время. Кроме того, на графике есть искажения- "шумы", а
компьютер принимает их за реальные изменения скорости. При этом сглаживать график нельзя, иначе
длительность моментов удара определится ещё с большей ошибкой.
Выделите интересный фрагмент, сделайте хороший масштаб, подпишите и распечатайте
его. Какие приблизительно ускорения были у шарика в момент удара в соответствии с графиком? А
во время свободного полёта?
15. Нам известна скорость перед первым падением, скорость сразу после отскока и время
взаимодействия шарика с плитой. Рассчитаем (оценим) среднее значение ускорения, напишем его в
соответствующем месте графика ускорения. Сделаем то же самое для нескольких следующих
отскоков.
87
ИССЛЕДУЕМ ПОТЕРИ ЭНЕРГИИ ПРИ ОТСКОКЕ ШАРИКА
Пронумеруйте отскоки шарика на графике.
Прочитайте, выполните и заполните таблицу внизу текста
16. Первый способ: Зная скорости до и после отскока, рассчитайте отношение кинетической энергии
до и после отскока шарика для первого, второго, третьего четвёртого (а лучше и пятого и т.д.)
отскоков.
17. Второй способ: Зная высоту, на которую поднимается шарик после каждого отскока, легко
рассчитать то же самое, ведь энергия в наивысшей точке подъема равна mgh. Высоту можно "снять" с
графика при помощи курсора, или, (хуже), измерить линейкой на "бумажном" графике. Поскольку
рассчитывается отношение высот, можно взять высоту в "миллиметрах графика". Разумеется,
отмерять надо от самого нижнего положения шарика в опыте.
18. Третий способ: Время, которое проводит шарик в полёте между отскоками, зависит от его
скорости. vy (t) = v0y - gt. В наивысшей точке подъёма vy(tподъема) = 0, следовательно, время подъёма
tподъема= v0y/g. Время падения такое же. Таким образом, время полёта пропорционально скорости,
следовательно, квадрат времени полёта пропорционален энергии. Рассчитайте потери энергии,
используя этот способ. Поскольку нас интересует отношение квадрата времени на одном участке к
квадрату времени на другом участке, время можно "снимать" с графика линейкой в миллиметрах, но,
конечно, более точно использовать курсор на экране.
Составьте таблицу:
Отношение энергии после отскока к энергии до отскока:
по скоростям
по высоте
по времени
первый отскок
второй
третий
четвёртый
…
…
Одинаковый ли процент механической энергии теряется при каждом отскоке шарика?
А.Е. Тарчевский, шк. 179 МИОО, март 2010
Использованы идеи и график с сайта
http://www.fourier-sys.com/pdfs/new_experiments/nova_physics/measuring_g_the_free_fall_acceleration.pdf
88
ЛАБОРАТОРНАЯ РАБОТА № 24
(ПРОБНАЯ)
ИЗУЧЕНИЕ КОЛЕБАНИЙ МАТЕМАТИЧЕСКОГО МАЯТНИКА
ОБОРУДОВАНИЕ:
Датчик
угла
поворота,
переходник, позволяющий укрепить датчик силы на
оси датчика угла поворота, датчик силы, шарики
разной массы на проволоке (хорошо, если тяжелый
шарик имеет массу 300 г), штатив, струбцина,
линейка, NOVA-5000, маленький сильный магнит,
лист толстого металла (меди или алюминия) – для
исследования затуханий.
УСТАНОВКА (см. рисунок). Датчик угла поворота
служит
для
отслеживания
колебаний
угла
отклонения маятника от вертикали, кроме того,
обеспечивает малое трение в оси за счёт
подшипников. Для измерения натяжения проволоки
во время колебаний применяется датчик силы,
закреплённый на оси датчика угла поворота. При
этом
используется
пластмассовая
трубочкапереходник, т.к. ось несколько коротковата. Кроме
того, датчик силы надо прикрепить НЕ с той
стороны, где подключается провод, поэтому
придётся снять пластмассовый блок с оси (и
положить его и крепежный винт в надёжное место).
Кабель к датчику силы подсоединяется только в тех
опытах, где измеряется сила натяжения. Когда сила не
измеряется кабель надо отсоединить, чтобы
уменьшить затухание колебаний. Чем длиннее
проволока, тем точнее будут измерения.
ВНИМАНИЕ:
- Обеспечьте надёжное крепление датчиков и штатива.
- Добейтесь, чтобы датчики "не болтались", не тёрлись
друг о друга, чтобы кабели не мешали проведению
опыта (особенно кабель датчика силы - его надо
закрепить в верхней лапке штатива, не показанной на
фотографии).
- Не перегружайте датчик угла поворота – не
подвешивайте груз массой более 400 г., не отклоняйте
маятник на слишком большие (близкие к 90 0) углы.
- Установите предел измерений датчика силы 10Н.
- Если топать по полу и касаться стола во время опыта с
измерением силы натяжения проволоки, на графике появятся кратковременные "пики".
ЦЕЛИ РАБОТЫ:
- Наблюдение колебаний угла отклонения маятника от вертикали и силы натяжения
проволоки.
- Сравнение силы натяжения проволоки в нижней точке с расчётным значением.
- Исследование зависимости периода колебаний от амплитуды.
- Определение ускорения свободного падения.
89
- Исследование зависимости периода колебаний от массы маятника.
- Исследование зависимости периода колебаний от длины маятника.
- Наблюдение затухающих колебаний и определение коэффициента затухания.
- Наблюдение апериодического процесса.
- Дополнительно- изучение физического маятника.
РАБОТА МОЖЕТ БЫТЬ ВЫПОЛНЕНА И СДАНА ПО ЧАСТЯМ
ПО СОГЛАСОВАНИЮ С УЧИТЕЛЕМ.
ХОД РАБОТЫ:
0. ВСПОМНИТЕ:
Т- период колебаний (время, за которое совершается одно полное колебание).
Циклическая частота (количество колебаний за 2π секунд) ω = 2π/T.
Частота (количество колебаний в единицу времени) ν = 1/T = ω/2 π
1. НАБЛЮДЕНИЕ КОЛЕБАНИЙ УГЛА ОТКЛОНЕНИЯ МАЯТНИКА ОТ ВЕРТИКАЛИ И
СИЛЫ НАТЯЖЕНИЯ ПРОВОЛОКИ.
- Подключите датчик, угла поворота, установите частоту записи 25 измерений в секунду, (на
большей частоте не будет выводиться график в реальном времени) длительность непрерывно.
- Проверьте, работает ли установка, ровно ли колеблется маятник, нет ли трения, не мешают
ли кабели.
- Датчик угла поворота считает "нулевым" тот угол, который был при запуске опыта.
Поэтому перед началом опыта маятник должен висеть, не качаясь. Затем запускаем запись,
после этого отклоняем маятник на нужный угол и отпускаем его.
(Впрочем, данное действие не обязательно, можно, но сложнее, привести график "к нулю"
уже после завершения опыта при помощи "мастера анализа". Угол равен нулю в тот момент,
когда маятник проходит положение равновесия, при этом сила натяжения максимальна.)
- Запустите опыт, наблюдайте колебания. (Отклонение от вертикали не надо делать большим10-15 градусов вполне достаточно).
- Выделите фрагмент графика, соответствующий одному- двум колебаниям. Убедитесь, что
полученный график угла отклонения – синусоида φ = φ0 sin ωt, где ω= 2π/T.
Распечатывать этот график не надо.
2. СРАВНЕНИЕ СИЛЫ НАТЯЖЕНИЯ ПРОВОЛОКИ В НИЖНЕЙ ТОЧКЕ С РАСЧЁТНЫМ
ЗНАЧЕНИЕМ.
- Подключите датчик силы. Убедитесь, что кабель датчика не мешает колебаниям.
- Зная максимальный угол отклонения маятника от вертикали и силу натяжения нити для
неподвижно висящего маятника, нетрудно рассчитать силу натяжения в нижней точке.
Нарисуйте рисунок с указанием сил, выведите соответствующую формулу. (Примените
закон сохранения энергии для нахождения скорости, II закон Ньютона для расчёта силы
натяжения.) - Объясните, почему происходят колебания силы натяжения, почему их
частота вдвое больше частоты колебаний маятника.
- Выполните три измерения силы:
- неподвижно висящий маятник (в покое – просто найдём и запишем mg )
- отклоните маятник на угол около 20 градусов, запустите измерения, отпустите
маятник, дайте сделать ему два-три колебания, остановите опыт.
- то же самое для угла отклонения около 60 градусов.
При помощи курсора получите нужные данные и заполните таблицу для двух
различных углов отклонения со столбцами:
90
Длина маятника, м :
Сила натяжения в покое, Н :
Максимальный угол отклонения от
вертикали, рад
максимальная сила натяжения (расчёт), Н
максимальная сила натяжения (опыт) , Н
- Распечатайте (только для одного угла отклонения маятника) на одном листе два графика
зависимости угла отклонения и силы натяжения нити от времени (выведите на график ось
ординат для силы) для приблизительно двух полных колебаний. Не забудьте подписать
график и обеспечить хороший масштаб.
3 ИССЛЕДОВАНИЕ ЗАВИСИМОСТИ ПЕРИОДА КОЛЕБАНИЙ ОТ АМПЛИТУДЫ.
- Отключите кабель от датчика силы.
- Чтобы точнее определить период, надо посмотреть время, за которое произошло не одно, а
несколько колебаний. Подумайте, сколько надо взять колебаний, чтобы ошибка определения
периода была достаточно малой.
Можно определять период по прохождению графика через "ноль", или по моменту, когда
отклонение максимально. (Внимание! За время полного колебания маятник дважды
проходит "через ноль".)
Исследуйте зависимость периода от амплитуды минимум для четырёх различных амплитуд –
например, около 5, 15, 30, 60 градусов. Запишите результаты и вывод. Теория утверждает,
что период малых колебаний не зависит от амплитуды. До каких углов отклонения колебания
можно считать "малыми"?
Внимание: Типичная ошибка:
Не надо определять период по времени одного только полного колебания. Чтобы точность была
высокой, возьмём, например, 10 или 30 колебаний. Тогда и погрешность определения периода
уменьшится в 10 или 30 раз!
4. ОПРЕДЕЛЕНИЕ УСКОРЕНИЯ СВОБОДНОГО ПАДЕНИЯ.
Воспользуйтесь результатами предыдущего опыта для малого отклонения маятника и
найдите ускорение свободного падения, измерив длину маятника (от оси до цента груза) и
пользуясь известной формулой для периода колебаний.
Определите погрешность найденного значения g.
5. ИССЛЕДОВАНИЕ ЗАВИСИМОСТИ ПЕРИОДА КОЛЕБАНИЙ ОТ МАССЫ МАЯТНИКА.
Используйте груз другой массы (более лёгкий) на проволоке той же длины.
(Возьмите другую проволоку для опыта, обеспечьте ту же длину (до центра шарика).
Определите период малых колебаний, запишите результаты и выводы.
6. ИССЛЕДОВАНИЕ ЗАВИСИМОСТИ ПЕРИОДА КОЛЕБАНИЙ ОТ ДЛИНЫ МАЯТНИКА.
Уменьшите длину маятника. Измерьте период. Проверьте соответствие опыта и расчёта
по формуле. Запишите вывод.
7 НАБЛЮДЕНИЕ ЗАТУХАЮЩИХ КОЛЕБАНИЙ С МАЛЫМ ЗАТУХАНИЕМ.
Перед началом опыта маятник должен висеть, не качаясь. Затем запускаем запись, после
этого отклоняем маятник на угол около 15 градусов и отпускаем его. Записывайте колебания
несколько минут, до тех пор, пока амплитуда не уменьшится значительно.
91
Постройте огибающую полученного графика при помощи мастера анализа, если надо выполните её сглаживание иконкой + .
Полученная кривая показывает уменьшение амплитуды колебаний с течением времени.
По согласованию с учителем (для учащихся 9 класса) можно не выполнять всех действий
этого и следующего пункта, а только понаблюдать затухающие колебания и распечатать
графики.
Для незатухающих колебаний верна зависимость: φ = φ0 sinωt . При затухании колебаний (в
предположении, что сила сопротивления пропорциональна скорости), выводят зависимость:
φ= φ0 e-βt sin ω1t, ω1 = (ω2 – β2)½ , т.е. амплитуда экспоненциально уменьшается, а частота
для малого затухания чуть меньше, чем у незатухающих колебаний. (Вывод этих формул
требует знания высшей математики и здесь не приведён.)
Запишите уравнение затухающих колебаний.
Аппроксимируем полученную кривую экспонентой (Мастер анализа - АппроксимацияЭкспонента). Если график удаётся хорошо приблизить экспонентой, подтверждается
предположение, что сила сопротивления для маятника пропорциональна скорости.
ВНИМАНИЕ: В Multilab при логарифмировании или аппроксимации полученной зависимости
экспонентой часто возникают проблемы –график полученной зависимости явно "глючный" (как на
NOVA-5000, так и на компьютере), устранить их не удаётся. При этом получившиеся
зависимости- действительно экспоненты (проверяли). Один из возможных путей- экспорт
таблицы из Multilab в Excel и дальнейшая обработка данных в Excel.
Величина β называется коэффициентом затухания. Амплитуда колебаний уменьшится в e раз
за время t = 1/ β.
Если не удалось выполнить аппроксимацию экспонентой из-за "глюков" программы,
распечатайте крупный график (зависимость амплитуды колебаний от времени), посмотрите,
за какое время амплитуда уменьшится в е = 2,72 раз (и найдите β). Убедитесь, что за
следующий такой же промежуток времени амплитуда уменьшится ещё в е раз, затем ещё…
и.т.д. Таким образом вы докажете, что полученная зависимость - экспонента.
Запишите коэффициент затухания.
Распечатайте полученный график. Будьте готовы обсудить результаты с учителем.
8 НАБЛЮДЕНИЕ ЗАТУХАЮЩИХ КОЛЕБАНИЙ СО
ЗНАЧИТЕЛЬНЫМ ЗАТУХАНИЕМ.
Для этого надо прикрепить к грузу снизу сильный маленький
магнит, а внизу расположить медный (алюминиевый
толстый) лист (см. рисунок). Затухание удобно регулировать
изменением расстояния от груза до этого листа.
Выполните опыт при двух- трёх разных затуханиях - от
небольшого до такого, при котором маятник остановится
после нескольких колебаний. Постройте
интересные
графики.
Выполнив действия, как и в предыдущем пункте, запишите
коэффициенты затухания
92
ЗАКРЫТЫЙ
КОНЕЦ
Рассчитайте по данным опыта с сильным затуханием период колебаний. Обратите
внимание на увеличение периода из-за большого затухания.
Проверьте соответствие экспериментально измеренного периода и рассчитанного по
формуле ω1 = (ω2 – β2)½ , Т1= 2π/ ω1, запишите вывод.
Будьте готовы обсудить результаты с учителем.
ОТКРЫТЫЙ
КОНЕЦ
9. Заметим, что при ω = β частота колебаний ω1 = 0, т.е. процесс становится
апериодическим. (Затухает, не успев совершить ни одного полного колебания). Попробуйте
пронаблюдать такой процесс, если это позволит установка.
10 ДОПОЛНИТЕЛЬНОЕ ЗАДАНИЕ
Установка вполне подходит и для исследования
колебаний физического маятника. Успехов!
А.Е. Тарчевский, шк. 179 МИОО, март 2011
93
Download